Lippincott CN

Pataasin ang iyong marka sa homework at exams ngayon gamit ang Quizwiz!

A 42-year-old man has a lymph node biopsy in the left side of his lateral cervical region or posterior triangle of his neck. After closure of the wound, the physician asks the patient to rotate his head to the right against resistance. What nerve is the doctor assessing with this test? (A) Hypoglossal (B) Accessory (C) Mandibular division of trigeminal (D) Great auricular (E) Upper trunk of the brachial plexus

The answer is B: Accessory. The accessory nerve (CN XI) passes deep to the sternocleidomastoid muscle and to the investing layer of deep cervical fascia and courses postero- laterally into the lateral cervical region (posterior triangle of the neck). Due to its superficial course, it is at risk during a lymph node biopsy, cannulation of the internal jugular vein, carotid endartectomy, stab wounds, or removal of a mela- noma in the lateral cervical region. CN XI innervates only the sternocleidomastoid muscle and the trapezius muscle, which are primarily involved in head and shoulder movements. Specifically, the left sternocleidomastoid muscle contracts to bring the mastoid process of the temporal bone closer to the sternum, which results in tilting the head toward the left side and elevation of the chin to the right. The elevation of the chin to the right is the action being tested by this physi- cian, so the accessory nerve is the correct answer. Choice A (Hypoglossal nerve) is incorrect. The left hypoglossal nerve (CN XII) is located in the anterior cervical region, so it would not be damaged by a lymph node biopsy in the lateral cervi- cal region, or posterior triangle of the neck. Moreover, CN XII innervates all of the intrinsic muscles of the tongue and most of its extrinsic muscles with the lone exception being the palatoglossus muscle, innervated by the vagus nerve (CN X). Therefore, damage to the left CN XII would cause ipsi- lateral deviation of the tongue during protrusion and cause dysarthria, or difficulty speaking. In this patient, the doctor is testing the rotation of the head to the right, so the hypo- glossal nerve is not being assessed. Choice C (Mandibular division of trigeminal) is incorrect. The mandibular (third) division of the trigeminal nerve (CN V3) is the only division of the trigeminal nerve (CN V) that supplies motor innerva- tion. It supplies the muscles derived from the mesoderm of the first pharyngeal arch, including the four muscles of mas- tication (temporalis, masseter, lateral pterygoid, and medial pterygoid) and four additional muscles: Mylohoid, Anterior belly of the Digastric, Tensor Tympani, and Tensor Veli Pala- tini (mnemonic = "MATT"). These muscles play no role in rotation of the head, and CN V3 is not located in the lateral cervical triangle where the biopsy occurred. Choice D (Great auricular) is incorrect. The great auricular nerve does course through the lateral cervical region; however, it is a cutaneous (sensory) nerve supplying the skin over the mastoid process of the temporal bone, parotid gland, and auricle. It does not provide any motor innervation, so this nerve would not be involved with rotation of the head to the right side. Choice E (Upper trunk of the brachial plexus) is incorrect. The upper trunk of the brachial plexus is comprised of the anterior (ven- tral) rami of C5 and C6, and it emerges between the anterior and middle scalene muscles in the lateral cervical region (pos- terior triangle of neck). However, damage to the upper trunk of the brachial plexus would result in an abnormal postural presentation of the upper limb known as the "waiter's tip" deformity (or Erb-Duchenne palsy). The upper trunk of the brachial plexus is not involved with rotation of the head to the right side.

A 23-year-old man was punched in the left eye in a bar fight, which resulted in periorbital edema and ecchymosis. In the ER, the man refuses to open his eye, and when his eyelids are pried opened he exhibits vertical diplopia, specifically when asked to look up. A coronal CT reformat image reveals asymmetry in the left orbit and the superior antrum of the maxillary sinus. What is the most likely cause of the patient's diplopia? (A) Paralysis of lateral rectus muscle (B) Entrapment of inferior rectus muscle (C) Detached retina (D) Paralysis of superior oblique muscle (E) Damage to infraorbital nerve

The answer is B: Entrapment of the inferior rectus muscle. The coronal CT (computed tomography) reformat image reveals a fractured orbital floor and the downward herniation of orbital contents into the superior antrum of the left maxillary sinus, leading to the entrapment of the inferior rectus muscle. In this CT, there is no visibility of the inferior rectus muscle within the left orbit because this extraocular muscle and orbital fat have herniated through the acquired defect in the floor of the orbit. The displacement, and subsequent impingement, of the infe- rior rectus muscle within the fractured floor of the orbit causes the vertical diplopia (double vision) in this patient. In this patient, this orbital ("blowout") fracture was caused by the traumatic blow to the left eye. Choice A (Paralysis of the lateral rectus muscle) is incorrect. The lateral rectus muscle of the orbit is innervated by the abducent nerve (CN VI), and it is the only muscle innervated by CN VI. Paralysis of this muscle would lead to horizontal diplopia as the left pupil would be resting in the adducted position due to the unop- posed action of the other extraocular muscles. However, due to blowout fracture site in the floor of the orbital rim and the patient exhibiting vertical diplopia, this muscle is not likely to be damaged due to its lateral location within the orbit. Choice C (Detached retina) is incorrect. A detached retina is often seen following trauma due to inflammation that pulls the sensory retina away from the retinal pigment epithe- lium. However, retinal detachment would lead to vision loss and blindness, not the vertical diplopia seen in this patient. Choice D (Paralysis of superior oblique muscle) is incorrect. The superior oblique muscle of the orbit is the only muscle innervated by the trochlear nerve (CN IV). This muscle would pull the eye inferolaterally, but it is clinically tested by ask- ing the patient to look down after the eye is placed in an adducted position. Paralysis of the superior oblique muscle would lead to diplopia as well as clumsiness when descend- ing stairs. However, due to the blowout fracture site in the floor of the orbital rim, this muscle is not likely to be dam- aged due to its superior location within the orbit. Choice E (Damage to infraorbital nerve) is incorrect. The infraorbital nerve, a branch of the second (maxillary) division of trigemi- nal nerve (CN V2), courses through the superior aspect (roof) of the maxillary sinus, and due to its location, it would be the most likely damaged nerve during the blowout fracture of the inferior floor of the orbit. However, the infraorbital nerve sup- plies sensory innervation to the maxillary sinus as well as the skin of the inferior eyelid, lateral nose, and upper lip. Damage to the infraorbital nerve causes paresthesia and numbness in the areas of cutaneous (sensory) distribution for this nerve; however, damage to this nerve would not cause the vertical diplopia seen in this patient.

A 16-year-old girl experiences mild ptosis and miosis (pupillary constriction) in her right eye following resection of a lymphangioma from the apex of her right lung. Vision in each eye is normal. These findings are most likely due to a lesion involving which structure on the right? (A) Cervical posterior roots (B) Thoracic posterior roots (C) Thoracic anterior roots (D) Thoracic posterior primary rami (E) Thoracic gray rami communicantes

The answer is C: Thoracic anterior roots. This patient is exhibiting miosis (constriction of the pupil) and ptosis (drooping of the eyelid), which are cardinal signs/symptoms of a loss of sympathetic fibers to the head (Horner syndrome). The only structure in this list that carries sympathetic fibers to the head is the anterior roots of the thoracic spinal nerves (T1-3). These anterior roots convey presynaptic sympathetic neurons to the T1-3 mixed spinal nerves, enter the sympa- thetic chain ganglia via the white rami communicantes, and ascend to synapse in the superior cervical ganglion. The post- synaptic sympathetic neurons are distributed to the head by following blood vessels within the associated periarterial plexi. Remember that the sympathetic chain ended at the first cervical level, so sympathetic fibers must follow blood ves- sels to reach their endpoint. In this patient, ptosis (drooping of the upper eyelid) and miosis (pupillary constriction) were noted, and these signs indicate the interruption of the sym- pathetic innervation to the head. The thoracic anterior roots are the only component of the sympathetic nervous system listed in these choices, and these anterior roots of T1-3 were interrupted during the resection of the lymphangioma in this patient. Knowledge of the sympathetic distribution to the head is vital to understanding the innervation of the head. Choice A (Cervical posterior roots) is incorrect. The patient is exhibiting a loss of sympathetic innervation to the head, due to the ptosis and miosis noted in the patient. The sympa- thetic nervous system is a thoracolumbar system, consisting of presynaptic cell bodies that reside in the intermediolateral cell column (IML), which exists from the vertebral levels of T1 through L2 (or L3). The cervical posterior (dorsal) roots are an incorrect answer because posterior roots only contain afferent (or sensory) information, and the autonomic nervous system is an entirely efferent (or motor) system. Moreover, presynaptic sympathetic fibers would ONLY reside in the anterior (ventral) roots of T1-L2 (or L3). Because the IML is somatotopically organized, the sympathetic fibers going to the head would exist in the IML from T1-3, and they would ONLY exist in the anterior (ventral) roots of these specific spi- nal nerves. Choice B (Thoracic posterior roots) is incorrect. The posterior (dorsal) roots contain only afferent (sensory) information. The autonomic nervous system is an entirely efferent (motor) system, which eliminates this option. Choice D (Thoracic posterior primary rami) is incorrect. Postsynaptic sympathetic fibers would exist in thoracic posterior (dorsal) primary rami in order to vasoconstrict or vasodilate the blood vessels going to the area supplied by these nerves, specifically the epaxial back muscles and the skin overlying these mus- cles. However, the patient is exhibiting a loss of sympathetic innervation to the head. Sympathetic fibers in the thoracic posterior (dorsal) primary rami have already synapsed in the sympathetic chain and would not be distributed to the head. Choice E (Thoracic gray rami communicantes) is incorrect. Sympathetic fibers in a thoracic gray rami communicantes have already synapsed in the sympathetic chain and would be distributed to the vertebral level at which they synapsed. Therefore, these sympathetic fibers in the thoracic region would not reach the head and would not be involved in the clinical signs/symptoms of this patient.

To differentiate between idiopathic unilateral paralysis of the muscles of facial expression (Bell palsy) and a herpes zoster infection of CN VII, the physician must look for small herpetic lesions (vesicles or blisters). Where are these skin lesions located in a herpes zoster infection involving the facial nerve? (A) Mental region of the mandible (B) Temporal and parietal region of the scalp (C) Upper lip and cheek of the face (D) Auricle of the external ear (E) Bridge and tip of the external nose

The answer is D: Auricle of the external ear. Unilateral facial paralysis can result from brain tumor, stroke, or a virus, such as herpes zoster. When no specific cause can be found (idio- pathic), the condition is called facial (Bell or CN VII) palsy. When differentiating between unilateral facial paralysis and herpes zoster infection of CN VII, the physician must immediately look for herpetic lesions in the skin supplied by the cutaneous innervation of the facial nerve. CN VII supplies cutaneous innervation to only a small portion of the auricle of the external ear, via the posterior division of the main trunk of the facial nerve. The geniculate ganglion of the facial nerve would be affected by the viral infection known as herpes zoster, or varicella zoster virus. This virus causes chickenpox, and after exposure to chickenpox, this virus can reside latent in ganglia of an individual for years. If this individual becomes immunocompromised, the skin (or dermatomes supplied by the infected ganglia) can develop shingles, a painful skin rash, which blisters, breaks open, crusts over, and then disappears. For CN VII, these herpetic lesions are found in the auricle of the external ear, which is the only cutaneous distribution site for the facial nerve. In Ramsay Hunt syndrome, a patient presents with Zoster eruptions in the auricle or external acoustic meatus of the ear and unilateral facial paralysis. This syndrome is often associated with other CN VII and CN VIII symptoms, including loss of taste to the anterior two thirds of the tongue (CN VII), tinnitus (CN VIII), vertigo (CN VIII), or hearing loss (CN VIII). Choice A (Mental region of the mandible) is incorrect. The mental region of the mandible is supplied by the mental nerve, a cutaneous nerve off the mandibular division of the trigeminal nerve (CN V ). If the herpes herpetic lesions could be found along the sensory distribution pattern of CN V3; however, in this patient, the facial nerve (CN VII) was affected due to unilateral paralysis of the facial muscles. Choice B (Temporal and parietal region of the scalp) is incorrect. The temporal and parietal regions of the scalp are supplied by the auriculotemporal nerve, a cutaneous nerve off the mandibular division of the trigeminal nerve (CN V3). If the herpes zoster virus infected the trigeminal (or semilunar) ganglion, herpetic lesions could be found along the sensory distribution pattern of CN V3; however, in this patient, the facial nerve (CN VII) was affected due to unilateral paralysis of the facial muscles. Choice C (Upper lip and cheek of the face) is incorrect. The upper lip and cheek of the face are sup- plied by the infraorbital nerve, a cutaneous nerve off the max- illary division of the trigeminal nerve (CN V2). If the herpes zoster virus infected the trigeminal (or semilunar) ganglion, herpetic lesions could be found along the sensory distribution pattern of CN V2; however, in this patient, the facial nerve (CN VII) was affected due to unilateral paralysis of the facial muscles. Choice E (Bridge and tip of the external nose) is incorrect. The bridge and tip of the external nose are supplied by the infratrochlear and external nasal nerves, respectively. These two nerves are cutaneous branches off the ophthalmic division of the trigeminal nerve (CN V1). If the herpes zoster virus infected the trigeminal (or semilunar) ganglion, herpetic lesions could be found along the sensory distribution pattern of CN V1; however, in this patient, the facial nerve (CN VII) was affected due to unilateral paralysis of the facial muscles.

A 37-year-old man presented with decreased emotional tear- ing on the right side and intermittent headaches. Magnetic resonance images (MRIs) revealed a facial nerve schwannoma located within the right pterygoid (vidian) canal. What nerve fibers are most likely injured by this tumor? (A) Taste fibers to the anterior two thirds of the tongue (B) Parasympathetic innervation to the submandibular and sublingual glands (C) Presynaptic sympathetic fibers (D) Presynaptic parasympathetic fibers (E) Postsynaptic parasympathetic fibers

The answer is D: Presynaptic parasympathetic fibers. The nerve of the pterygoid (vidian) canal consists of presynaptic parasympathetic fibers from the greater petrosal nerve of the facial nerve (CN VII) and postsynaptic sympathetic fibers from the deep petrosal nerve, derived from the periarterial arterial plexus enveloping the internal carotid artery. The presynaptic parasympathetic fibers follow the greater pet- rosal nerve (of CN VII), traverse the pterygoid canal, enter the pterygopalatine fossa to synapse in the pterygopalatine ganglion, and carry the postsynaptic parasympathetic fibers to the lacrimal gland via branches of the first two divisions of the trigeminal nerve (CN V1 and CN V2). The parasympathetic fibers of CN VII produce emotional tears from the lacrimal glands. Choice A (Taste fibers to the anterior two thirds of the tongue) is incorrect. The chorda tympani, a branch of the facial nerve (CN VII), joins the lingual branch of the man- dibular nerve (CN V3) in the infratemporal fossa. The chorda tympani nerve conveys taste sensation from the anterior two thirds of the tongue and carries presynaptic parasympathetic fibers to the submandibular ganglion for innervation of the submandibular and sublingual salivary glands. However, the chorda tympani nerve is not associated with the pterygoid canal, so it would not be damaged by a facial nerve schwan- noma at this location. Choice B (Parasympathetic innervation to the submandibular and sublingual glands) is incorrect. The chorda tympani, a branch of the facial nerve (CN VII), joins the lingual branch of the mandibular nerve (CN V3) in the infratemporal fossa. The chorda tympani nerve conveys taste sensation to the anterior two thirds of the tongue and carries presynaptic parasympathetic fibers to the submandibular gan- glion for innervation of the submandibular and sublingual salivary glands. However, the chorda tympani nerve is not associated with the pterygoid canal, so it would not be dam- aged by a facial nerve schwannoma at this location. Choice C (Presynaptic sympathetic fibers) is incorrect. All sympathetic fibers in the head are postsynaptic because they synapse in the superior cervical ganglion before reaching their targets in the head. Therefore, the sympathetic fibers traveling in the pterygoid canal are postsynaptic. These postsynaptic sym- pathetic fibers control the smooth muscle tone (vasodilation and vasoconstriction) of the blood vessels lying underneath the nasal mucosa. Choice E (Postsynaptic parasympathetic fibers) is incorrect. The parasympathetic fibers derived from the greater petrosal nerve (of CN VII) do not synapse until after they have left the pterygoid (vidian) canal. After leaving this canal, these presynaptic parasympathetic fibers synapse in the pterygopalatine ganglion distal to the pterygoid canal. Therefore, the parasympathetic fibers in the pterygoid (vidian) canal are presynaptic. These fibers serve to increase emotional tearing of the lacrimal glands.

A computed tomography (CT) scan shows a left crescent-shaped extraaxial hematoma, compressing the brain of a 17-year-old woman, who impacted the front of her head on the steering wheel during a head-on motor vehicular accident. Given the radiologic imaging results and the history of the accident, what blood vessel(s) is/are the most likely source for this cerebral hemorrhage? (A) Middle meningeal artery (B) Superficial temporal artery (C) Cerebral arterial circle (D) Superior cerebral veins (E) Occipital artery

The answer is D: Superior cerebral veins. Following trau- matic impact to the front of the head, the superior cerebral veins may be torn as they drain into the superior sagittal sinus. Tearing of the superior cerebral veins leads to a subdural (or dural border) hemorrhage, which was pictured on the pro- vided CT. Eight to ten superior cerebral veins drain the supe- rior aspect of the cortical hemisphere, and these vessels can be damaged following impact to the front of the skull due to the resulting coup and countercoup movements of the brain within the skull following impact. This violent force on these bridging veins shears the superior cerebral veins entering the superior sagittal sinus, resulting in the pictured cerebral hematoma. The CT scan shows the characteristic crescent- shaped subdural hematoma, with its concave surface away from the cranium. Choice A (Middle meningeal artery) is incorrect. The middle meningeal artery is often torn follow- ing impact to the side of the head, fracturing the skull in the area of the pterion. The middle meningeal artery, specifically its anterior division, runs in close proximity to the pterion, so a fracture of this weakened area of the lateral skull may lacer- ate the middle meningeal artery, which leads to an epidural hemorrhage as the blood pools between the endosteal layer of the dura mater and the calvaria. In this patient, the cerebral hematoma resulted due to traumatic impact to the front of the skull, and the CT verified a hematoma in the area of the supe- rior cerebral veins, within the subdural space (between the dura and arachnoid layers). Choice B (Superficial temporal artery) is incorrect. The superficial temporal artery, the ter- minal branch of the external carotid artery, lies outside of the skull. If this artery were lacerated, bleeding of the scalp would be profuse. Due to the extracranial location of the superficial temporal artery, damage to this artery would not cause intrac- ranial hematoma noted on the CT. Choice C (Cerebral arterial circle) is incorrect. The cerebral arterial circle (of Willis) is an anastomoses of arteries located on the ventral surface of the brain in the area of the interpeduncular fossa, optic chiasm, and hypothalamus. These vessels are prone to saccular (berry) aneurysm, particularly where the arteries join together. Upon rupturing, blood spills into the subarachnoid space because all major vessels of the brain travel within the subarach- noid space. Subarachnoid hemorrhages are often seen in the absence of trauma, and the three cardinal signs and symptoms of a subarachnoid hemorrhage are (1) loss of consciousness (lethargy), (2) nuchal rigidity (stiff neck), and (3) a sudden onset of the "worst headache of your life," which were not present in this patient. This patient experienced a traumatic blow to the front of her head, and the hematoma, illustrated on the CT, confirmed a subdural hematoma, implicating the superior cerebral veins as the source of the bleed. Choice E (Occipital artery) is incorrect. The occipital artery, a branch of the external carotid artery, lies outside of the skull. If this artery were lacerated, bleeding of the scalp would be profuse; however, damage to this artery would not cause intracranial hematoma noted on the CT.

A patient damages the cranial nerve indicated by the arrow in the figure. What signs or symptoms would be evident in a patient with this nerve lesion? (A) Inability to elevate the ipsilateral shoulder (B) Deviation of the uvula to the contralateral side (C) Loss of gag reflex on the ipsilateral side (D) Ipsilateral loss of parotid gland secretion (E) Deviation of the tongue to the ipsilateral side

The answer is E: Deviation of the tongue to the ipsilateral side. One of the rootlets of the hypoglossal nerve (CN XII) is identified in this photo. CN XII emerges from the brainstem in the medulla between the pyramid medially and the olive (inferior olivary nucleus) laterally. The hypoglossal nerve innervates all of the intrinsic muscles of the tongue and most of its extrinsic muscles with the lone exception being the palatoglossus muscle are innervated by the vagus nerve (CN X). Damage to CN XII produces ipsilateral deviation of the tongue due to the unopposed muscular contractions of the genioglossus muscle contralaterally. The mnemonic "The tongue licks the wound" will help you remember that the tongue deviates to the ipsilateral side in a lower motor lesion of CN XII. Choice A (Inability to elevate the ipsilateral shoulder) is incorrect. The accessory nerve (CN XI) emerges from the cervical aspect of the spinal cord, ascends through the magnum foramen, and leaves the cranial cavity through the jugular foramen, accompanying the glossopharyngeal (CN IX) and vagus (CN X) nerves. It pro- vides motor innervation to the ipsilateral sternocleidomastoid and trapezius muscles. Damage to CN XI causes an inability to elevate the shoulder leading to "drooping" of the ipsilateral shoulder due to paralysis of the trapezius muscle. A CN XI lesion also affects lateral rotation of the scapula causing an inability to abduct the upper limb greater than 90 degrees. Loss of innervation to the sternocleidomastoid muscle causes paresis (weakness) in turning the head to the contralateral side. However, the hypoglossal nerve (CN XII) was identified in the photo. Choice B (Deviation of the uvula to the contral- ateral side) is incorrect. The vagus nerve (CN X) emerges from the brainstem in the medulla posterior to the olive. Damage to CN X causes asymmetry in soft palate elevation and con- tralateral deviation of the uvula. The pharyngeal branches of the vagus nerve innervate all of the musculature of the soft palate, except the tensor veli palatini (tensor of the soft pal- ate), which is innervated by the mandibular division of the trigeminal nerve (CN V3). On examination, the arch of the soft palate droops on the affected side and the uvula deviates to the unaffected side as a result of the unopposed actions of the intact muscles acting on the soft palate on the unaffected (contralateral) side. However, the hypoglossal nerve (CN XII) was identified in the photo. Choice C (Loss of gag reflex on the ipsilateral side) is incorrect. The gag (or pharyngeal) reflex prevents foreign objects from entering the pharynx, except during normal swallowing (deglutition). The afferent limb of the gag reflex is supplied by the glossopharyngeal nerve (CN IX), and the efferent limb is supplied by the vagus nerve (CN X). Both the glossopharyngeal and vagus nerves emerge from the brainstem in the medulla posterior to the olive. Due to the hypoglossal being identified in this figure, the gag reflex would not be affected on the ipsilateral side. Choice D (Ipsilateral loss of parotid gland secretion) is incorrect. The parotid gland is supplied by the parasympathetic fibers from the glossopha- ryngeal nerve (CN IX) that enable secretions of this salivary gland. The glossopharyngeal nerve (CN IX) emerges from the brainstem in the medulla posterior to the olive, so this cra- nial nerve is not identified in this photo and ipsilateral loss of parotid gland secretions would not be seen in this patient.

A 27-year-old man comes to his doctor complaining that the floor of his mouth is painful and swollen. Bimanual palpation and later computed tomography show an 8-mm stone (sialolith) in the right submandibular salivary duct. If the submandibular duct is partially obstructed, where would the physician observe salivation in the oral cavity after applying pressure to the right submandibular gland? (A) Second maxillary molar tooth (B) Oral vestibule (C) Palatine mucosa (D) Sublingual fold (E) Lingual frenulum

The answer is E: Lingual frenulum. The openings of the submandibular ducts reside on either side of the lingual frenulum, and the saliva produced by the submandibular glands drains through the sublingual caruncles (or papillae), openings at the base of the lingual frenulum. By applying pressure to the right submandibular gland, the physician would look at the lingual frenulum for the salivary secretions of the sub- mandibular glands. If no salivation is observed, then the right submandibular salivary duct is completely obstructed. Choice A (Second maxillary molar tooth) is incorrect. The duct of the parotid gland drains saliva into the oral vestibule via a small opening opposite the second maxillary molar teeth on either side of the oral cavity. In this case, the physician needs to look for the drainage of the submandibular duct, which is at the sublingual caruncle located within the base of the lingual frenulum. Choice B (Oral vestibule) is incorrect. The duct of the parotid gland drains saliva into the oral vestibule via a small opening opposite the second maxillary molar teeth on either side of the oral cavity. In this case, the physician needs to look for the drainage of the submandibular duct, which is at the sublingual caruncle located within the base of the lingual frenulum. Choice C (Palatine mucosa) is incorrect. The ducts of the palatine glands reside deep to the mucosa of the hard palate, and they drain their mucous secretions into the supe- rior aspect of the oral cavity. In this case, the physician needs to look for the drainage of the submandibular duct, which is at the sublingual caruncle located within the base of the lin- gual frenulum. Choice D (Sublingual folds) is incorrect. The sublingual folds lie in the floor of the mouth and represent the location of the drainage of the sublingual glands. The small sublingual ducts conduct saliva into the oral cavity through numerous openings within the sublingual folds.

During extraction of her impacted wisdom teeth, a 22-year-old woman suffers damage to her right inferior alveolar nerve. Which of the following conditions is most likely to result? (A) Inability to compress the cheek (B) Paresthesia of the lower lip (C) Weakness in closing the jaw (D) Decreased salivary flow (E) Reduced taste in the body of the tongue

The answer is B: Paresthesia of the lower lip. The inferior alveolar nerve, a branch of the mandibular division of the trigeminal nerve (CN V3), leaves the ramus of the mandible after traveling through the extent of that bone, conveying afferent fibers from the mandibular teeth. Its mental nerve branch enters the mandible through the mental foramen after supplying sensory (cutaneous) innervation to the skin overly- ing the mandible, including the lower lip. Due to its location within the mandible and close relation to the dental arcade, the inferior alveolar nerve may be damaged during dental extrac- tions, especially more complicated extractions of impacted third molars ("wisdom teeth"). Thus, paresthesia in the lower lip may result from such procedures. Choice A (Inability to compress the cheek) is incorrect. Compression of the cheek is an important action in all aspects of feeding and in other oral functions as well, and it is produced by the contraction of the buccinator muscle, one of the facial muscles. Being a muscle of facial expression, the buccinator is controlled by the facial nerve (CN VII), through its terminal facial branches. Choice C (Weakness in closing the jaw) is incorrect. Chew- ing is the result of the complex interactions of the muscles of mastication, which are controlled by the motor branches of the mandibular division of the trigeminal nerve (CN V3). Of these four mastication muscles, the masseter, temporalis, and medial pterygoid muscles act in closing the jaw. The lateral pterygoid and anterior belly of the digastric muscles act in the opening of the oral fissure by depressing the mandible. The inferior alveolar nerve gives rise to the mylohyoid nerve before it enters the mandible via the mandibular foramen. The mylohyoid nerve supplies the anterior belly of the digas- tric and mylohyoid muscles. Therefore, while damage to the proximal part of the inferior alveolar nerve may influence jaw opening by affecting the anterior digastric muscle, damage to its distal (intramandibular) part does not affect any muscles of mastication, sparing both opening and closing of the mandible. Choice D (Decreased salivary flow) is incorrect. Secretomotor control of the submandibular and sublingual salivary glands is governed by parasympathetic neurons that originate in the facial nerve through its chorda tympani branch and ultimately reach the oral floor via the lingual nerve, a branch of the mandibular division of the trigeminal nerve (CN V3). Control of the parotid salivary gland is conveyed through parasym- pathetic fibers that originate in the glossopharyngeal nerve, travel through its tympanic and lesser petrosal branches, and ultimately reach the parotid through the auriculotemporal nerve (another branch of CN V3). Choice E (Reduced taste in the body of the tongue) is incorrect. Taste fibers in the body (anterior two thirds) of the tongue are provided by the facial nerve via its chorda tympani branch. General sensory fibers to the same area are supplied by the lingual nerve, a branch of the mandibular division of the trigeminal nerve (CN V3). The chorda tympani nerve unites with the lingual nerve within the infratemporal fossa. Thus, whereas the taste and general sensory components to the body of the tongue arise from separate cranial nerve pathways, they share a final common pathway to their target region. Remember that the parasympa- thetic supply to the oral floor (submandibular and sublingual salivary glands; oral floor mucus glands) also shares most of the chorda tympani/lingual nerve pathway. However, the auto- nomic fibers leave the lingual nerve in the oral floor to synapse in the submandibular ganglion.

A 40-year-old woman suffers from headaches, nausea, vomit- ing, and multiple lower cranial nerve involvement. Her physician orders a CT soft tissue neck study, and the given coronal CT shows a mass lesion (tumor) centered at the jugular fora- men and identified by arrows. This tumor has destroyed the jugular foramen and hypoglossal canal on the right side and damaged the cranial nerves traversing these foramina. In this patient, which of the following functions will remain intact? (A) Control of the true vocal fold (B) Taste in the anterior two thirds of the tongue (C) Symmetric protrusion of the tongue (D) Sensation in the tympanic cavity (E) Elevation of the shoulder

The answer is B: Taste in the anterior two thirds of tongue. The chorda tympani nerve, a terminal branch of the facial nerve (CN VII), conveys taste sensation to the anterior two thirds of the tongue and carries presynaptic (preganglionic) parasympathetic fibers to the submandibular and sublingual glands to enable salivation. CN VII enters the skull through the internal acoustic meatus, and in this coronal CT, the margins of this foramen remain intact and are not involved in this pre- sentation. Therefore, the tumor in the posterior cranial fossa is not currently affecting CN VII and its chorda tympani branch, which exits the skull via the petrotympanic fissure. This image shows a massive lesion within the right petrous temporal bone at the right jugular foramen with gross bone destruction. Involvement of the hypoglossal canal was also noted, though not shown on this particular CT. Therefore, the functions of the glossopharyngeal (CN IX), vagus (CN X), accessory nerve (CN XI), and hypoglossal (CN XII) nerves would be lost. Taste supplied to the anterior two thirds of the tongue by the chorda tympani nerve of CN VII would be the only function remaining intact. Choice A (Control of the true vocal fold) is incorrect. All of the intrinsic laryngeal muscles are controlled by branches of the vagus nerve (CN X). Specifically, all of these muscles are supplied by the recurrent laryngeal nerve of CN X, except for the cricothyroid muscle, which is innervated by the exter- nal laryngeal nerve off the superior laryngeal nerve of CN X. The pictured lesion has caused gross bone destruction within the right petrous temporal bone at the right jugular foramen. Therefore, the main stem of the vagus nerve is compromised at the right jugular foramen, causing loss of control and sen- sation in the entire right side of the larynx. Remember that the glossopharyngeal (CN IX), vagus (CN X), and (spinal) accessory (CN XI) nerves emerge from the cranium through the jugular foremen, and the given CT showed erosion of this bony canal by the invading tumor. Choice C (Symmetric pro- trusion of the tongue) is incorrect. The hypoglossal nerve (CN XII) innervates all of the intrinsic muscles of the tongue and most of its extrinsic muscles with the lone exception being the palatoglossus muscle, innervated by the vagus nerve (CN X). Therefore, damage to the right CN XII would produce ipsilat- eral deviation of the tongue during protrusion due to the unop- posed muscular contractions of the contralateral genioglossus muscle. The mnemonic "The tongue licks the wound" will help you remember that the tongue deviates to the ipsilateral side in a lower motor neuron lesion of CN XII. A patient with a CN XII deficit would also display dysarthria and fasciculations within the tongue musculature, and the right CN XII would have been damaged in this patient due to the invading tumor destroying the hypoglossal canal, as noted in the clinical case. Choice D (Sensation in the tympanic cavity) is incorrect. Sensation from the walls of the tympanic (middle ear) cavity is carried by the branches of the tympanic nerve, a branch of the glossopharyn- geal nerve (CN IX). This lesion has caused gross bone destruc- tion at the right jugular foramen, resulting in damage to CN IX. The invading nature of this tumor would cause a sensory defi- cit in the right tympanic cavity. Remember that the glossopha- ryngeal (CN IX), vagus (CN X), and accessory (CN XI) nerves emerge from the cranium through the jugular foramen, and this tumor would lead to functional loss of these three cranial nerves. Choice E (Elevation of the shoulder) is incorrect. The (spinal) accessory nerve (CN XI) innervates the trapezius and sternocleidomastoid muscles. The trapezius elevates, retracts, depresses, and rotates the scapula. Thus, damage to CN XI at the jugular foramen results in notable deficits in shoulder action, including elevation. Remember that the glossopharyn- geal (CN IX), vagus (CN X), and accessory (CN XI) nerves emerge from the cranium through the jugular foramen, and the given CT showed erosion of this canal by the invading tumor.

A 68-year-old man was choking on a piece of steak at a family restaurant. Despite attempts to dislodge the food via abdominal thrusts (or the Heimlich maneuver), his upper airway remained blocked. An emergency medical technician (EMT), eating at the scene, performed an emergency tracheotomy to enable the man to breathe. Which subcutaneous structure was most likely cut during this procedure? (A) Cricoid cartilage (B) Thyrohyoid membrane (C) Cricothyroid membrane (D) Tracheal rings (E) Isthmus of thyroid gland

The answer is C: Cricothyroid membrane. In an emergency tracheotomy, the cricothyroid membrane is incised in order to establish a direct airway for the patient. This procedure is also called a cricothyrotomy or cricothyroidotomy, and it is used as a last resort to circumvent upper airway obstructions. The cri- cothyroid membrane is an important component of the conus elasticus, which is composed of the vocal ligaments, median cricothyroid membrane, and lateral cricothyroid membranes. The cricothyroid membrane is the perfect location to perform an emergency tracheotomy because of several nearby palpa- ble landmarks, and it is located below the (true) vocal folds, which serve as the main inspiratory sphincter of the larynx. Do not confuse the emergency tracheotomy with a tracheos- tomy, which is a procedure performed in a hospital setting and involves surgically creating a hole in the cartilaginous rings of the trachea. Choice A (Cricoid cartilage) is incorrect. Due to the thickness of the cricoid cartilage, incising through this car- tilage would be difficult outside the hospital setting. Moreover, an incised cricoid cartilage would need surgical intervention to heal due to its lack of blood supply. Moreover, damaging the cricoid cartilage would be detrimental to the integrity of the larynx and the laryngeal skeleton. Choice B (Thyrohyoid membrane) is incorrect. Though it is easily palpated due to its position above the laryngeal prominence (or the "Adam's apple"), the thyrohyoid membrane is located between verte- bral levels C3 and C4 and may not establish a direct airway. To combat against an upper airway obstruction, the cricothy- roid membrane (between vertebral levels C5 and C6) would be a better option. Damage to the thyrohyoid membrane could also compromise the superior laryngeal artery and the inter- nal branch of the superior laryngeal nerve, which pierce this membrane to enter the larynx. Choice D (Tracheal rings) is incorrect. In a tracheostomy, a hole is created surgically in the cartilaginous rings of the trachea. However, this procedure is usually performed in a hospital setting under sterile conditions. Damage to the thyroid gland and infrahyoid muscles can easily occur if a tracheostomy is not performed correctly. Incising the cricothyroid membrane would be a much easier means of estab- lishing an airway, especially considering the emergency condi- tions surrounding this patient's choking incident. Choice E (Isthmus of the thyroid gland) is incorrect. Cutting through the isthmus of the thyroid gland would not establish an airway for this patient, so this option can be easily eliminated. Due to its location at the seventh cervical vertebra, this glandular tissue is often transected or resected during a tracheostomy, when the tracheal rings are incised. However, a tracheostomy is performed in a hospital setting under sterile conditions.

During preparation to extract the right maxillary (upper) canine tooth, a dentist has difficulty anesthetizing this tooth and its associated gingivae. Therefore, the dentist administers a regional nerve block, in which the anesthetic syringe needle penetrates the oral mucosa at the apex of the maxillary vestibule and is pushed beyond the roots of the teeth. Due to this injection, the patient experiences numbness and paresthesia within the upper canine and neighboring teeth, as well as the skin of the right inferior eyelid, cheek, lateral nose, and upper lip. What nerve was blocked and produced the described numbness? (A) Anterior superior alveolar (B) External nasal (C) Infratrochlear (D) Infraorbital (E) Nasopalatine

The answer is D: Infraorbital. The infraorbital nerve is a terminal branch of the maxillary (second) division of the trigeminal nerve (CN V2). This nerve passes through the supe- rior aspect (roof) of the maxillary sinus and emerges from the infraorbital foramen. It supplies the maxillary incisor, canine, and premolar teeth via its anterior and middle superior alveo- lar branches in the roof of the maxillary sinus. Ultimately, it supplies the inferior eyelid, cheek, lateral nose, and upper lip via its terminal branches in the face. In the described regional nerve block, the tip of the syringe needle is placed at the infraorbital foramen and the area is flooded with anesthetic, resulting in an infraorbital nerve block. The anesthetic agent also percolates through the thin maxillary bone to block the anterior and middle superior alveolar nerves. The infraorbital nerve is the only nerve listed, which could cause the extent of numbness and paresthesia described. Choice A (Anterior superior alveolar) is incorrect. The anterior superior alveolar nerve branches off the infraorbital nerve in the superior aspect (roof) of the maxillary sinus. It descends in a canal in the ante- rior wall of the maxillary sinus to supply the maxillary incisor and canine teeth. Blocking this nerve would numb most of the targeted dental area in this patient but would not account for the affected cutaneous region. In fact, the infraorbital nerve block is often administered following failure to block only the anterior superior alveolar nerve. Choice B (External nasal) is incorrect. The external nasal nerve is a terminal branch of the anterior ethmoidal nerve from the ophthalmic (first) division of the trigeminal nerve (CN V1). It emerges between the bone and cartilage of the external nose to supply the tip of the nose. The external nasal nerve would not be responsible for the par- esthesia and numbness described in this patient. Choice C (Infratrochlear) is incorrect. The infratrochlear nerve arises from the nasociliary nerve of the ophthalmic (first) division of the trigeminal nerve (CN V1). It runs along the superior border of the medial rectus muscle, an extraocular muscle within the orbit, to supply cutaneous (sensory) innervation to the skin of the medial eyelids and bridge of the nose. It also supplies the conjunctiva, lacrimal sac, and caruncle of the eye. The infratrochlear nerve would not be responsible for the paresthesia and numbness described in this patient. Choice E (Nasopalatine) is incorrect. The nasopalatine nerve arises off the posterior superior nasal branches of the maxillary (sec- ond) division of the trigeminal nerve (CN V2). It runs antero- inferior between the mucous membrane and periosteum of the nasal septum to enter the incisive canal in the roof of the mouth. It supplies the hard palate posterior to the maxillary incisor and canine teeth. The nasopalatine nerve would not be responsible for the paresthesia and numbness described in this patient.

A male 1st-year medical student mistakenly enters the women's locker room and finds a group of his female colleagues changing their clothes. He is shocked and embarrassed by his mistake and immediately runs away with his heart pounding. Given his agitated state, what ganglion, housing neuron cell bodies, is experiencing an extremely high rate of activity? (A) Ciliary ganglion (B) Trigeminal ganglion (C) Inferior (nodose) vagal ganglion (D) Superior cervical ganglion (E) Pterygopalatine ganglion

The answer is D: Superior cervical ganglion. The student's reaction is a classic sympathetic "fight, fright, flight" response. All sympathetic fibers destined for the head synapse in the superior cervical ganglion. This ganglion marks the cranial end of the sympathetic chain, and is located near the base of the skull, at about vertebral level C2. Presynaptic neurons ascend through the sympathetic chain to the superior cervical ganglion. There, they synapse with postsynaptic neuron cell bodies that distribute their axonal processes throughout the head. These sympathetic fibers head control vasomotion, pilo- motion, and sudomotion. Choice A (Ciliary ganglion) is incor- rect. The ciliary ganglion is a parasympathetic ganglion, located in the orbit. The fibers that synapse in this ganglion supply the ciliary and sphincter pupillae muscles in the eye. It has no input into this sympathetic response. Choice B (Trigeminal ganglion) is incorrect. The trigeminal (semilunar) ganglion is a sensory ganglion. It houses the cell bodies of the pseudounipolar neu- rons carrying general sensory information from the trigeminal nerve (CN V). It has no input into this sympathetic response. Choice C (Inferior [nodose] vagal ganglion) is incorrect. The vagus nerve possesses two ganglia, superior and inferior, which are sensory ganglia. The superior vagal ganglion houses cell bodies of general sensory neurons. The inferior vagal ganglion houses the cell bodies of visceral afferent neurons. It has no input into this sympathetic response. Choice E (Pterygopala- tine ganglion) is incorrect. The pterygopalatine ganglion is a parasympathetic ganglion, located within the pterygopalatine fossa. It sends fibers to the lacrimal gland and glands within the nasal cavity. It has no input into this sympathetic response.

If normal evagination of the endodermal lining of the embryonic pharynx does not take place between the pharyngeal (branchial) arches, the pharyngeal (branchial) pouches will not form. Maldevelopment of the first pharyngeal pouch during embryonic weeks 4 to 5 is most likely to result in a congenital disorder of which of the following structures? (A) Thyroid gland (B) Thymus gland (C) Parathyroid glands (D) Facial muscles (E) Tympanic cavity

The answer is E: Tympanic cavity. Evagination of the first pha- ryngeal pouch forms an elongate diverticulum off the pharynx (the tubotympanic recess). The distal part of this recess wid- ens to form the tympanic (or middle ear) cavity. The proximal part remains more tubular and forms the pharyngotympanic (auditory; eustachian) tube. Development of the tubotympanic recess induces the formation of the overlying first pharyngeal cleft (groove), which forms the external acoustic meatus. The closing plate (pharyngeal membrane) between the tubotym- panic recess and the first pharyngeal groove (i.e., the tissue interface between the two) forms the tympanic membrane (eardrum). Choice A (Thyroid gland) is incorrect. The thy- roid gland first forms as a single, ventromedian diverticulum (thyroid diverticulum) off the floor of the embryonic pharynx, between the tuberculum impar and copula of the incipient tongue. It descends caudally along the midline, anterior to the gut tube, until achieving its final location in the neck. The thyroid gland is not derived from the pharyngeal pouches, so it can be easily eliminated as an answer. Choice B (Thy- mus gland) is incorrect. The thymus gland begins as bilateral epithelial primordia in the ventral portions of the third pha- ryngeal (branchial) pouches, which lose their connections with the pharyngeal walls, descend caudally and medially, and ultimately migrate into the upper anterior thorax where they fuse with each other. Descent of the thymic primordia is notably influenced by shifting of the embryonic heart into the differentiating thorax. Choice C (Parathyroid glands) is incorrect. The parathyroid glands develop from the epithelia of both the third and fourth pharyngeal pouches. The inferior parathyroid appears in the dorsal part of the third pouch, near the incipient thymus gland. The parathyroid is drawn caudally with the migration of the thymic tissue, eventually fixing into place near the caudal dorsal aspect of the thyroid gland. The superior parathyroid first forms in the dorsal part of the fourth pharyngeal pouch. This tissue attaches to the migrating thy- roid gland and takes position in the more cranial dorsal aspect of the thyroid. Choice D (Facial muscles) is incorrect. The facial muscles (muscles of facial expression) are small skeletal muscles that form from the mesoderm of the second (hyoid) pharyngeal arch. Other muscles derived from the second arch include the stapedius (in the middle ear), and the stylohyoid and posterior belly of the digastric (in the upper neck). All the second arch muscles are supplied by the facial nerve (CN VII).

A 65-year-old man complains of a persistent nosebleed. His physician uses a cotton swab to apply pressure at the source, the inferior and posterior aspects of the lateral nasal wall. Which artery is the most likely source of the bleeding? (A) Greater palatine (B) Infraorbital (C) Facial (D) Anterior ethmoidal (E) Sphenopalatine

The answer is E: Sphenopalatine. The sphenopalatine artery supplies most of the blood to the nasal cavity, particularly the inferior and posterior aspects of the nasal cavity. Therefore, it is highly probable that the sphenopalatine artery, a terminal branch of the maxillary artery, was the source of the epistaxis (nosebleed) in this patient, as seen in the given illustration located on the next page. Choice A (Greater palatine) is incor- rect. The greater palatine artery supplies blood primarily to the hard palate; however, it may supply a small amount of blood to the anterior and inferior aspects of nasal cavity via its com- munication with the sphenopalatine artery through the inci- sive canal. However, due to the location of the injury in this patient, the greater palatine artery would not be involved with this persistent nosebleed. Choice B (Infraorbital) is incorrect. The infraorbital artery courses in the roof of the maxillary sinus to give blood to this area as well as the superior canine and incisor teeth, inferior aspect of the orbit, and superior aspect of the lip. The infraorbital artery would not be responsible for the nosebleed seen in this patient. Choice C (Facial) is incorrect. The lateral nasal branch of the facial artery would supply the anterior and inferior aspects of the nasal cavity in the region of the vestibule; however, bleeding of this artery would be easily stopped by applying pressure to the alae of the nose. Choice D (Anterior ethmoidal) is incorrect. The anterior ethmoidal artery supplies the anterior, superior aspect of the nasal cavity after entering the nasal cavity through the cribi- form plate of the ethmoid bone. This artery does not supply blood to posterior and inferior aspect of the lateral nasal wall, which is the location of the bleed in this patient.

A 9-year-old girl with a history of middle ear infections presents with pain, tenderness, and inflammation located posterior to her right auricle. What is the most likely diagnosis, given her current symptoms? (A) Otitis externa (B) Blockage of pharyngotympanic tube (C) Mastoiditis (D) Perforated tympanic membrane (E) Ménière syndrome

C. Mastoiditis Mastoiditis, an infection of the air cells within the mastoid process, is the most likely diag- nosis due to the symptoms (pain, tenderness, and inflam- mation) being localized posterior to the right auricle, where the mastoid process of the temporal bone resides. Before the invention of antibiotics, mastoiditis was the leading cause of child mortality; today, however, it is rarely seen in developed countries. In this patient, mastoiditis could be confirmed with imaging studies. In the coronal CT located on the next page, the indicated fluid accumulation can be seen within the right mastoid air cells and mastoid antrum and the extracra- nial inflammation located behind the auricle, characteristic of mastoiditis, can be visualized lateral to the encircled mastoid process. Choice A (Otitis externa) is incorrect. Otitis externa (or swimmer's ear) is a bacterial infection affecting skin lin- ing the external acoustic meatus, which results from polluted water being trapped within this meatus. It presents with pain, inflammation, itchiness, and ear discharge; these symptoms may lead to temporary conductive hearing loss if inflammation and discharge block the external acoustic meatus. When otitis externa is suspected, a physician can push on the tragus of the external ear to see if the intensity of pain increases, which is how swimmer's ear is diagnosed. This patient presented with pain, inflammation, and tenderness; however, these symptoms were localized posterior to her right auricle, not within the external acoustic meatus. Choice B (Blockage of pharyngo- tympanic tube) is incorrect. The pharyngotympanic (auditory; eustachian) tube connects the nasopharynx to the tympanic (middle ear) cavity, which can provide a route for infection into the tympanic cavity. When it is occluded, the tympanic cavity is prone to middle ear infections (otitis media). This patient had a history of otitis media, which presents with pain and inflammation due to a collection of fluid located deep to the tympanic membrane. Because her symptoms (pain, inflammation, and tenderness) were localized posterior to her right auricle, mastoiditis is a more accurate diagno- sis. Choice D (Perforated tympanic membrane) is incorrect. A perforated tympanic membrane is often seen secondary to severe otitis media (middle ear infection). A ruptured tympanic membrane would cause a conductive hearing loss, which was not evident in this patient. Choice E (Ménière syndrome) is incorrect. Ménière syndrome affects the inner ear, and it pres- ents with periodic episodes of vertigo and dizziness, tinnitus, and fluctuating, progressive sensori neural hearing loss. It is caused by blockage of the cochlear aqueduct, which affects the drainage of endolymph within the inner ear. These symptoms were not seen in this patient.

A 67-year-old man presents with shingles on his left forehead, upper eyelid, and bridge of the nose as shown in the figure. Shingles (or herpes zoster) is caused by the varicella zoster virus, which causes chickenpox in children and young adults. After the initial chickenpox outbreak, this virus usually resides latent in sensory ganglia in the body for many years. When a patient is immunocompromised, this virus can cause shingles unilaterally along the infected nerve's dermatome distribution. What sensory ganglion is most likely affected in this patient? (A) Otic ganglion (B) Trigeminal ganglion (C) Superior cervical ganglion (D) Ciliary ganglion (E) Submandibular ganglion

B: Trigeminal Nerve. As seen in the photo, shingles is evident in the dermatome distribution pattern of the ophthalmic division of the left trigeminal nerve (CN V1). There- fore, the trigeminal (or semilunar) ganglion is the most prob- able ganglion infected by the varicella zoster virus. There are two types of cranial ganglia: sensory (afferent) and autonomic. The sensory ganglia, which are most often affected by shin- gles, are equivalent to the spinal (dorsal root) ganglia of spinal nerves in that they house typical pseudounipolar cell bodies of afferent neurons and do not contain synapses. Other cranial sensory ganglia include the geniculate (CN VII), vestibular and spiral (CN VIII), superior and inferior glossopharyngeal (CN IX), and superior and inferior vagal (CN X) ganglia; however, shingles would be present in their dermatome dis- tribution patterns if these ganglia were infected by the vari- cella zoster virus. Choice A (Otic ganglion) is incorrect. The otic ganglion is an autonomic ganglion, which houses the postsynaptic (postganglionic) parasympathetic cell bodies that provide secretomotor fibers to the parotid gland. This ganglion is located deep within the infratemporal fossa, adja- cent to the mandibular division of the trigeminal nerve (CN V3). The otic ganglion receives presynaptic (preganglionic) parasympathetic fibers from the lesser petrosal nerve of the glossopharyngeal nerve (CN IX). Because it is an autonomic ganglion, it is unlikely to be infected by the varicella zoster virus, which causes shingles unilaterally. Choice C (Superior cervical ganglion) is incorrect. The superior cervical ganglion is also an autonomic ganglion; specifically, it is a component of the sympathetic system. This autonomic ganglion is located at the cranial end of the sympathetic chain, near the base of the skull. There are two superior cervical ganglia, located bilater- ally, which contain the cell bodies for all the postsynaptic sym- pathetic fibers that supply the head. Because it is an autonomic ganglion, it is unlikely to be infected by the varicella zoster virus, which causes shingles unilaterally. Choice D (Ciliary ganglion) is incorrect. The ciliary ganglion is another para- sympathetic autonomic ganglion. It is located in the orbit and carries fibers related to the oculomotor nerve (CN III). Because it is an autonomic ganglion, it is unlikely to be infected by the varicella zoster virus, which causes shingles unilaterally. Choice E (Submandibular ganglion) is incorrect. The subman- dibular ganglion is a parasympathetic autonomic ganglion. It is located in the posterolateral oral floor and carries fibers related to the facial nerve (CN VII) to supply secretomotor fibers to the submandibular and sublingual glands. Because it is an autonomic ganglion, it is unlikely to be infected by the varicella zoster virus, which causes shingles unilaterally.

A 4-year-old girl presents with nausea, vomiting, papilledema, and a headache, which is more severe when she wakes up in the morning. A sagittal T1-weighted MRI reveals a medulloblastoma in the posterior cranial fossa. Its location has compressed the 4th ventricle, impeding the flow of cerebrospinal fluid (CSF) from the 3rd ventricle via the cerebral aqueduct, indicated by the black arrow. The MRI shows massive dilation of the lateral and 3rd ventricles, shallow cortex, and effaced sulci within the cerebral hemispheres. Which of the following selections describes this condition? (A) Dandy-Walker syndrome (B) Chiari malformation (C) Noncommunicating hydrocephalus (D) Congenital communicating hydrocephalus (E) Acquired communicating hydrocephalus

C. noncommunicating hydrocephalus Noncom- municating hydrocephalus, or obstructive hydrocephalus, is caused by an obstruction of CSF flow between or within the ventricles of the brain, which prevents CSF from entering the subarachnoid space. In this 4-year-old girl, the medulloblas- toma obstructs the 4th ventricle, preventing CSF arriving into the fourth ventricle via the cerebral aqueduct (of Sylvius) from entering the subarachnoid space via the 4th ventricular outlets (paired lateral foramina of Luschka and median foramen of Magendie). This acquired noncommunicating hydrocephalus (G: water in the head) is causing the associated abnormalities in the cerebral hemispheres (dilation of the third and lateral ven- tricles, shallow cortex, and effaced sulci). The girl's symptoms are mainly secondary due to the increased intracranial pressure (ICP), which is due to the blockage of the 4th ventricle and CSF flow. Papilledema is optic disc swelling caused by increased ICP, and it can be observed with an ophthalmoscope in a fundo- scopic examination. Brain tumors are the second most com- mon malignancy among children less than 20 years of age, and they are often located infratentorially (or beneath the cerebel- lum tentorium, a dural fold, which separates the occipital lobe of the brain from the cerebellum). Choice A (Dandy-Walker syndrome) is incorrect. Dandy-Walker syndrome is a congeni- tal brain malformation characterized by the absence of the cer- ebellar vermis, leading to the enlargement of the 4th ventricle. In this patient, the presence of the medulloblastoma causes an acquired noncommunicating hydrocephalus, as noted on the MRI. Choice B (Chiari malformation) is incorrect. A Chiari malformation (or Arnold-Chiari malformation) is a downward displacement of the cerebellar tonsils and medulla through the magnum foramen, which may cause hydrocephalus. Chiari I malformations are more common in women, and the average age for diagnosis is approximately 27 years of age. Chiari II malformations generally present with open spinal dysraphism at birth. Because the downward displacement of the cerebel- lar tonsils, characteristic of the Chiari malformation, was not noted, a noncommunicating hydrocephalus is the better diag- nosis. Choice D (Congenital communicating hydrocephalus) is incorrect. A communicating hydrocephalus, or nonobstructive hydrocephalus, is caused by an impairment of CSF resorption at the arachnoid granulations, located along the superior sagit- tal sinus. Conditions leading to a communicating hydrocepha- lus include subarachnoid hemorrhage, meningitis, Chiari mal- formation, or congenital absence of the arachnoid granulations. However, the medulloblastoma is blocking flow of CSF between the ventricles of the brain, which is the definition of a noncom- municating (obstructive) hydrocephalus. Choice E (Acquired communicating hydrocephalus) is incorrect. A communicating hydrocephalus, or nonobstructive hydrocephalus, is caused by an impairment of CSF resorption at the arachnoid granulations, located along the superior sagittal sinus. Conditions leading to a communicating hydrocephalus include subarachnoid hemor- rhage, meningitis, Chiari malformation, or congenital absence of the arachnoid granulations. However, the medulloblastoma is blocking flow of CSF in the fourth ventricle, which is the def- inition of a noncommunicating (obstructive) hydrocephalus.

A 43-year-old man presents with loss of control of facial expression across the entire right side. The corner of his mouth droops on the right side, but he can clench his jaw and chew on demand. During examination, his physician also notes loss of hearing on the right side, and the patient has dif- ficulty maintaining balance while standing on one foot. The patient's corneal (blink) reflex is absent in the right eye, but cutaneous sensation is normal on the entire face. The physician orders radiographic imaging in anticipation of finding a tumor. What is the most likely location of the tumor? (A) Internal acoustic meatus (B) Foramen ovale (C) Foramen rotundum (D) Geniculum of the facial canal (E) Stylomastoid foramen

The answer is A: Internal acoustic meatus. The loss of facial expression and drooping corner of the mouth indi- cate paralysis of the facial muscles and damage to the facial nerve (CN VII). The intact ability to clench the jaw and chew denotes proper functioning of the muscles of mastication and an intact mandibular division of the trigeminal nerve (CN V3). The hearing loss and unsteady balance indicate failure in the inner ear complex implicating the right ves- tibulocochlear nerve (CN VIII; auditory nerve). The absence of the blink reflex is related to loss of the orbicularis oculi muscle, the facial muscle responsible for closing the eye- lids, and this evidence reinforces a problem with the facial nerve. Normal cutaneous sensation across the face indicates the entire trigeminal pathway is intact. Therefore, the sus- pected tumor affects both CN VII and CN VIII, but not the trigeminal nerve (CN V). The only location where CN VII and CN VIII can be affected simultaneously is at the internal acoustic meatus (in the wall of the posterior cranial fossa), where the paired nerves leave the cranial cavity to enter the petrous part of the temporal bone. Very quickly thereafter, the nerves diverge and follow separate pathways to their target regions. The given contrast-enhanced T1-weighted MRI demonstrates a right-sided vestibular schwannoma (acoustic neuroma), located at the internal acoustic meatus and identified by the arrow, which confirms this diagnosis. This vestibular schwannoma, which is clearly seen in white due to the gadolinium-based intravenous contrast, resides at the cerebellopontine angle and affects the facial and ves- tibulocochlear nerves as they emerge from this location. This tumor would also increase intracranial pressure potentially causing pontomedullary brain stem compression. Choice B (Foramen ovale) is incorrect. The mandibular division of the trigeminal nerve (CN V3) passes through the foramen ovale in the floor of the middle cranial fossa. At this location, a tumor would affect the muscles of mastication and cutane- ous sensation over the mandibular region of the face, which is not evident in this patient. Choice C (Foramen rotundum) is incorrect. The maxillary division of the trigeminal nerve (CN V2) passes through this opening in the anterior wall of the middle cranial fossa. Nerve damage here would affect cutaneous sensation across the midfacial region, which is not evident in this patient. Choice D (Geniculum of the facial canal) is incorrect. The facial nerve travels through the facial canal within the petrous part of the temporal bone, including the knee-like bend (geniculum) of the canal. Thus, tumor growth in this location would affect the facial nerve and pro- duce the facial paralysis described in this case. However, the vestibulocochlear nerve would not be affected at this site, as it has already separated from the facial nerve. Choice E (Stylomastoid foramen) is incorrect. The main branch of the facial nerve exits the skull through the stylomastoid foramen at the base of the skull. Tumor growth here would affect the facial nerve, producing the described facial paralysis. How- ever, the vestibulocochlear nerve would not be affected by problems at this location.

An 18-year-old male skier is brought to the emergency room after a high-speed collision with a tree. His forehead absorbed much of the impact, resulting in multiple fractures and lacerations. His ER physician noted a clear nasal discharge, which tested positive for glucose. Given the patient's presentation, what cranial nerve was most likely damaged? (A) Olfactory nerve (B) Optic nerve (C) Abducent nerve (D) Facial nerve (E) Hypoglossal nerve

The answer is A: Olfactory nerve. The traumatic impact to the forehead caused a defect (or communication) in the floor of the patient's anterior cranial fossa. The cribiform plate of the ethmoid bone, which physically separates the nasal cav- ity from the anterior cranial fossa, was fractured due to the patient's presentation with cerebrospinal fluid (CSF) rhinor- rhea, as the clear nasal discharge that tested positive for glu- cose would imply. Olfactory nerve (CN I) fibers descend into the nasal cavity via the foramina within the cribiform plate of the ethmoid bone, so when this bone is fractured the olfactory nerve is most likely damaged. During a cranial nerve exami- nation, the patient exhibited anosmia (loss of smell) and may even experience alteration of taste perceptions due to the com- plex interactions between the olfaction and gustatory sensory pathways. Severe head trauma often results in anosmia due to the delicate olfactory nerves traversing the foramina within the cribiform plate of the ethmoid bone. Choice B (Optic nerve) is incorrect. Damage to the optic nerve (CN II) causes ipsilat- eral blindness or other visual deficits of the affected eye. This finding was not reported in this patient. Choice C (Abducent nerve) is incorrect. Damage to the abducent nerve (CN VI) causes ipsilateral paralysis to the lateral rectus muscle, which is the only extraocular muscle innervated by CN VI. When the abducent nerve is damaged, the patient displays diplopia, and the ipsilateral eye rests in the adducted position due to the unopposed action of the other extraocular muscles. The abducent nerve is the cranial nerve with the longest intradural course, and it can often be damaged (impinged) as it arches over the crest of the petrous part of the temporal bone or by an aneurysm of the internal carotid artery in the cavernous sinus. This nerve lesion can occur following increased intracranial pressure from a supratentorial (above the cerebellum tento- rium) space-occupying lesion in the brain, which could result from a tumor, hemorrhage, or blockage of CSF. Because this patient presented with CSF rhinorrhea, the olfactory nerve is the cranial nerve most likely damaged. Choice D (Facial nerve) is incorrect. Damage to the facial nerve (CN VII) causes the following sequelae of signs and symptoms: ipsilateral paralysis of the muscles of facial expression, hyperacusis (sen- sitivity to noise), loss of taste to the anterior two thirds of the tongue, loss of secretion from the submandibular and sublin- gual salivary glands, and loss of emotional tearing. However, none of these findings were reported in this patient. Choice E (Hypoglossal nerve) is incorrect. Damage to the hypoglossal nerve (CN XII) causes ipsilateral deviation of the tongue dur- ing protrusion, dysarthria (difficulty speaking), and fascicula- tions (involuntary twitching of muscle fibers). However, none of these findings were reported in this patient.

A 15-year-old young man developed cavernous sinus thrombophlebitis after a 1-week history of a single acne-like lesion at the anterior tip of his nose. He presented with a headache, periorbital edema, diplopia, and a fever (103°F or 39.4°C). What vein is the most likely route for the spread of this infection to the cavernous sinus? (A) Ophthalmic (B) Superior cerebral (C) Great cerebral (of Galen) (D) Maxillary (E) Supraorbital

The answer is A: Ophthalmic. Both the superior and inferior ophthalmic veins provide a pathway for the spread of infection from the anterior tip of the nose to the cavernous sinus as these veins connect the facial vein to the cavernous sinus. Because the facial vein does not contain valves, infection would spread in the direction of least physical resistance. An infection arising on the tip of the nose may spread to the cavernous sinus causing thrombophlebitis. The "danger tri- angle of the face" stretches from the labial commissures bilat- erally to the bridge of the nose. Any lacerations within this triangle, such as the acne pustules frequently seen in maturing adolescents, can cause an infection in the facial vein, which may spread to the cavernous sinus. Remember that veins of the face and the scalp do NOT contain valves, which is clini- cally relevant due to the potential spread of infection. The symptoms seen in this patient, headache, fever, and perior- bital edema were due to the spread of the infection through the ophthalmic veins to the cavernous sinus. Once the infec- tion resided in the cavernous sinus, it could affect the cranial nerves, oculomotor nerve (CN III), trochlear nerve (CN IV), and abducent nerve (CN VI), which control the extraocular muscles, leading to the diplopia, or double vision, seen in this patient. Choice B (Superior cerebral) is incorrect. The supe- rior cerebral vein drains blood from the superolateral aspects of the cerebrum into the superior sagittal sinus. Though these veins are clinically significant because they are often disrupted from a traumatic blow to the front of head resulting in a sub- dural cerebral hemorrhage, they do not communicate with the cavernous sinus and would not be involved with spread of infection from the tip of the nose. Choice C (Great cere- bral [of Galen]) is incorrect. The great cerebral vein (of Galen) merges with the inferior sagittal dural venous sinus to form the straight dural venous sinus. It is a single, midline vein that drains deep structures of the brain. The great cerebral vein does not communicate with the cavernous sinus and would not drain blood from the tip of the nose. Choice D (Maxillary) is incorrect. The maxillary vein is formed by the drainage of the pterygoid plexus of veins, located between the temporalis and lateral pterygoid muscles in the infratemporal region, and drains into the retromandibular vein. The maxillary vein does not directly communicate with the cavernous sinus; however, it can communicate with the ophthalmic veins through the pterygoid plexus via the inferior orbital fissure. However, the maxillary vein is located some distance away from the "danger triangle of the face" and is unlikely to be involved. Choice E (Supraorbital) is incorrect. The supraorbital vein drains the anterosuperior aspects of the scalp, but it is not involved with venous drainage of the tip of the nose, where the infection originated in this patient. Though this vein does drain into the ophthalmic veins, it is not the correct answer due to the loca- tion of the lesion on the tip of the nose.

An MRI of the right internal carotid artery reveals atherosclerotic plaques causing stenosis of the vessel's lumen within the cavernous sinus. The stenosis is causing increased pressure within the internal carotid artery as it courses through the cavernous sinus, resulting in an aneurysm. Given its location, what cranial nerve would most likely be damaged? (A) Trochlear nerve (B) Abducent nerve (C) Maxillary nerve (D) Ophthalmic nerve (E) Oculomotor nerve

The answer is B: Abducent nerve. All of the listed nerves travel within the cavernous sinus; however, the abducent nerve (CN VI) parallels the course of the internal carotid artery within the cavernous sinus, which makes it the most likely nerve to be damaged. The internal carotid artery and abducent nerve are located medially as they traverse the cavernous sinus. In the given figure, the other nerves that travel through the cavern- ous sinus (CNs III, IV, V1, and V2) lie laterally. Therefore, the increased pressure, seen within the internal carotid artery due to the atherosclerotic plaque causing the stenosis and result- ing aneurysm, would most likely affect the abducent nerve. Choice A (Trochlear nerve) is incorrect. The trochlear nerve (CN IV) traverses the cavernous sinus; however, it lies along the lateral wall of the sinus. The abducent nerve, which paral- lels the internal carotid artery, would most likely be affected by increased pressure within this vessel due to the stenosis and subsequent aneurysm. Choice C (Maxillary nerve) is incor- rect. The maxillary nerve (or second division of the trigeminal nerve) traverses the cavernous sinus; however, it lies along the lateral wall of the sinus. The abducent nerve, which parallels the internal carotid artery, would most likely be affected by increased pressure within this vessel due to the stenosis and subsequent aneurysm. Choice D (Ophthalmic nerve) is incor- rect. The ophthalmic nerve (or first division of the trigeminal nerve) traverses the cavernous sinus; however, it lies along the lateral wall of the sinus. The abducent nerve, which paral- lels the internal carotid artery, would most likely be affected by increased pressure within this vessel due to the stenosis and subsequent aneurysm. Choice E (Oculomotor nerve) is incorrect. The oculomotor nerve (CN III) traverses the cavern- ous sinus; however, it lies along the lateral wall of the sinus. The abducent nerve, which parallels the internal carotid artery, would most likely be affected by increased pressure within this vessel due to the stenosis and subsequent aneurysm.

A 7-year-old boy was kicked in the right side of his head during a sledding accident. He arrived at the ER with no loss of consciousness but complained of a severe headache and vomiting. A computed tomography (CT) scan revealed a biconvex hyperdense extraaxial collection of blood. What blood vessel is the most likely source of the bleed? (A) Superficial temporal artery (B) Middle meningeal artery (C) Superior cerebral veins (D) Cerebral arterial circle (of Willis) (E) Middle cerebral artery (MCA)

The answer is B: Middle meningeal artery. The middle men- ingeal artery is often torn following impact to the side of the head, fracturing the skull in the area of the pterion. The pterion, an osteological feature on the side of the head that marks the junction of the parietal, frontal, squamous temporal, and sphe- noid bones, is prone to fracture following traumatic impact to the side of the head, which this boy experienced during his sledding accident. The middle meningeal artery, specifically its anterior division, runs in close proximity to the pterion, so a fracture of this weakened area of the lateral skull may lacer- ate the middle meningeal artery, which frequently leads to an epidural hemorrhage as the blood pools between the endosteal layer of the dura mater and the calvaria. This epidural hem- orrhage was confirmed by the CT scan showing a biconvex hyperdense extraaxial collection of blood pooling beneath the pterion in the epidural space, which is characteristic of an epi- dural hematoma. Also, note the edema at the place of impact located extracranially in this patient's scalp. Remember that a CT scan reveals "blood and bone," and due to it being inex- pensive relative to an MRI, it is the perfect diagnostic tool in trauma. The symptoms displayed in this patient, including a severe headache and vomiting, are signs of increased intrac- ranial pressure (ICP) due to the extravasation of blood from the middle meningeal artery compressing brain tissue within the skull. Choice A (Superficial temporal artery) is incorrect. The superficial temporal artery, the terminal branch of the external carotid artery, lies outside of the skull. If this artery were lacerated, bleeding of the scalp would be profuse. Due to its extracranial location, damage to the superficial tempo- ral artery would not cause the vomiting seen in this patient, which is indicative of increased ICP. Choice C (Superior cere- bral veins) is incorrect. Traumatic impact to the front of the head can tear the superior cerebral veins. The impact shears the superior cerebral veins as they empty into the superior sagit- tal sinus, leading to a subdural hemorrhage (or dural border hematoma). Due to the location of the trauma and the pooling blood revealed on the CT scan, these veins were not damaged. Choice D (Cerebral arterial circle) is incorrect. The cerebral arterial circle (of Willis) is an anastomoses of arteries located on the ventral surface of the brain in the area of the interpe- duncular fossa, optic chiasm, and hypothalamus. These ves- sels are prone to saccular (berry) aneurysm, particularly where the arteries join together. Upon rupturing, blood spills into the subarachnoid space because all major vessels of the brain travel within the subarachnoid space. Subarachnoid hemorrhages are often seen in the absence of trauma, and the three cardinal signs and symptoms of a subarachnoid hemorrhage are (1) loss of consciousness (lethargy), (2) nuchal rigidity (stiff neck), and (3) a sudden onset of the "worst headache of your life", which were not present in this patient. Choice E (Middle cerebral artery [MCA]) is incorrect. The MCA travels deep within the brain in between the frontal and temporal lobes, so it should not be involved in fracture in the area of the pterion. If the MCA was compromised, it would lead to a subarachnoid hem- orrhage, which was not evident on the provided CT scan.

As part of an initial oral examination of a new patient, a dental hygienist inspects the vestibule of the mouth. Which of the following structures is encountered in this area? (A) Lingual frenulum (B) Opening of the parotid duct (C) Opening of the submandibular duct (D) Uvula (E) Palatine tonsil

The answer is B: Opening of the parotid duct. The vestibule of the mouth is the narrow space between the cheeks and lips and the hard wall of teeth and related gingivae (gums). The duct of the parotid gland drains into the vestibule, opposite the upper (maxillary) second molar tooth. The opening of the parotid duct can be visualized in an oral examination and readily located by the tongue. The integrity of the wall of the oral ves- tibule is critical in food handling and articulate speech. Also, the vestibule is important in oral hygiene as food and other materials may collect and fester there if the area is not cleaned properly. Choice A (Lingual frenulum) is incorrect. The lin- gual frenulum is a thin, midline fold of tissue that anchors the underside of the tongue to the floor of the oral cavity proper. Abnormal shortening of the frenulum results in ankyloglossia (tongue-tie). In this condition, the tongue has limited mobil- ity, which may impact food handling and speech, potentially producing a speech impediment. Choice C (Opening of the submandibular duct) is incorrect. The submandibular duct runs forward across the oral floor and drains at the sublingual papilla (caruncle) on the side of the lingual frenulum. Choice D (Uvula) is incorrect. The uvula is the soft midline extension of the soft palate that is most often identified as "that thing that hangs down in the back of the mouth." It serves to assist in sealing off the nasopharynx during swallowing so that food does not regurgitate into the nasal passages. Deviation of the uvula during phonation (saying "Ah") indicates lesion of the vagus nerve (CN X), which innervates the muscle of the uvula (musculus uvulae). Choice E (Palatine tonsil) is incorrect. The palatine tonsil lies on the side of the oropharynx, between the palatoglossal and palatopharyngeal arches. It is highly vascular and may bleed profusely during tonsillectomy.

A 42-year-old woman noticed that her right upper eyelid was drooping and her right pupil was constricted. She goes to her physician where a thorough examination revealed ptosis, miosis, anhydrosis, flushing of her face, and narrowing of the palpebral fissure (the slit between the upper and lower eyelids) on the right side of the patient. Which of the following structure is most likely damaged in this patient? (A) Superior division of the oculomotor nerve (B) Superior cervical ganglion (C) Nerve of the pterygoid canal (D) Ciliary ganglion (E) Ophthalmic division of trigeminal nerve

The answer is B: Superior cervical ganglion. The family of deficits described in this patient comprises a classic generalized sympathetic innervation deficit known as Horner syndrome. Normal sympathetic functions in the head include elevation of the upper eyelid (by the superior tarsal muscle), vasoconstriction of cutaneous arteries, sweat gland secretion, pupillary dilatation (via the dilator pupillae muscle), and forward positioning of the eyeball in the orbit (via the orbital muscle sling). Loss of sympathetic input results in ptosis (drooping of the eyelid), miosis (constricted or "pin-point" pupil), anhydrosis (warm, flushed, dry skin), and enophthalmosis (sunken eye). Such widespread deficits indicate damage at the source point of sympathetic input to the head, and the superior cervical ganglion is the only choice involved in sympathetic innerva- tion. Choice A (Superior division of the oculomotor nerve) is incorrect. This nerve conveys motor fibers to the superior rectus muscle of the eyeball and the levator palpebrae superio- ris in the upper eyelid. Both are skeletal muscles and are not affected by loss of autonomic supply. The levator palpebrae superioris is the major elevator of the upper eyelid. Loss of its motor control does produce ptosis of the upper eyelid, but not the other deficits described in this patient. Choice C (Nerve of the pterygoid canal) is incorrect. The nerve of the pterygoid canal is a combination of sympathetic fibers (from the deep petrosal nerve) and parasympathetic fibers (from the greater petrosal nerve). It runs into the pterygopalatine ganglion, with eventual distribution to the lacrimal gland, nasal cavity, and oral roof. Damage to the nerve of the pterygoid canal would result in loss of lacrimation, dry nasal walls, and dilation of vessels along the course of the maxillary nerve. Choice D (Cili- ary ganglion) is incorrect. The ciliary ganglion is the synapse point for parasympathetic input to the eyeball. These fibers, derived from the oculomotor nerve, control the sphincter pupillae and ciliary muscles. Sympathetic fibers also pass through the ciliary ganglion on their way to the dilator pupil- lae muscle. Damage to this ganglion would disrupt autonomic functions in the eye, but would not produce the extraocular deficits seen in this case. Choice E (Ophthalmic division of trigeminal nerve) is incorrect. The ophthalmic division of the trigeminal nerve (CN V1) conveys general sensory axons from the forehead, upper eyelid, orbit, and eye. Lesion of this nerve would result in a cutaneous deficit in the upper face, and sen- sory loss in the orbit and cornea.

A 36-year-old woman comes to her physician complaining of heart palpitations, weight loss, anxiety, insomnia, fatigue, and amenorrhea. The physician palpates a 1.5-cm mass on her neck, which elevates when she swallows, located inferior to the cricoid cartilage yet off the midline. What is the most likely structure involved with her presentation? (A) Enlarged deep cervical lymph node (B) Thyroid nodule (C) Benign parathyroid adenoma (D) Thyroglossal duct cyst (E) Branchial cyst

The answer is B: Thyroid nodule. A thyroid nodule refers to any abnormal growth that forms within the thyroid gland. Adult women (4% to 8%) are particularly prone to thyroid nodules, but, fortunately, only 10% of thyroid nodules are reported to be cancerous. The majority of thyroid nodules are asymptomatic; however, if the cells of the thyroid nodule are producing thyroid hormones, either thyroxine (T4) or triiodothyronine (T3), a thy- roid nodule can lead to hyperthyroidism. Patient with hyper- thyroidism may present with heart palpitations, weight loss, anxiety, insomnia, fatigue, heat intolerance, excessive sweating, exophthalmos (protruding eyes), and even amenorrhea (an absence of menstrual flow). Due to the symptoms of this patient and the mass being located at the location of the thyroid gland, a thyroid nodule is most likely diagnosis in this patient. Choice A (Enlarged deep cervical lymph node) is incorrect. The deep cer- vical lymph nodes lie within or in close proximity to the carotid sheath correlating in location with the internal jugular vein. While a few of the deep cervical lymph nodes could reside near the location of the mass, an enlarged deep cervical lymph node would not lead to the symptoms that brought this patient to the doctor, which were characteristic of hyperthyroidism. Choice C (Benign parathyroid adenoma) is incorrect. A parathyroid adenoma is a benign tumor of the parathyroid glands, which usually increases the circulation of parathyroid hormone (PTH). This condition of hyperparathyroidism leads to an increase in blood calcium levels due to elevated resorption of bone and may be asymptomatic in many patients. Symptomatic patients would present with lethargy, muscle pain, nausea, constipation, confusion, kidney stones, and even an increased risk of bone fractures due to the increased bone resorption. A physician can perform blood tests to test for calcium, chloride, potassium, and bicarbonate levels, and women over the age of 60 have the high- est risk for developing hyperparathyroidism. The presence of a parathyroid adenoma has been reported in 80% to 85% of patients who present with hyperparathyroidism; however, the symptoms of this patient do not correlate with the presence of a parathyroid adenoma. Choice D (Thyroglossal duct cyst) is incorrect. The thyroid gland first appears as a single, ventro- median diverticulum (thyroid diverticulum) off the floor of the embryonic pharynx, between the tuberculum impar and copula of the incipient tongue. It descends along the midline, anterior to the gut tube, remaining connected to the tongue by a nar- row canal (the thyroglossal duct). The thyroglossal duct usually solidifies and is obliterated after the final descent of the thyroid gland into its normal terminal position. A thyroglossal cyst is a cystic remnant of the thyroglossal duct. It is always located in or close to the midline of the neck. Most commonly (∼50%), it is found near the body of the hyoid bone. Because the mass in this patient is located off the midline, a thyroglossal duct cyst is unlikely. Moreover, a thyroglossal cyst would not cause the symptoms seen in this patient. Choice E (Branchial cyst) is incorrect. Branchial cysts are located along the anterior border of the sternocleidomastoid muscle. Most often, these cysts are the remnants of the second pharyngeal cleft, located just below the angle of the mandible. Second branchial cleft cysts represent approximately 67% to 93% of all pharyngeal apparatus anoma- lies. However, branchial cysts may be found anywhere along the anterior margin of the sternocleidomastoid muscle. Very frequently, branchial cysts are inconspicuous at birth, becom- ing evident as they enlarge throughout childhood. Due to the location of the mass, symptoms of the patient, and age of the patient, a branchial cyst can be ruled out in this patient.

During a difficult childbirth, a physician uses obstetric forceps to grip the infant's head as an aid to extracting her from the birth canal. However, the forceps are misapplied at the right stylomastoid foramen and crush its contents at the opening of the foramen. Which of the following ipsilateral deficits is the baby most likely to suffer? (A) Reduced blood flow to the inner ear (B) No sensation in the external acoustic meatus (C) Inability to close the eyelids (D) Lack of taste on the body of the tongue (E) Inability to tense the eardrum

The answer is C: Inability to close the eyelids. The stylomas- toid foramen is located in a well-protected position between the styloid and mastoid processes, at the base of the skull. It is the terminal opening of the facial canal, transmitting the main branch of the facial nerve out of the petrous temporal bone to the exterior base of the skull. From there, the facial nerve sends sensory branches to the external ear, motor branches to small muscles in the upper neck (stylohyoid and posterior belly of the digastric), and terminal motor branches to the facial muscles (muscles of facial expression). However, the mastoid process is not developed at birth, leaving the stylomastoid foramen and the emerging facial nerve exposed. Undue pres- sure applied to the mastoid area (as from misapplied obstet- ric forceps) may entrap and damage the facial nerve. In this case, crush injury to the facial nerve would produce unilateral paralysis of the facial muscles, including the orbicularis oculi. This sphincter-like muscle surrounds the orbit, acting to close the eyelids. Choice A (Reduced blood flow to the inner ear) is incorrect. Vascular supply to the inner ear is provided by small labyrinthine vessels that accompany the facial and ves- tibulocochlear nerves through the internal acoustic meatus. Supply to the external ear is mainly from branches of the pos- terior auricular and superficial temporal arteries. The posterior auricular arterial branching is susceptible to damage in this scenario. Supply to the middle ear is mainly from branches of the maxillary artery. These vessels are unlikely to be affected by compression around the incipient mastoid process. Choice B (No sensation in the external acoustic meatus) is incorrect. The external ear receives afferent innervation from a plethora of cranial nerves, including trigeminal (CN V), facial (CN VII), vagus (CN X), and possibly even the glossopharyngeal (CN IX). The trigeminal and vagus are the major nerves, supplying most of the sensory fibers to this region, including the exter- nal acoustic meatus. The facial nerve provides relatively small contributions to the sensation of the external acoustic meatus because its sensory innervation is concentrated on the auricle. Choice D (Lack of taste on the body of the tongue) is incor- rect. The facial nerve provides taste fibers to the body (anterior two thirds) of the tongue and parasympathetic fibers to the oral floor via its chorda tympani branch. However, the chorda tympani nerve leaves the facial nerve within the facial canal, traverses the tympanic (middle ear) cavity, and exits the base of the skull near the temporomandibular joint (TMJ). Therefore, this forceps injury would not affect the taste or parasympathetic functions governed by the facial nerve. Choice E (Inability to tense the eardrum) is incorrect. The tensor tympani muscle attaches to the handle of the malleus, acting to pull this ear ossicle medially and tense the tympanic membrane (eardrum). This muscle is innervated by the mandibular division of the trigeminal nerve (CN V3). The stapedius muscle attaches to the stapes, acting to tighten the oval window. This muscle is controlled by the facial nerve, via a branch off the facial canal, proximal to the stylomastoid foramen.

Genetic testing of a baby boy with facial and cardiovascular anomalies reveals a small deletion in chromosome 22, specifically 22q11.2. This 22q11.2 deletion (DiGeorge) syndrome often results in migration defects of neural crest cells within the pharyngeal pouches. In this DiGeorge syndrome patient, the thymus and inferior parathyroid glands are absent. Which of the following pharyngeal pouches is most likely affected? (A) First (B) Second (C) Third (D) Fourth (E) Fifth

The answer is C: Third. The thymus and inferior parathyroid glands are derived from the third pharyngeal pouch. Therefore, migration of neural crest cells into the third pharyngeal pouch has been diminished in this patient with DiGeorge (22q11.2 deletion) syndrome. The patient would present with immu- nodeficiency due to an absence of the thymus, and hypocalce- mia (low blood calcium levels) due to the absence of inferior parathyroid glands. DiGeorge syndrome is also characterized by conotruncal hearts defects, such as tetralogy of Fallot or interrupted aortic arch. Moreover, facial anomalies usually resemble first arch syndrome with the patient presenting with a hypoplastic mandible, low-set ears, hypertelorism (increased distance between the eyes), and microstomia (abnormally small mouth). Choice A (First) is incorrect. Evagination of the first pharyngeal pouch forms an elongated diverticulum off the pharynx (the tubotympanic recess). The distal part of this recess widens to form the tympanic (middle ear) cavity. The proximal part remains more tubular and forms the pharyngotympanic (auditory; eustachian) tube. Therefore, agenesis of the thymus and inferior parathyroid glands is not due to maldevelopment of the first pharyngeal pouch. Choice B (Second) is incorrect. The second pharyngeal pouch helps to form the tonsillar fossa and surface epithelium of the palatine tonsil. Secondarily, lym- phatic tissue, which becomes the palatine tonsils, is incorpo- rated into the second pharyngeal pouch. Maldevelopment of the second pharyngeal pouch does not cause agenesis of the thymus and inferior parathyroid glands. Choice D (Fourth) is incorrect. The fourth pharyngeal pouch forms the supe- rior parathyroid gland and the ultimobranchial body, which gives rise to the parafollicular (C) cells of the thyroid gland. Maldevelopment of the fourth pharyngeal pouch does not cause agenesis of the thymus and inferior parathyroid glands. Choice E (Fifth) is incorrect. The fifth pharyngeal pouch only exists transiently in human embryologic growth and develop- ment. No adult structures are derived from the fifth pharyngeal pouch, so this option can be easily eliminated.

A physician examines a 53-year-old woman and notes deviation of the uvula to the right and asymmetry in the elevation of the soft palate, with the palatal arch of the left side sagging when compared to the right. The muscles involved in these abnormal findings are most likely derived from the mesoderm of what pharyngeal arch? (A) First (B) Second (C) Third (D) Fourth (E) Sixth

The answer is D: Fourth. This patient is exhibiting atrophy of the muscles on the left side of the soft palate, which are derived from the mesoderm of the fourth pharyngeal arch. In this patient, the palatal arch droops on the affected (left) side, and the uvula deviates to the unaffected (right) side as a result of the unopposed action of the intact (contralateral) muscles acting on the soft palate. This patient presentation would be indicative of a left vagus nerve (CN X) injury. Remember that the pharyngeal branches of CN X innervate all of the musculature of the soft palate, except the tensor veli palatini (tensor of the soft palate), which is innervated by the mandibular division of the trigemi- nal nerve (CN V3). Besides forming most of the muscles of the soft palate, the mesoderm of the fourth pharyngeal arch would give rise to the muscles of the pharynx (with the exception of the stylopharyngeus muscle) and the cricothyroid, a muscle of the larynx. Choice A (First) is incorrect. The muscles derived from the mesoderm of the first pharyngeal arch include four muscles of mastication (temporalis, masseter, lateral pterygoid, and medial pterygoid) and four additional muscles: Mylohoid, Anterior belly of the Digastric, Tensor Tympani, and Tensor Veli Palatini (mnemonic = "MATT"). The tensor veli palatini muscle is the only muscle of the soft palate derived from the first pharyngeal arch. However, damage to the first pharyangeal arch musculature would not affect elevation of the soft palate or deviation of the uvula. Choice B (Second) is incorrect. The muscles derived from the mesoderm of the second pharyngeal arch include the muscles of facial expression and three addi- tional muscles: posterior belly of digastric, stylohyoid, and sta- pedius. None of these listed muscles are associated with the soft palate, so they play no role in this patient's presentation. Choice C (Third) is incorrect. The only muscle derived from the mesoderm of the third pharyngeal arch is the stylopharyn- geus, a muscle of the pharynx. The stylopharyngeus is the only muscle innervated by the glossopharyngeal nerve (CN IX). Because this muscle is not associated with the soft palate, it plays no role in this patient's presentation. Choice E (Sixth) is incorrect. The muscles derived from the mesoderm of the sixth pharyngeal arch include the intrinsic muscles of the larynx and the skeletal muscle of the upper esophagus. Because these muscles are not involved with the soft palate, they play no role in this patient's presentation.

The external laryngeal nerve of a 23-year-old man becomes ensnared and tightly compressed by a tortuous superior thyroid artery, which parallels the course of this nerve. Which of the following functions is most likely to be affected? (A) Sensation above the true vocal fold (B) Sensation below the true vocal fold (C) Abduction of the vocal cord (D) Tension of the vocal cord (E) Depression of the hyoid bone

The answer is D: Tension of the vocal cord. The superior thyroid artery accompanies the external laryngeal nerve (or external branch of the superior laryngeal nerve) as they descend from their origins high in the neck. The artery sup- plies the superior pole of the thyroid gland while the external laryngeal nerve innervates the cricothyroid muscle in the lar- ynx and the lower part of the inferior pharyngeal constric- tor. The cricothyroid muscle tilts the thyroid cartilage forward (anterior), causing elongation, tension, and adduction of the vocal cords. Thus, compression of the external laryngeal nerve may reduce the tension of the ipsilateral vocal cord, causing a weakened voice, slight hoarseness, and difficulty raising the pitch of the voice. Choice A (Sensation above the true vocal fold) is incorrect. The true vocal folds (or simply vocal folds) mark the line of separation between the two sensory fields within the larynx. Sensation from the mucosa above the vocal fold (i.e., lining the inlet, vestibule, vestibular folds, and ven- tricle) is carried by the internal laryngeal nerve, a terminal branch of the superior laryngeal nerve. The external laryngeal nerve is the other terminal branch of the superior laryngeal nerve, and it innervates the cricothyroid and lower part of the inferior pharyngeal constrictor muscles. Choice B (Sensation below the true vocal fold) is incorrect. The (true) vocal folds mark the line of separation between the two sensory fields within the larynx. Sensation from the mucosa below the vocal fold (i.e., lining the infraglottic cavity) is carried by the inferior laryngeal nerve, the terminal branch of the recurrent laryngeal nerve. Choice C (Abduction of the vocal cord) is incorrect. Abduction of the vocal cord (i.e., opening of the rima glottidis) is caused by the contraction of the posterior cricoarytenoid muscle, which laterally rotates the arytenoid cartilage. This muscle is controlled by the inferior laryngeal nerve. Remember that the inferior laryngeal (recurrent laryngeal) nerve supplies all the intrinsic laryngeal muscles except one, the cricothyroid. Choice E (Depression of the hyoid bone) is incorrect. Depres- sion of the hyoid is caused by the actions of the four infrahyoid (strap) muscles (Thyrohyoid, Omohyoid, Sternohyoid, and Sternothyroid). The mnemonic to remember the infrahyoid muscles is "TOSS". Three of the muscles (the "OSS" of the "TOSS") are innervated by the ansa cervicalis, a branch of the cervical plexus. The lone remaining muscle, the thyrohyoid muscle, is innervated by a C1 branch off the hypoglossal nerve (CN XII).

A 10-year-old boy was admitted to the hospital with a sore throat, earache, and high fever. On examination, he had severely swollen palatine tonsils (tonsillitis). What nerve carries the sensory input for most of the patient's symptoms? (A) Greater palatine (B) Lesser palatine (C) Vagus (D) Posterior superior alveolar (E) Glossopharyngeal

The answer is E: Glossopharyngeal. The glossopharyngeal nerve (CN IX) is responsible for visceral sensation to the posterior one third of the tongue, palatine tonsils, soft pal- ate, pharyngotympanic tube, tympanic (or middle ear) cav- ity, and pharynx. Therefore, CN IX would be responsible for the pain associated with the boy's earache and sore throat. The palatine tonsillitis may be blocking the opening of the pharyngotympanic tube (auditory or eustachian), exacerbat- ing the symptoms of the patient. The glossopharyngeal nerve is also responsible for the afferent limb of the gag reflex. Choice A (Greater palatine) is incorrect. The greater pala- tine nerve, a branch off the maxillary (second) division of the trigeminal nerve (CN V ), supplies sensory innervation be responsible for the pain associated with the boy's earache and sore throat. Choice B (Lesser palatine) is incorrect. The lesser palatine nerve, a branch of the maxillary division of the trigeminal nerve (CN V2), supplies sensory innervation to the soft palate and aspects of the palatine tonsil. However, it would not be responsible for the pain associated with the boy's earache and sore throat. Choice C (Vagus) is incorrect. The vagus nerve (CN X) serves as the efferent limb of the gag reflex because it gives motor innervation to a majority of muscles serving the soft palate, pharynx, and larynx. CN X does provide sensory (afferent) innervation in the larynx but not the areas affected in this patient. Choice D (Posterior superior alveolar) is incorrect. The posterior superior alveo- lar nerve, which branches off the maxillary division of the trigeminal nerve (CN V2), supplies sensory innervation to the gingivae and posterior teeth of maxilla. This nerve would not be responsible for the pain associated with the boy's earache and sore throat.

While participating in a bar fight, the orbit of a 25-year-old man is pierced by a broken pool cue stick, which extends back to the superior orbital fissure. Which of the following nerves is most likely damaged? (A) Optic nerve (B) Facial nerve (C) Mandibular division of trigeminal (D) Maxillary division of trigeminal (E) Ophthalmic division of trigeminal

The answer is E: Ophthalmic division of trigeminal. The oph- thalmic (first) division of trigeminal nerve (CN V1) supplies sensory (cutaneous) innervation to the skin of the upper eye- lid, anterior aspect of the nose, forehead, and anterior scalp. CN V1 enters the middle cranial fossa through the superior orbital fissure, and its three terminal branches (lacrimal, naso- ciliary, and frontal nerves) are distributed throughout the orbit, anterior cranial fossa, anterior scalp, and nasal cavity. Other cranial nerves traverse the superior orbital fissure, including the oculomotor (CN III), trochlear (CN IV), and abducent (CN VI) nerves, to supply extraocular eye muscles. The supe- rior orbital fissure also contains the superior and inferior oph- thalmic veins and sympathetic fibers from the carotid plexus. With the broken pool cue residing in close proximity to the superior orbital fissure, the ophthalmic division of the trigem- inal nerve is most likely damaged in this question. Choice A (Optic nerve) is incorrect. The optic nerve (CN II) leaves the orbit via the optic canal to relay vision from the retina to the brain. Because only the superior orbital fissure is involved in this patient, the optic nerve is not likely damaged. Choice B (Facial nerve) is incorrect. The facial nerve enters the petrous temporal bone through the internal acoustic meatus, traveling through this foramen with the vestibulocochlear nerve (CN VII). It has three terminal branches (chorda tympani, greater petrosal, and the main branch of the facial nerve), and none of these branches traverse the superior orbital fissure, which was involved in this patient. Choice C (Mandibular division of trigeminal nerve) is incorrect. The mandibular (third) division of the trigeminal nerve (CN V3) supplies general sensation to the skin of the lower lip, chin, cheek, and even the anterior auricular and lateral scalp. This sensory innervation is sup- plied via three cutaneous nerves: mental, buccal, and auricu- lotemporal. The mandibular division of the trigeminal nerve also supplies innervation to the mandibular teeth and gingivae via the inferior alveolar nerve, and it is the only division of CN V that supplies motor innervation. CN V3 enters the middle cranial fossa through the foramen ovale, so it would not be affected by damage in the superior orbital fissure. Choice D (Maxillary division of trigeminal nerve) is incorrect. The maxillary (second) division of the trigeminal nerve (CN V2) supplies sensory (cutaneous) innervation to the skin to the lower eyelid, cheek, upper lip, upper dentition and gingivae, and lateral aspects of the nose. CN V2 enters the middle cra- nial fossa through the foramen rotundum, so it would not be affected by damage in the superior orbital fissure.

Mandibulofacial dysostosis (Treacher-Collins syndrome) is a developmental disorder characterized by craniofacial deformities, including malformed or absent ears, zygomatic and mandibular hypoplasia, and downward slanting eyes exhibiting ptosis of the lateral eyelids. This condition is the result of lack of migration of neural crest cells into what pharyngeal arch? (A) First pharyngeal arch (B) Second pharyngeal arch (C) Third pharyngeal arch (D) Fourth pharyngeal arch (E) Fifth pharyngeal arch

The answer is A: First pharyngeal arch. Mandibulofacial dys- ostosis (Treacher-Collins syndrome) affects derivatives of the first pharyngeal arch, specifically the migration of the neural crest cells into the arch. The cranial neural crests of the first pharyngeal arch are responsible for formation of the mandible, malleus, incus, squamous portion of the temporal bone, tem- poral bone, palatine bone, and vomer. As the figure shows, this patient exhibits micrognathia (small lower jaw), malar (zygomatic) hypoplasia, and malformed external ears, and this craniofacial deformities clearly indicate problems with the neural crest migration of the first pharyngeal arch. Man- dibulofacial dysostosis is seen in 1 in 10,000 births. Choice B (Second pharyngeal arch) is incorrect. Neural crest cells of the second pharyngeal arch are responsible for formation of the stapes, styloid processes, and the upper body and lesser horns of the hyoid bone. Malformation of these structures derived from the second pharyngeal arch would not lead to the char- acteristic craniofacial deformities seen in a patient with man- dibulofacial dysostosis (Treacher-Collins syndrome). Choice C (Third pharyngeal arch) is incorrect. Neural crest cells of the third pharyngeal arch are responsible for formation of the lower body and greater horns of the hyoid bone. Malformation of these third pharyngeal arch structures would not lead to the characteristic craniofacial deformities seen in a patient with mandibulofacial dysostosis. Choice D (Fourth pharyngeal arch) is incorrect. Neural crest cells of the fourth pharyngeal arch are not responsible for any skeletal elements. Therefore, failure of the neural crest cell migration of the fourth pharyn- geal arch structures would not lead to any craniofacial deformi- ties, and this option can be easily eliminated. Choice E (Fifth pharyngeal arch) is incorrect. The fifth pharyngeal arch only exists transiently in human embryologic growth and develop- ment. No structures are derived from the fifth pharyngeal arch in adults, so this option can be easily eliminated.

A 17-year-old girl uses Accutane, an acne drug implicated in interfering with normal development of the facial primordia, early in her unexpected pregnancy. Examination of the newborn reveals the craniofacial defect of an anterior palatal cleft. In this case, the defect is related to failure of fusion of what craniofacial processes? (A) Maxillary and medial nasal processes (B) Opposite maxillary processes (C) Medial and lateral nasal processes (D) Maxillary and lateral nasal processes (E) Opposite medial nasal processes

The answer is A: Maxillary and medial nasal processes. The face and palate are formed from the differentiation, growth, and merging of five facial primordia (facial prominences; facial swellings; facial processes): the single frontal (fronto- nasal) prominence and the paired maxillary and mandibular prominences (see figure on next page). The frontal promi- nence secondarily gives rise to paired nasal placodes and their bordering medial and lateral nasal prominences. Failure of the facial prominences to merge and fuse results in gaps (clefts) left remaining between primordial tissue zones. These gaps are the basis for palatofacial clefts of varying location and severity. The infant in this case suffers an anterior palatal cleft (cleft lip; harelip). Anterior clefts are located anterior to the incisive foramen. That location plus its position lateral to the midline indicates the defect was caused by incomplete fusion of the maxillary and medial nasal processes. Here, the defect is rela- tively small and unilateral. In more severe cases, the cleft may extend deeper and/or occur bilaterally. Cleft lip is a relatively common condition (1:1,000 births) that occurs more often in males and has an incidence that increases slightly with mater- nal age. Unilateral cleft lip is the most common craniofacial congenital defect. Choice B (Opposite maxillary processes) is incorrect. The opposite maxillary processes normally develop horizontal palatine shelves that fuse with each another in the midline to form the secondary palate (i.e., the bulk of the hard and soft palate). Defects in this fusion are termed posterior clefts (cleft palate) because they are located posterior to the incisive foramen. They vary in degree from relatively minor (cleft of only the uvula) to severe (cleft of the entire second- ary palate). Posterior clefts occur less often than anterior clefts (1:2,500 births), occur more often in females, and are not related to maternal age. Choice C (Medial and lateral nasal processes) is incorrect. The medial and lateral nasal processes border each nasal placode. The tissue between each pair of nasal processes invaginates and canalizes to eventually form a nasal passage (i.e., the airway from naris to choana). Thus, the nasal processes fuse with each other only at their upper and lower margins to allow formation of the nasal passages. Choice D (Maxillary and lateral nasal processes) is incorrect. The maxillary prominence normally overgrows the nasolacri- mal groove to fuse with its matching lateral nasal prominence. This development causes the nasolacrimal groove to sink into the face, where it canalizes and forms the nasolacrimal duct. Failure of this fusion is rare. When occurring, the result is an oblique facial cleft. In this craniofacial malformation, the defect runs from the medial canthus of the eye into the upper lip, with the nasolacrimal duct typically exposed to the sur- face. Choice E (Opposite medial nasal processes) is incorrect. The two medial nasal processes normally fuse in the mid- line to form the bridge of the nose, the nasal septum, and the intermaxillary segment of the face (philtrum of the upper lip; premaxilla; primary palate). Failure in this fusion is rare. The resultant malformation may be a midline grooved or cleft (bifid) nose and/or defects in the intermaxillary segment of the face. Such conditions may indicate a more extensive loss of midline tissue (including neural tissue), characteristic of holoprosencephaly.

A 9-year-old girl suffers from the mumps, causing unilateral inflammation of her right parotid gland. The girl is experiencing great pain due to the stretched capsule of the parotid gland. Her physician is also concerned about the condition of the structures contained within the gland. Which of the following structures may be directly compressed in this situation? (A) Buccal nerve of CN V3 (B) Auriculotemporal nerve (C) Posterior belly of the digastric muscle (D) External jugular vein (E) Facial artery

The answer is B: Auriculotemporal nerve. The parotid is the largest salivary gland, occupying much of the retromandibu- lar space. It is enclosed by two capsules: (1) the connective tissue capsule of the gland itself and (2) a dense fibrous cap- sule derived from the investing layer of deep cervical fascia. As a result, inflammation and swelling of the parotid, caused by mumps in this patient, can increase pressure significantly within the tightly bound glandular mass. Several structures are contained within the parotid as they pass across the face. Each of these structures is subject to undue pressure in dis- ease conditions and must be carefully considered in surgery of this salivary gland. The auriculotemporal nerve, a branch of the mandibular division of the trigeminal nerve (CN V3), traverses the substance of the parotid in its course from the infratemporal fossa to the lateral scalp. This nerve is the major sensory supply across the mandibular region, provides the parasympathetic secretomotor route to the parotid, and inner- vates the connective tissue within the gland itself. Irritation of the auriculotemporal nerve due to inflammation and swell- ing of the parotid produces severe pain. Pain would also be derived from the great auricular nerve, a branch of the cervical plexus composed of fibers from the C2 and C3 spinal nerves, which innervates the parotid sheath as well as the skin overly- ing the angle of the mandible and inferior lobe of the auricle. Other structures contained within the parotid include the facial nerve, the formation of the retromandibular vein (super- ficial temporal, maxillary, and retromandibular veins), and the termination of the external carotid artery (external carotid, superficial temporal, and maxillary arteries). Choice A (Buc- cal nerve of CN V3) is incorrect. The buccal nerve is a sensory (afferent) branch of the mandibular division of the trigeminal nerve (CN V3). It passes anterior through the infratemporal fossa, emerges from under the masseter muscle, and distrib- utes across the cheek. Dentists often refer to this nerve as the long buccal nerve. In this case, it lies anterior to the parotid gland and its enveloping capsules. Choice C (Posterior belly of the digastric muscle) is incorrect. The posterior belly of the digastric originates from the deep aspect of the mastoid pro- cess and passes into the neck to attach to the hyoid bone. It would not be directly compressed by the inflammation of the parotid sheath due to its location inferior to the parotid gland. Choice D (External jugular vein) is incorrect. The external jugular vein is formed in the upper neck by the union of the posterior branch of the retromandibular vein with the posterior auricular vein inferior to the parotid gland. It would not be directly compressed by the inflammation of the parotid sheath. Choice E (Facial artery) is incorrect. The facial artery originates in the upper neck, from the external carotid artery. It ascends to cross the mandible at the anterior edge of the masseter muscle, anterior to the parotid gland, to enter the face.

A 59-year-old man with a herpes zoster infection within the mandibular division of the trigeminal nerve (CN V3) complains of weakness when opening his mouth. A comprehensive evaluation reveals that his problems are due to difficulty protruding the mandible, and when protrusion is accomplished the mandible deviates to the left side. What muscle is most likely weakened? (A) Anterior belly of digastric (B) Lateral pterygoid (C) Masseter (D) Medial pterygoid (E) Temporalis

The answer is B: Lateral pterygoid. The lateral pterygoid muscle acts on the temporomandibular joint (TMJ) to cause protrusion (or protraction) of the mandible. During the con- traction of the lateral pterygoid muscle, the mandibular con- dyle slides anterior (translation) to be located inferior to the articular eminence of the temporal bone, which enables the mouth to open passively due to gravity. Acting unilaterally, the lateral pterygoid muscle produces side-to-side move- ments. So, the patient has all of the signs of a paralyzed lateral pterygoid muscle on the left side, including weakness pro- truding the mandible, weakness opening the oral fissure (or mouth), and the lateral deviation of the mandible during pro- trusion. In this patient, herpes zoster, or shingles, is a painful skin rash affecting the mandibular division (or motor root) of the trigeminal nerve (CN V3). Shingles is seen in patients who have had previous exposure to the varicella zoster virus, which causes chickenpox in children or young adults. After the initial exposure to chickenpox, this virus can reside latent in ganglia of an individual for years. If this individual becomes immunocompromised, the skin (or dermatomes supplied by the infected ganglia) can develop shingles, a painful skin rash, which blisters, breaks open, crusts over, and then disap- pears. In this patient, the herpetic lesions were found in the sensory distribution of the left CN V3, which means the virus resides in the trigeminal (or semilunar) ganglion. This herpes zoster infection also affected the motor root of CN V3, which is why the lateral pterygoid muscle displayed weakness in this patient. Choice A (Anterior belly of digastric) is incorrect. The anterior belly of the digastric muscle is a suprahyoid muscle that assists in the elevation of the hyoid bone during swal- lowing. It has no role in the deviation of the mandible during protrusion. Choice C (Masseter) is incorrect. The masseter primarily works to close the jaw. Though its superficial fibers may play a limited role in protrusion of the mandible, it is the deviation of the mandible to the left that signals involvement of the left lateral pterygoid muscle. Choice D (Medial ptery- goid) is incorrect. The medial pterygoid functions to elevate the mandible. Though it may play a limited role in protrusion of the mandible, it is the deviation of the mandible to the left that signals involvement of the left lateral pterygoid muscle. Choice E (Temporalis) is incorrect. The temporalis muscle is also involved with elevation of the mandible leading to closure of the jaw; however, its middle and oblique fibers are the pri- mary retractors of the mandible. These actions did not display weakness in this patient.

A 33-year-old woman presents with rapid weight gain, particularly in the trunk and face with sparing of the limbs, excess sweating, thinning of the skin, and hirsutism (facial male-pattern hair growth). A full examination also reveals bitemporal hemianopsia (or tunnel vision). What of the following labeled areas on the given X-ray of the lateral skull will be of most interest to the physician? (A) Frontal sinus (B) Floor of the anterior cranial fossa (C) Hypophyseal fossa. (D) Sphenoidal sinus (E) Mastoid sinuses

The answer is C: Hypophyseal fossa. This patient is exhibit- ing symptoms associated with Cushing disease, which results from the presence of a noncancerous tumor (adenoma) in the pituitary gland. The pituitary gland (or hypophysis) lies in the hypophyseal fossa of the sphenoid bone, which is identi- fied by the "C" in this standard lateral plane film (X-ray) of the head. This fossa is part of the sella turcica, a complexly shaped part of the sphenoid located in the center of the floor of the cranial cavity. Cushing disease, also called pituitary adrenocor- ticotropic hormone (ACTH) hypersecretion, causes elevated cortisol levels, which lead to rapid weight gain, particularly in the face and trunk with sparing of the limbs (central obe- sity), excess sweating, thinning of the skin, muscle weakness, and hirsutism (facial hair growth). These symptoms are all part of Cushing syndrome, but Cushing disease refers specifi- cally to the presence of a pituitary adenoma. The presence of a pituitary adenoma can be confirmed with an MRI of the pitu- itary gland. In this patient, the expanding pituitary adenoma expands upward out of the hypophyseal fossa to impinge about the optic chiasm, causing the bitemporal hemianop- sia (tunnel vision). This visual deficit further implicates the pituitary gland as the location of the mass. The pituitary fossa (and its bounding parts of the sella turcica) is clearly defined and serves as a major orientation point in a lateral X-ray of the head. Choice A (Frontal sinus) is incorrect. The paranasal sinuses are radiolucent areas that may be mistaken for cranial fossae in some cases. The frontal sinus is located within the frontal bone. It may be more or less distinct, depending on its size and the thickness of its walls. Due to the signs and symptoms of this patient, the frontal sinuses would have no involvement in the diagnosis of Cushing disease. Choice B (Floor of the anterior cranial fossa) is incorrect. This obliquely lying plate of bone is formed mainly by the lesser wing of the sphenoid and the orbital plate of the frontal bone. It forms both the floor of the anterior cranial fossa and the roof of the orbit. In this view, it extends anterosuperior from the sella tur- cica. Due to the signs and symptoms of this patient, the floor of the anterior cranial fossa would have no involvement in the diagnosis of Cushing disease. Choice D (Sphenoidal sinus) is incorrect. This paranasal sinus lies within the body of the sphenoid bone, inferior to the hypophyseal fossa. Notice that the transnasal approach to the pituitary gland requires the sur- geon to penetrate the sphenoid sinus but only as a means to reach the hypophyseal fossa. Due to the signs and symptoms of this patient, the sphenoidal sinus would have no involve- ment in the diagnosis of Cushing disease. Choice E (Mastoid sinuses) is incorrect. The mastoid sinuses are not paranasal sinuses. The mastoid sinuses are a complex of air cells within the mastoid part of the temporal bone, which connect with the tympanic (middle ear) cavity. Due to the signs and symptoms of this patient, the mastoid sinuses would have no involve- ment in the diagnosis of Cushing disease.

A newborn baby girl is unable to move her head to the right, even when her pediatrician tries to assist the movement. Her range of motion in the neck is limited in rotation and lateral bending, and her head posture is abnormally tilted toward the right and her chin is elevated and turned toward the left side. What muscle is most likely responsible for the baby's abnormal range of movement and head posture? (A) Platysma (B) Trapezius (C) Sternocleidomastoid (D) Masseter (E) Digastric

The answer is C: Sternocleidomastoid. The sternocleidomas- toid muscle (SCM) is abnormally shortened and/or exces- sively contracting in this baby girl with congenital torticollis (L: twisted neck). The etiology of congenital torticollis is unknown, but it is thought to be due to damage to the SCM during birth or intrauterine malposition. In this baby, the con- genital torticollis presents with the head tilted (or laterally bent) toward the affected SCM (right side in this patient) and the chin is elevated and turned toward the contralateral (left) side. Bilateral contraction of the SCM causes flexion of the neck to move the chin toward the sternum. When the right SCM contracts alone, it functions to bring the mastoid process of the temporal bone closer to the sternum, which results in tilting the head toward the right side and elevation of the chin to the left. The excessive contraction (or tone) of the right SCM causes the inability of this baby to have her head turned to the right side. The accessory nerve (CN XI) provides motor innervation to the trapezius and SCMs, and the congenital tor- ticollis, seen in this patient, is due to shortening or excessive contraction of the right SCM. Choice A (Platysma) is incor- rect. The platysma is a muscle of facial expression that resides in the neck and lower face to depress the mandible and wrinkle the skin of neck, as seen when a person is placed in a stressful situation. The platysma originates in the subcutaneous tissue near the clavicle and inserts into the modiolus, lateral to the labial commissures. This muscle is innervated by the cervical branch of the main branch of the facial nerve (CN VII). Due to its superficial origin in the subcutaneous fascia, this muscle can only wrinkle the skin of the neck, not abnormally twist it. This baby has congenital torticollis due to the shortening or excessive contraction of the SCM. Choice B (Trapezius) is incorrect. Damage to the trapezius muscle would lead to asymmetry when shrugging the shoulders or "drooping" of the affected shoulder because the actions of this muscle include elevation and lateral rotation of the scapula during abduction of the upper limb to greater than 90 degrees. The accessory nerve (CN XI) provides motor innervation to the trapezius and SCMs; however, the congenital torticollis, seen in this patient, is due to the shortening or excessive contraction of the SCM. Choice D (Masseter) is incorrect. The masseter muscle primarily works to close the jaw, though its superficial fibers may play a limited role in protrusion of the mandible. The mandibular (third) division of the trigeminal nerve (CN V3) supplies this muscle of mastication, which would have no role in the abnormal twisting of the neck and head posture seen in this newborn. Choice E (Digastric) is incorrect. The digastric muscle is a suprahyoid muscle that attaches to the body and greater horn of the hyoid bone and lies below the body of the mandible. This muscle consists of two muscle bellies, anterior and posterior, which are innervated by the mandibular (third) division of the trigeminal nerve (CN V3) and facial nerve (CN VII), respectively. When it contracts, the digastric muscle acts to elevate the hyoid bone, which is important in swallowing (or deglutition); it is not involved with rotation and lateral bending of the neck.

A 21-year-old professional boxer receives a series of powerful punches to the side of his face, which fractures the left mandible slightly superior to the mandibular angle. Resultant muscle spasticity causes his jaw to close, making it difficult to remove his mouthpiece. Which of the following muscles is acting to close the jaw? (A) Temporalis (B) Lateral pterygoid (C) Zygomaticus major (D) Masseter (E) Posterior digastric

The answer is D: Masseter. The masseter muscle, one of four muscles of mastication, is a powerful elevator (adductor) of the mandible, acting to close the mouth and approximate the occlusal surfaces of the teeth. The provided CT shows the left mandible is fractured slightly superior to the mandibular angle, causing the associated edema evident on the image. The masseter muscle, lying laterally to the fracture, originates from the inferior border and medial surface of the zygomatic arch and inserts into the angle, lower lateral surface of the ramus, and posterior aspect of the body of the mandible. This man- dibular fracture is located in the midpoint of the insertion of the masseter, which is more clearly seen on the right (unaffected) side of the CT scan. Because a large part of the insertion of the masseter inserts distal to the fracture site, it is capable of clos- ing the mouth making it difficult to remove the mouthpiece. Of note, the medial pterygoid muscle is a deep mirror of the masseter in its insertion and action, so both the masseter and medial pterygoid muscles would be acting to close the mouth in this boxer. Choice A (Temporalis) is incorrect. The temporalis muscle, another muscle of mastication, runs from the temporal fossa to the coronoid process and anterior edge of the ramus of the mandible. It also acts as an elevator (adductor) of the man- dible, to close the mouth. In this CT, the fracture is distal to the attachment of the temporalis, so this muscle could only elevate the fractured ramus of the mandible, which would not effec- tively close the mouth and approximate the occlusal surfaces of the teeth. Due to the fracture site, the spasticity of the masseter muscle would result in the closing of the mouth making it diffi- cult to remove the mouthpiece. Choice B (Lateral pterygoid) is incorrect. The lateral pterygoid is a muscle of mastication which acts to protract (or protrude) the mandible. It plays a significant role in opening of the mouth and medial-lateral excursion. In this case, the lateral pterygoid can move the proximal detached segment of the jaw because of its attachment above the fracture, to the neck of the mandible and the jaw joint capsule and disc. Choice C (Zygomaticus major) is incorrect. The zygomaticus major is a muscle of facial expression. It originates on the lat- eral aspect of the zygomatic bone (explaining its name) and inserts into the skin and other facial muscles at the corner of the mouth. It functions as a dilator of the oral fissure by elevat- ing the corners of mouth, as in smiling when the muscle con- tracts bilaterally or sneering to show disdain when the muscle contracts unilaterally. This muscle would be paralyzed in facial nerve palsy, but it would not be affected by this mandibular fracture. Choice E (Posterior digastric) is incorrect. The poste- rior digastric (or posterior belly of the digastric) is a suprahyoid muscle, not a muscle of mastication. It runs from the mastoid part of the skull to the hyoid bone, and acts to stabilize and elevate the hyoid during swallowing. Its counterpart, the ante- rior digastric, may play a role in opening the mouth, especially against resistance

A 35-year-old woman comes to her dentist complaining of tenderness and pain in her cheek near the parotid gland, as well as bad breath and a foul-tasting mouth at meal times. A radiopaque fluid is injected into the parotid duct system through cannulation, and this sialography of the parotid duct confirms blockage by a calculus (sialolith). What muscle, through which the parotid duct passes, is most likely causing the stenosis where the sialolith now resides, blocking the drainage of the parotid duct? (A) Buccinator (B) Mentalis (C) Temporalis (D) Orbicularis oris (E) Masseter

The answer is A: Buccinator. The buccinator muscle is pierced by the duct of the parotid gland as it passes toward the oral cavity to drain in the oral vestibule via a small opening opposite the second maxillary molar teeth. The muscular tone of the buccinator muscle can create a stenosis of the parotid duct, and it is the most likely site for a sialolith or calculus (L. pebble) of the parotid duct. The parotid duct arises from the anterior aspect of the parotid gland, and it courses across the superficial surface of the masseter muscle. Then, it dives medially to pierce the buccinator muscle and enters the oral cavity to drain at the second maxillary molar tooth. If sialo- lithiasis is confirmed, having a patient eat sour foods may dislodge the sialolith; however, surgical intervention may also be required to remove the calculus from the parotid duct. Choice B (Mentalis) is incorrect. The mentalis is a muscle of facial expression located on the chin that elevates and pro- trudes the lower lip or elevates the skin of the chin. It origi- nates from the incisive fossa of the mandible and contracts to show doubt in a patient. This muscle is not pierced by the duct of the parotid gland, so it plays no role in the stenosis of the parotid duct or its obstruction by the sialolith. Choice C (Temporalis) is incorrect. The temporalis is a muscle of mas- tication that elevates the mandible in closing of the mouth, and its posterior, horizontally-oriented fibers serve as the pri- mary means of retruding (retracting) the mandible when it is in a protruded (protracted) position. This muscle arises from the temporal fossa and inserts into the tip and medial border of the coronoid process of the mandible. This muscle is not pierced by the duct of the parotid gland, so it would play no role in the stenosis of the parotid duct or its obstruction by the sialolith. Choice D (Orbicularis oris) is incorrect. The orbic- ularis oris is a muscle of facial expression that encircles the mouth to act as the sphincter of the oral fissure. It originates from the incisive fossae of the mandible and maxilla and con- tracts in a patient to close the oral fissure, protrude the lips (as in kissing, whistling, or sucking), and resists distension when blowing (as seen when playing the trumpet). This muscle is not pierced by the duct of the parotid gland, so it would play no role in the stenosis of the parotid duct or its obstruction by the sialolith. Choice E (Masseter) is incorrect. The masseter is a muscle of mastication that primarily works to close the jaw, but its superficial fibers also play a limited role in protrusion of the mandible. The duct of the parotid gland does course across the superficial surface of the masseter muscle, but this muscle would play no role in the stenosis of the parotid duct because the masseter is not pierced by it.

Lesion of the trunks of the brachial plexus is most likely to occur from a penetrating wound into which of the following labeled areas in the given drawing of the neck regions? (A) Occipital Triangle (B) Omoclavicular triangle (C) Submandibular triangle (D) Carotid triangle (E) Muscular triangle

A. Occipital Triangle The neck can be demar- cated into anterior and lateral cervical regions by the obliquely running sternocleidomastoid (SCM) muscle. Both of these regions can be further subdivided into smaller triangular areas relative to the SCM and other neck muscles. This sche- matic plan allows a topographic organization of the structures within the neck. The lateral cervical region (or posterior cer- vical) triangle is demarcated by the posterior border of the SCM, anterior border of the trapezius, and the middle part of the clavicle. It is subdivided into an upper, larger occipi- tal triangle and a lower, smaller omoclavicular (subclavian or supraclavicular) triangle by the course of the inferior belly of the omohyoid muscle. The roots of the brachial plexus emerge from the interscalene triangle (scalene hiatus) to form the three trunks of the brachial plexus in the occipital triangle. These trunks descend through the occipital triangle and form the divisions of the plexus as they approach the clavicle. Penetrat- ing wounds into the occipital triangle are highly likely to con- tact some part of the trunks of the brachial plexus. Choice B (Omoclavicular triangle) is incorrect. The omoclavicular (sub- clavian or supraclavicular) triangle is the lower, smaller part of the lateral cervical region (or posterior cervical triangle). Its major contents include the third part of the subclavian artery, the suprascapular artery, and supraclavicular lymph nodes. A deep penetrating wound here may damage the divi- sions of the brachial plexus. Choice C (Submandibular tri- angle) is incorrect. The anterior cervical region (or triangle) is demarcated by the anterior border of the SCM, inferior border of the mandible, and the anterior midline of the neck. It is subdivided into four regions (or triangles) by the courses of the superior belly of the omohyoid and both bellies of the digastric muscles: the submandibular, submental, carotid, and muscular triangles. The submandibular (digastric) triangle is bounded by the mandible and the two bellies of the digastric muscle. Its major contents include the submandibular salivary gland, hypoglossal nerve (CN XII), and portions of the facial artery and vein. Trauma with this triangle would not affect the brachial plexus. Choice D (Carotid triangle) is incorrect. The carotid triangle represents a crowded region that is bounded by the SCM, superior belly of the omohyoid, and posterior belly of the digastric muscles. It houses several major neu- rovascular structures, including the common carotid artery and its branches, internal jugular vein, vagus and hypoglos- sal nerves, ansa cervicalis, and deep cervical lymph nodes. Trauma with this triangle would not affect the brachial plexus. Choice E (Muscular triangle) is incorrect. The muscular trian- gle is a large area demarcated by the SCM, superior belly of the omohyoid, and the anterior midline of the neck. It is occupied mainly by the infrahyoid (strap) muscles, and also includes the larynx and the thyroid and parathyroid glands. Trauma with this triangle would not affect the brachial plexus.

A 62-year-old male factory worker went to his doctor complaining of a progressive hearing loss. Audiometric tests reveal an inability to detect high-frequency sound waves, but the rest of his hearing scores within the normal range. What is the most likely location of injury for this sensorineural hearing loss? (A) Tympanic membrane (B) External acoustic meatus (C) Hair cells in the apex of the cochlea (D) Hair cells located in the middle of the cochlea (E) Hair cells in the base of the cochlea

E. The answer is E: Hair cells in the base of the cochlea. The hair cells in the base of the cochlea detect high frequency sound tones, so the factory worker's inability to detect these types of sounds implies damage to these specific sensory receptors. Being detected in the basal turn of the cochlea, high-frequency sounds travel the shortest distance in the cochlea while low- frequency sounds travel the farthest to reach the apex of the cochlea. People with high-frequency hearing loss would have trouble detecting consonant sounds, such as distinguishing between the words "thaw", "raw", and "law", because conso- nants are high-frequency sound components of human speech. Based upon his age and occupation, his sensorineural hearing loss is probably due to extended exposure to loud equipment, which has damaged the hair cells specifically at the base of the cochlea. Choice A (Tympanic membrane) is incorrect. A tear within the tympanic membrane that would separate the outer and middle ear would represent a conductive hearing loss where the patient would have trouble hearing sounds with low amplitudes irrespective of the frequency. Because this patient is unable to detect only sounds at high frequencies, a conductive hearing loss is not likely. Choice B (External acoustic meatus) is incorrect. A blockage of the external acoustic meatus, a part of the outer ear, would represent a conductive hearing loss where the patient would have trouble hearing sounds with low amplitudes irrespective of the frequency. Because this patient is unable to detect only sounds at high frequencies, a conductive hearing loss is not likely. Choice C (Hair cells in the apex of the cochlea) is incorrect. The hair cells in the apex of the cochlea detect low-frequency sound tones because these sounds travel farther. Whales communicate with low- frequency vocalizations that can be heard over great distances, and this fact enables a student to remember that low-frequency sounds travel farther within the cochlea. Though this patient is experiencing a sensorineural hearing loss, it is the hair cells in the base of the cochlea that are damaged because he is unable to detect high-frequency sounds. Choice D (Hair cells located in the middle of the cochlea) is incorrect. The hair cells in the middle of the cochlea would detect sounds in the middle of a human's hearing capacity for sound frequencies. How- ever, based upon the patient's inability to hear high-frequency sounds, these particular hair cells would not be damaged in this case of sensorineural hearing loss.

A 25-year-old professional boxer loses a fight when he is rendered unconscious by his opponent. After he regains consciousness, the ringside physician notes the boxer has a severe headache, nausea, and even vomiting. Being concerned about intracranial trauma, what cranial nerve can be observed by the physician, without the aid of radiographic imaging, to gain more information on whether the boxer has increased intracranial pressure? (A) Optic nerve (B) Oculomotor nerve (C) Olfactory nerve (D) Trigeminal nerve (E) Trochlear nerve

The answer is A: Optic nerve. Unlike other cranial nerves, the optic nerve (CN II) develops as an anterior extension of the diencephalon, part of the forebrain. Due to this unique embryological origin, the optic nerve is enveloped with extension of the cranial meninges (dura, arachnoid, and pia mater) and contains cerebrospinal (CSF) fluid in its subarachnoid space. The optic nerve is formed when the axons of the retinal ganglion cells coalescence at the optic disc. A physician can view the optic disc by performing a fundoscopic examination of the back of the eye (or fundus) with an ophthalmoscope. This boxer exhibits signs of increased intracranial pressure (ICP), such as headache, nausea, and vomiting, so the physician can look at the optic disc to look for papilledema, or swelling of the optic disc, which is another sign of increased ICP. If the optic disc is elevated, swollen, darker than normal, or its edges are blurred, then the fundoscopic examination can reveal evidence of increased ICP. These changes in the optic disc are due to the pressure transmitted via the CSF within the optic nerve. Choice B (Oculomotor nerve) is incorrect. The oculomotor nerve, CN III, innervates most of the extraocular eye muscles (with the exception of superior oblique and lateral rectus muscles) and the levator palpebrae superioris, which elevates the upper eyelid. Damage to the left CN III leads to diplopia (due to the left pupil resting in an abducted and lateral position) and ptosis (drooping of the eye- lid) associated with the left eye. However, the physician would be unable to correlate damage to the oculomotor nerve with increased ICP. Choice C (Olfactory nerve) is incorrect. The olfactory nerve, CN I, provides olfaction (or a sense of smell) to the patient. Damage to this nerve causes anosmia, or loss of olfaction, which is frequently seen in patients with severe head trauma due to the delicate olfactory nerves traversing the cribiform plate of the ethmoid bone to enter the olfactory bulb. However, damage to CN I would imply head trauma due to the punches the boxer received, not increased ICP. Choice D (Trigeminal nerve) is incorrect. The trigeminal nerve (CN V) is the main sensory nerve for the face and scalp, and it has three divisions: the ophthalmic (CN V1), maxillary (CN V2), and mandibular (CN V3). CN V3 is the only division of the trigeminal nerve (CN V) that supplies motor innervation. It supplies the eight muscles derived from the mesoderm of the first pharyngeal arch, including the muscles of mastication. Damage to the trigeminal nerve causes numbness (paresthesia) and paralysis of the muscles of mastication; however, it would not give additional information concerning increased ICP. Choice E (Trochlear nerve) is incorrect. The trochlear nerve, CN IV, innervates only one muscle, the superior oblique muscle, in the orbit. This muscle pulls the eye inferolaterally, but it's clinically tested by asking the patient to look inferiorly after the left eye is placed in an adducted position. Damage to this nerve would lead to diplopia, but it would not give additional information concerning increased ICP.

A newborn infant exhibits labored, gasping breathes and unusual vocal tones when crying. Thorough examination reveals the child has malformed vocal folds due to displaced attachments of the vocal ligaments. Which of the following structures is most likely defective in this condition? (A) Arytenoid cartilages (B) Cricoid cartilage (C) Epiglottic cartilage (D) Corniculate cartilages (E) Cuneiform cartilages

The answer is A: Arytenoid cartilages. The paired vocal liga- ments (true vocal cords) are contained within the free, upper, thickened edges of the conus elasticus. Each of these ligaments stretches from its anterior attachment near the midline on the deep side or posterior surface of the thyroid cartilage to its posterior attachment on the vocal process of the arytenoid car- tilage. Rotation and translation of the arytenoid cartilages and tilting of the thyroid cartilage determine the tension and posi- tion of the vocal ligaments. These conditions control the size and shape of the rima glottidis and the tension and vibration of the vocal folds. Thus, malformed vocal ligaments affect both respi- ration and phonation. Choice B (Cricoid cartilage) is incorrect. The single (unpaired) cricoid cartilage is the only completely circular cartilage in the respiratory tract. It articulates with the thyroid and arytenoid cartilages via synovial joints that permit regulation of the vocal folds. Choice C (Epiglottic cartilage) is incorrect. The epiglottic cartilage is a single (unpaired), roughly spoon-shaped structure that attaches to the posterior surface of the thyroid cartilage. Elevation of the larynx against the back- ward movement of the tongue during swallowing allows the epiglottis to act like a trap door in closing the laryngeal inlet. Choice D (Corniculate cartilages) is incorrect. The corniculate cartilages are small, paired structures contained within the free margins of the aryepiglottic folds. They may aid in support- ing these folds. Choice E (Cuneiform cartilages) is incorrect. The cuneiform cartilages are small, paired structures contained within the free margins of the aryepiglottic folds. They may aid in supporting these folds.

A 36-year-old man flips over the handlebars of his motorcycle and falls on the asphalt pavement, striking his head. He was not wearing a helmet. Although alert after the fall, he has a clear nasal discharge that tests positive for glucose. The patient most likely has a fracture of which of the following bones? (A) Ethmoid (B) Vomer (C) Sphenoid (D) Maxilla (E) Frontal

The answer is A: Ethmoid. A fracture of the ethmoid bone, specifically its cribiform plate, which separates the nasal cav- ity from the anterior cranial fossa, would enable cerebrospi- nal fluid (CSF), the clear discharge that tests positive for glu- cose, to leak from the nose. The traumatic blow to the head has broken the cribiform plate of the ethmoid bone, which caused a communication between the patient's anterior cra- nial fossa and nasal cavity, which is noted by the black arrow in the given sagittal CT scan. This patient presents with CSF rhinorrhea, when can lead to meningitis and other intrac- ranial complications, and this condition can be lethal if not properly treated. The given sagittal CT shows several fracture sites, including within the cribiform plate of the ethmoid bone as well as fractures of the anterior and posterior walls of the frontal sinus. The cribiform plate is fractured in several loca- tions, and one of these fracture sites is indicated by the white arrow in the given CT. Choice B (Vomer) is incorrect. The unpaired vomer bone forms the bony posteroinferior compo- nent of the nasal septum. So, fracturing the vomer bone would not lead to the CSF rhinorrhea presentation of this patient. Choice C (Sphenoid) is incorrect. Portions of the sphenoid bone, specifically the crest and anterior part of the sphenoid body, do form a small component of the posterior roof of the nasal cavity. When a patient presents with CSF rhinorrhea, the cribiform plate of the ethmoid bone is the most likely fracture site, which will lead to a communication between the anterior cranial fossa and the nasal cavity. The given CT was added for visual evidence of this type of fracture. It should be noted that the sphenoid bone is fractured in this individual, which is typical of this type of trauma to the anterior skull base. Remember that the thinness of the cribiform plate of the ethmoid bone makes it more susceptible to injury, so the sphe- noid bone is not the best answer for this question. Choice D (Maxilla) is incorrect. The maxilla contributes to the anterolat- eral walls of the nasal cavity and forms most of the boundary between the nasal and oral cavities. Though it is susceptible to injury, fracturing the maxilla would not provide a communi- cation between the anterior cranial fossa and the nasal cavity or the CSF rhinorrhea seen in this patient. Choice E (Frontal) is incorrect. The nasal spine of the frontal bone does form a small part of the roof of the nasal cavity. However, this bone is not the BEST selection for this question as it does not con- tribute nearly as much to the roof as the ethmoid bone. The thinness of the cribiform plate of the ethmoid bone makes it a more likely candidate to cause CSF rhinorrhea after a fracture. The given CT was added for visual evidence of a fracture to the cribiform plate of the ethmoid bone. It should be noted that the frontal bone, specifically the anterior and posterior walls of the frontal sinus, is fractured in this individual.

Most senior citizens dislike the bass audio frequencies augmented by a large subwoofer in a teenager's car. But, one 68-year-old man came to his doctor complaining of hyperacusis, or heightened sensitivity to these loud, low-frequency sounds. The results from an audiometer hearing test reveal normal hearing for a man of his age. Given the presentation, what cranial nerve is most likely involved with his hyperacusis? (A) Facial nerve (B) Vestibulocochlear nerve (C) Glossopharyngeal nerve (D) Vagus nerve (E) Hypoglossal nerve

The answer is A: Facial Nerve Along with the muscles of facial expression, posterior belly of the digastric, and stylohyoid muscles, the facial nerve also innervates the stapedius muscle, which contracts to pull the stapes away from the oval window of the cochlea. This action lowers the amplitude of sounds waves and decreases the transmission of vibrations to the cochlea. Paralysis of the stapedius results in hyperacusis due to heightened reaction of the stapes to sound vibration. Therefore, a person would be more sensitive to loud sounds, particular low-frequency sounds, like the bass emitted from a subwoofer. The stapedius muscle contracts involuntarily, along the tensor tympani muscle innervated by CN V3, in response to high intensity sound waves, forming the acous- tic reflex. Choice B (Vestibulocochlear nerve) is incorrect. Because of the normal results from the audiometer hear- ing examination, the vestibulocochlear nerve (CN VIII) has not been damaged. However, the stapedius muscle, which decreases the impact of the footplate of the stapes on the oval window of the cochlea, has been injured. Given this patient's loss of the acoustic reflex, extended exposure to loud sounds could damage the cochlea in the future. Choice C (Glossopharyngeal nerve) is incorrect. The glossopharyngeal nerve (CN IX) supplies only one muscle, the stylopharyn- geus, and it is not involved with the acoustic reflex or sound transduction. So, the hyperacusis seen in this patient is not due to CN IX. Choice D (Vagus nerve) is incorrect. The vagus nerve (CN X) supplies all the muscles of the larynx as well as most of the muscles of soft palate (exception = tensor veli palatini of CN V3) and pharynx (exception = stylopharyngeus of CN IX). Despite its extensive motor innervation, the vagus nerve does not play a role in the acoustic reflex or sound conduction. Choice E (Hypoglossal nerve) is incorrect. The hypoglossal nerve (CN XII) innervates the intrinsic and (most of the) extrinsic muscles of tongue (exception = palatoglossus muscle of CN X). Losing innervation to the muscles of the tongue would cause ipsilateral tongue deviation, but would not cause hyperacusis.

A newborn infant has difficulty in breastfeeding due to unilateral atrophy in the area of the face occupied by the levator labii superioris, levator anguli oris, and lateral upper part of the orbicularis oris muscles. This region of the face is derived from which of the following embryonic sources? (A) Maxillary process (B) Mandibular process (C) Frontal process (D) Lateral nasal process (E) Medial nasal process

The answer is A: Maxillary process. The muscles in question are located in the midface/lateral upper lip region. This area is formed around the maxillary process of the first pharyngeal arch. The atrophied muscles in this patient are derived from mesoderm of the second pharyngeal arch, which migrates into all the facial primordia. However, the underlying mid- facial skeleton is derived from neural crest cells that migrate into the maxillary process portion of the first pharyngeal arch. Overall, the maxillary process gives rise to the lateral part of the upper lip, the midface, and the secondary palate. Choice B (Mandibular process) is incorrect. The mandibu- lar process, the lower portion of the first pharyngeal arch, gives rise to the lower face region, including the entire lower lip and lower jaw. Choice C (Frontal process) is incorrect. The frontal (frontonasal) process is the only facial primor- dial component not derived from the first pharyngeal arch. Rather, this unpaired element forms from mesenchymal pro- liferation ventral to the forebrain vesicle (prosencephalon). The frontal process gives rise to the upper face (forehead) and part of the nose (bridge and tip of the nose and rostral nasal septum). Choice D (Lateral nasal process) is incorrect. The nasal processes are the elevated rims of the nasal pla- codes that form in the frontal process. The lateral nasal pro- cess gives rise to a small part of the face, the sides of the nose and the alae. Choice E (Medial nasal process) is incorrect. The medial nasal process gives rise to significant structures in the midline of the face, including contributions to the bridge and tip of the nose and nasal septum and the entire intermax- illary segment of the face.

A 13-year-old girl visits her pediatrician complaining of loss of sensitivity in her lower eyelid, the skin below her right eye, and in her upper lip and teeth. Which of the following sites is the most likely location for the nerve lesion responsible for these signs and symptoms? (A) Foramen rotundum (B) Foramen spinosum (C) Superior orbital fissure (D) Foramen ovale (E) Optic canal

The answer is A: Foramen rotundum. The foramen rotundum is an opening in the greater wing of the sphenoid bone that enables the maxillary (second) division of the trigeminal nerve (CN V2) to pass into the middle cranial fossa. CN V2 supplies sensory (cutaneous) innervation to the skin to the lower eye- lid, cheek, upper lip, upper dentition and gingivae, and lateral aspects of the nose. Due to the sensory deficits within this patient, this nerve is damaged along its route, and the foramen rotundum is the most likely location. Choice B (Foramen spinosum) is incorrect. The foramen spinosum conveys the middle meningeal artery and the meningeal branch (nervus spinosum) of the mandibular division of the trigeminal nerve (CN V3). This artery and nerve supply the dura mater in the cranial cavity. Thus, this site has no relation to the deficits in this patient. Choice C (Superior orbital fissure) is incorrect. The superior orbital fissure is a large opening that conveys the oculomotor (CN III), trochlear (CN IV), ophthalmic division of the trigeminal (CN V1), and abducent (CN VI) nerves as well as the superior ophthalmic veins into the orbit. None of these structures have a functional role in the deficits in question. Choice D (Foramen ovale) is incorrect. The foramen ovale (or oval foramen) is an opening in the greater wing of the sphenoid bone that enables the mandibular (third) division of the trigeminal nerve (CN V3) and a small meningeal artery to pass into the middle cranial fossa. CN V3 supplies cutaneous (general) sensation to the lower lip, chin, cheek, and anterior auricular and posterior temporal regions. This sensory inner- vation is supplied via three cutaneous nerves: mental, buccal, and auriculotemporal. The mandibular division of the trigem- inal nerve also supplies innervation to the mandibular teeth and gingivae via the inferior alveolar nerve. Due to the listed sensory deficits in this patient, this nerve would not have been damaged along its route. Choice E (Optic canal) is incorrect. The optic canal carries the optic nerve (CN II) and ophthalmic artery into the orbit. Again, these structures have no bearing on the deficits in this patient.

A 12-year-old boy suffers a fracture of the floor of the right side of the middle cranial fossa during an automobile accident. Subsequent physical examination reveals he is devoid of emotional tearing on the ipsilateral side. Which of the following nerves is most likely damaged? (A) Greater petrosal nerve (B) Lesser petrosal nerve (C) Deep petrosal nerve (D) Lacrimalnerve (E) Chorda tympani nerve

The answer is A: Greater petrosal nerve. Secretomotor control of the lacrimal gland is provided by parasympathetic neurons derived from the facial nerve. These fibers branch from the facial nerve as the greater petrosal nerve at the geniculum of the facial canal, within the petrous part of the temporal bone. This parasympathetic nerve leaves the temporal bone and courses along the floor of the middle cranial fossa on its way to the foramen lacerum, pterygoid canal, and pterygo- palatine fossa. It is readily damaged in trauma to the floor of the middle cranial fossa. Lesion of the nerve will result in loss of emotional tearing plus reduced mucus secretion in the nasal cavity (dry nasal passages). Choice B (Lesser petrosal nerve) is incorrect. The lesser petrosal nerve is a parasympathetic bundle derived from the tympanic branch of the glossopha- ryngeal nerve in the middle ear. It is the presynaptic (pregan- glionic) element in the secretomotor pathway to the parotid gland via the otic ganglion. Choice C (Deep petrosal nerve) is incorrect. The deep petrosal nerve is a sympathetic bundle that branches off the carotid plexus at the exterior base of the skull. It joins with the greater petrosal nerve to form the nerve of the pterygoid canal that travels through the pterygoid canal into the pterygopalatine fossa. The deep petrosal nerve may be damaged in a fracture of the floor of the middle cranial fossa. However, it does not control lacrimation, and thus is not the element related to the deficit here. Obviously, you must take care to differentiate the three petrosal nerves in accounting for autonomic relations in the head. Choice D (Lacrimal nerve) is incorrect. The lacrimal nerve is a branch of the ophthalmic division of the trigeminal nerve within the orbit. It conveys general sensory fibers from the upper eyelid, conjunctiva, and lacrimal sac. Further, its distal portion carries the postsynap- tic parasympathetic secretomotor fibers to the lacrimal gland. However, the lacrimal nerve itself would not be damaged by a fracture within the middle cranial fossa. Choice E (Chorda tympani nerve) is incorrect. The chorda tympani nerve is a branch of the facial nerve that runs through the middle ear cavity and continues into the infratemporal fossa. It carries parasympathetic secretomotor fibers to two salivary glands, the submandibular and sublingual glands, via the submandib- ular ganglion. It also conveys taste sensation from the anterior part of the tongue.

A 22-year-old man receives a stab wound in the left anterior cervical region, at the C2 vertebral level. The wound was 3 cm deep and located anterior to the sternocleidomastoid muscle (SCM) and superior to the greater horn of the hyoid bone. During a postoperative examination, the patient displays dysarthria, or difficulty speaking. Which of the following structures is most likely damaged? (A) Hypoglossal nerve (B) Accessory nerve (C) Mandibular division of trigeminal nerve (D) Lingual branch of glossopharyngeal nerve (E) Roots of the brachial plexus

The answer is A: Hypoglossal nerve. The left hypoglossal nerve (CN XII) is located in the anterior cervical region at the location of the stab wound. CN XII innervates all of the intrin- sic muscles of the tongue and most of its extrinsic muscles with the lone exception being the palatoglossus muscle, innervated by the vagus nerve (CN X). Therefore, damage to the right CN XII would produce dysarthria, or difficult speaking, which was seen in this patient due to the loss of innervation to the tongue musculature. Though not noted in this patient, dam- age to the hypoglossal nerve also causes ipsilateral deviation of the tongue due to the unopposed muscular contractions of the contralateral genioglossus muscle. The mnemonic "The tongue licks the wound" will help you remember that the tongue deviates to the ipsilateral side in a lower motor neu- ron lesion of CN XII. Choice B (Accessory nerve) is incor- rect. The accessory nerve (CN XI) passes deep to the SCM and to the investing layer of deep cervical fascia and courses pos- terolaterally into the lateral cervical region (posterior triangle of neck). Cutting this nerve would not produce the dysarthria displayed in this patient because CN XI innervates only the SCM and the trapezius muscle, which are primarily involved in head and shoulder movements. Also, the accessory nerve is not located in the anterior cervical region in the loca- tion of the stab wound. Choice C (Mandibular division of trigeminal nerve) is incorrect. The mandibular (third) divi- sion of the trigeminal nerve (CN V3) is the only division of the trigeminal nerve (CN V) that supplies motor innervation. It supplies the muscles derived from the mesoderm of the first pharyngeal arch, including the four muscles of mastication (temporalis, masseter, lateral pterygoid, and medial ptery- goid) and four additional muscles: Mylohoid, Anterior belly of the Digastric, Tensor Tympani, and Tensor Veli Palatini (mnemonic = "MATT"). These muscles play no role in the dysarthria noted in this patient, and CN V3 is not located in close proximity to the stab wound. Choice D (Lingual branch of glossopharyngeal nerve) is incorrect. The lingual branch of the glossopharyngeal nerve (CN IX) enters the posterior one third of the tongue to provide general sensation and taste to the region. It does not provide any motor innervation, so this nerve would not be involved with the dysarthria noted in this patient. Moreover, due to the location of the knife wound, this nerve would not be affected. Choice E (Roots of the brachial plexus) is incorrect. The roots of the brachial plexus represent the anterior (ventral) rami of C5-8 and T1, and these nerves emerge between the anterior and middle scalene muscles in the lateral cervical region (posterior triangle of neck). The roots of the brachial plexus are located too inferior to be damaged by the knife.

A 32-year-old woman undergoes LASIK refractive surgery to improve her visual acuity and rid herself of eyeglasses. To combat dry eyes, a common complication of this surgery, her ophthalmologist inserts a silicone plug, to block drainage of tears from the patient's left eye. Obstruction of what structure prevents tears from entering the lacrimal apparatus? (A) Lacrimal punctum (B) Lacrimal canaliculus (C) Lacrimal sac (D) Nasolacrimal duct (E) Inferior nasal meatus

The answer is A: Lacrimal punctum. The lacrimal apparatus is composed of the lacrimal gland, lacrimal canaliculi, lacrimal sac, and nasolacrimal duct. The lacrimal gland, located in the upper lateral corner of the orbit, secretes tears into the supe- rolateral conjunctival fornix. Tears are spread across the eye by the blinking actions of the eyelids and accumulate in the medial corner (canthus) of the eye. They drain into the paired lacrimal canaliculi by passing through the lacrimal puncta (pores). This photo shows a silicone plug present in the infe- rior punctum of the patient's left eye. If permitted to enter the punctum, tears pass through the lacrimal canaliculus, the lac- rimal sac, through the nasolacrimal duct, and enter the nasal cavity via inferior nasal meatus. Obstruction of a punctum with a silicone plug effectively closes the doorway into a canalicu- lus and prevents normal tear drainage. This plug placement would keep tears within the eye to lubricate and protect the cornea, which is altered in LASIK surgery to improve visual acuity. Choice B (Lacrimal canaliculus) is incorrect. The lacri- mal canaliculi are paired canals into which the puncta drain in the margin of the eyelids, at their medial corners. The lacrimal punctum is obstructed with the silicone plug, so tears would not enter the lacrimal canaliculus. Choice C (Lacrimal sac) is incorrect. The lacrimal sac is the slightly dilated upper end of the nasolacrimal duct. It collects tears from the canaliculi. Because the lacrimal punctum is obstructed with the silicone plug, tears would not reach the lacrimal canaliculus or the lacrimal sac. Choice D (Nasolacrimal duct) is incorrect. The nasolacrimal duct extends from the lacrimal sac to its termina- tion on the lateral nasal wall, which opens into the inferior meatus. Because the lacrimal punctum is obstructed with the silicone plug, tears would not reach the nasolacrimal duct of the lacrimal apparatus. Choice E (Inferior nasal meatus) is incorrect. The inferior nasal meatus receives the drain- age of the lacrimal apparatus. However, due to placement of the silicone plug, tears would not drain into the nasal cavity. Remember that the inferior nasal meatus does not contain the drainage points of any of the four paranasal sinuses, unlike the other nasal meati.

A 54-year-old man comes to his doctor complaining of an inability to open his left eye. When he physically pries open his affected eye with his fingers, the gaze of his left eye is directed inferiorly and laterally, causing diplopia. His left pupil is also dilated in comparison to the right one. What nerve is most likely affected in this patient? (A) Left oculomotor nerve (B) Right oculomotor nerve (C) Left abducent nerve (D) Right abducent nerve (E) Left trochlear nerve

The answer is A: Left oculomotor nerve. The left oculomotor nerve, CN III, innervates four (of six) extraocular eye muscles and the levator palpebrae superioris in the ipsilateral (left) eye. Damage to the left CN III leads to ptosis (or drooping) of the left upper eyelid due to loss of innervation to the leva- tor palpebrae superioris muscle, which is why the patient is having trouble opening his left eye without assistance from his fingers. His left eye is also located in an abducted and lat- eral position due to loss of innervation to the four extraocular muscles. Due to the unopposed muscular contractions of the lateral rectus muscle, innervated by the abducent nerve (CN VI), and the superior oblique muscle, innervated by the tro- chlear nerve (CN IV), the eye resides in this abnormal "down and out" position causing diplopia (or double vision). Finally, the oculomotor nerve also has a parasympathetic component that enables accommodation of the eye, which involves pupil- lary constriction. Damage to the parasympathetic component of CN III causes pupillary dilation due to the unopposed sym- pathetic innervation to the dilator pupillae muscle of the iris. All of these signs and symptoms are evident in the left eye of this patient, which implies damage to the left CN III. Choice B (Right oculomotor nerve) is incorrect. Damage to the right CN III leads to diplopia (due to the right pupil resting in an abducted and lateral position), ptosis of the upper eyelid, and pupillary dilation, which is noted in this patient. However, the left eye is affected in this patient, which implies involve- ment of the left CN III. Choice C (Left abducent nerve) is incorrect. The only extraocular muscle innervated by the left abducent nerve, CN VI, is the left lateral rectus muscle, which turns the left globe laterally. If this nerve were damaged, the patient would display diplopia, and the left pupil would be resting in the adducted position due to the unopposed action of the other extraocular muscles. Choice D (Right abducent nerve) is incorrect. The only extraocular muscle innervated by the right abducent nerve, CN VI, is the right lateral rectus muscle, which turns the right globe laterally. If this nerve were damaged, the patient would display diplopia, and the right pupil would be resting in the adducted position due to the unopposed action of the other extraocular muscles. Because extraocular eye movements were affected in the left eye of this patient, this option can be easily eliminated. Choice E (Left trochlear nerve) is incorrect. The left trochlear nerve inner- vates only one muscle, the superior oblique muscle, in the orbit. This muscle pulls the eye inferolaterally; however, this nerve is tested clinically by asking the patient to look inferi- orly after the left eye is placed in an adducted position. While a patient with left trochlear nerve damage would experience diplopia, especially when looking inferiorly, this answer does not explain the ptosis and pupillary dilation seen in this patient.

An 82-year-old woman develops a dural meningioma (tumor) that compresses the confluence of the dural venous sinuses. On a contrast venogram from an angiographic series, drainage from which of the following vessels would be impeded by the tumor? (A) Superior sagittal sinus (B) Transverse sinus (C) Cavernous sinus (D) Superior petrosal sinus (E) Inferior petrosal sinus

The answer is A: Superior sagittal sinus. The dural venous sinuses (dural sinuses) are valveless venous channels formed between the layers of the dura mater. All the blood from the brain drains into the dural sinuses and then out of the cra- nium via true veins. Most of the blood from the brain drains through the dural sinuses to the internal jugular veins. The cerebral hemispheres drain through various dural sinuses to the confluence of dural venous sinuses or confluens sinuum, a common collection pool at the junction of these three dural septa: falx cerebri, falx cerebelli, and tentorium cerebelli. Blood within the confluence of sinuses, located in close prox- imity to the internal occipital protuberance, flows into the transverse sinuses to the sigmoid sinuses to the internal jugu- lar veins. The superior sagittal sinus is a large venous channel located along the attached border of the falx cerebri. It drains the more superficial cerebral veins directly to the confluence of sinuses. Obstruction of the confluence of sinuses prevents proper drainage through the superior sagittal sinus, resulting in back pressure buildup in that channel. Choice B (Trans- verse sinus) is incorrect. The transverse sinuses are large venous channels within the attached border of the tentorium cerebelli. They drain blood from the confluence of sinuses into the sigmoid sinuses. Because the transverse sinus is distal to the confluence of sinuses, compression at this location would not hinder drainage through the transverse sinus; however, it will reduce the amount of blood draining through the trans- verse sinuses. Choice C (Cavernous sinus) is incorrect. The basal veins from the brainstem drain primarily to the cavern- ous sinuses, located on either side of the sella turcica. The cavernous sinuses drain to the internal jugular veins via the superior and inferior petrosal sinuses. It should be mentioned that multiple important alternative connections between the cavernous sinuses and other pericranial veins can exist. Com- pression of the confluence of sinuses would not directly affect blood draining from the cavernous sinuses because their con- nections to the internal jugular veins are well distal to the con- fluence of the sinuses. Choice D (Superior petrosal sinus) is incorrect. The superior petrosal sinus lies along the crest of the petrous part of the temporal bone and carries blood from the cavernous sinus into the distal part of the transverse sinus. Normal flow through this channel would not be compromised because it is located distal to the compressed confluence of the sinuses. Choice E (Inferior petrosal sinus) is incorrect. The inferior petrosal sinus runs in close proximity to the junction of the clivus of the sphenoid bone and the petrous temporal bone. It carries blood from the cavernous sinus to the distal end of the sigmoid sinus or directly into the origin of the inter- nal jugular vein. This drainage path is located well distal to the confluence of the sinuses and would not be obstructed by this tumor.

A professional student finds out she has a perfect score on the anatomy portion of her board examination and her muscles of facial expression produce a long anticipated smile. What muscle is assisting her in elevating her labial commissure bilaterally to smile? (A) Zygomaticus major (B) Zygomaticus minor (C) Levator labii superioris (D) Buccinator (E) Orbicularis oris

The answer is A: Zygomaticus major. The zygomaticus major functions as a dilator of the oral fissure by elevating the labial commissures (corners of the mouth), in order to smile when the muscles contract bilaterally. The contraction of the zygomati- cus major muscle bilaterally would be the proper facial expres- sion after scoring a perfect score on the anatomy portion of her board examination. Interestingly, this muscle can also produce a sneer when this muscle contracts unilaterally to show disdain. Study hard for your examination, so you can make this muscle contract bilaterally. The zygomaticus major muscle originates on the lateral aspect of the zygomatic bone to insert into the labial commissures. It is a muscle of facial expression that is innervated by the facial nerve (CN VII), which innervates all of muscles of facial expression derived from the mesoderm of the second pharyngeal (branchial) arch. Choice B (Zygomaticus minor) is incorrect. The zygomaticus minor is also a muscle of facial expression innervated by the facial nerve (CN VII). It functions as a dilator of the oral fissure by retracting (elevat- ing) the upper lip to show the upper teeth, which also deep- ens the nasolabial sulcus. It originates from the orbicular oris muscle and the zygomatic bone of the lateral face to insert into the upper lip. It has similar functions as the levator labii supe- rioris muscle. It would not enable this professional student to smile after seeing her examination scores because it does not elevate the labial commissures specifically. Choice C (Levator labii superioris) is incorrect. The levator labii superioris is also a muscle of facial expression innervated by the facial nerve (CN VII). It functions as a dilator of the oral fissure by retracting (ele- vating) the upper lip to show the upper teeth, which also and deepens the nasolabial sulcus. It originates on the infraorbital margin of the maxilla, above, and therefore covers, the infraor- bital foramen. It would not enable this professional student to smile after seeing her examination scores because it does not elevate the labial commissures specifically. Choice D (Buccina- tor) is incorrect. The buccinator is also a muscle of facial expres- sion innervated by the facial nerve (CN VII). It originates on the alveolar ridges of the maxillary and mandibular molar teeth and contracts to gives tension to the cheek to keep food between the occlusal surfaces of the teeth. The tone of the buccinator muscle provides resistance to keep teeth from tilting laterally and pre- vents patients from looking like a hamster, with food lodged in the oral vestibule, when they chew food. It would not enable this professional student to smile after seeing her examination scores. Choice E (Orbicularis oris) is incorrect. The orbicularis oris is also a muscle of facial expression innervated by the facial nerve (CN VII). It encircles the mouth to act as the important sphincter of the oral fissure, and in performing this task, it functions to close the oral fissure as when protruding the lips to kiss, whistle, or suck. It also resists distension as when blowing into a brass instrument, like a trumpet. The orbicularis oris muscle originates on the incisive fossae of the mandible and maxilla and attaches to the modiolus, a convergence of several muscles of facial expres- sion at the corners of mouth. It would not enable this professional student to smile after seeing her examination scores.

A 10-year-old girl presents with a smooth, round neck mass the size of a golf ball at the upper third of the anterior border of her right sternocleidomastoid muscle. Her mother says this once small, peanut-sized mass grew without pain or inflammation after it became noticeable 2 months previously. The mass does not affect the girl's daily activities. What is the most likely diagnosis for the pictured mass? (A) Thyroglossal cyst (B) Branchial cyst (C) Undescended thymus (D) Preauricular cyst (E) Ectopic palatine tonsil

The answer is B: Branchial cyst. The second pharyngeal (branchial) arch normally overgrows the more caudal pharyn- geal arches and merges with the epicardial ridge in the lower part of the developing neck region. This development causes the second, third, and fourth pharyngeal clefts (grooves) to lose contact with the surface of the neck and coalesce into a common cervical sinus. The cervical sinus is usually obliter- ated early in development. However, if the second arch does not properly grow caudally, remnants of the second, third, and/or fourth clefts may persist as a branchial cyst (lateral cer- vical cyst; cervical lymphoepithelial cyst) along the lateral side of the neck. This type of cyst may or may not be connected to the surface by a small drainage canal (branchial fistula). Thus, branchial cysts are the remnants of the cervical sinus and its duct. They are located along the anterior border of the SCM. Most often, these cysts are the remnants of the second pharyngeal cleft, located just below the angle of the mandible. Second branchial cleft cysts represent approximately 67% to 93% of all pharyngeal apparatus anomalies. However, bran- chial cysts may be found anywhere along the anterior margin of the SCM. Very frequently, branchial cysts are inconspicuous at birth, becoming evident as they enlarge throughout child- hood. Choice A (Thyroglossal cyst) is incorrect. The thyroid gland first appears as a single, ventromedian diverticulum (thyroid diverticulum) off the floor of the embryonic pharynx, between the tuberculum impar and copula of the incipient tongue. It descends along the midline, anterior to the gut tube, remaining connected to the tongue by a narrow canal (the thy- roglossal duct). The thyroglossal duct usually solidifies and is obliterated after the final descent of the thyroid gland into its normal terminal position. A thyroglossal cyst is a cystic rem- nant of the thyroglossal duct. It is always located in or close to the midline of the neck. Most commonly (∼50%), it is found near the body of the hyoid bone. However, a thyroglossal cyst may lie anywhere along the normal migratory route of the thyroid gland. Choice C (Undescended thymus) is incorrect. The thymus gland begins as epithelial primordia in the ven- tral wings of both the paired third pharyngeal (branchial) pouches. Losing their connections with the pharyngeal walls, these epithelial primordia descend caudally and medially and ultimately migrate into the upper anterior thorax where they fuse with each other. Sometimes, the tail portion of one or both of the descending primordia may persist in the neck. In such cases, thymic tissue may be embedded in the thyroid gland or may be found as isolated thymic pockets near the midline. Partial or complete absence of the thymus gland is a third pharyngeal pouch component of DiGeorge syndrome, a complex collection of craniofacial and cardiovascular anom- alies. Choice D (Preauricular cyst) is incorrect. The auricle (pinna) of the external ear develops from multiple prolifera- tions (auricular hillocks) in the dorsal ends of the first and sec- ond pharyngeal (branchial) arches, surrounding the opening of the first pharyngeal cleft. Its initial position is in the incipi- ent neck region, with development of the neck and mandible causing the auricle to ascend to its final position at the side of the head. Development of the auricle is complicated, and congenital abnormalities of the auricle (preauricular cysts and pits, auricular sinuses, skin tags) are common. Importantly, auricular malformations are often associated with other con- genital disorders. Choice E (Ectopic palatine tonsil) is incor- rect. The fossa, epithelium, and crypts of the palatine tonsils are derived from the second pharyngeal (branchial) pouches and are related to the developing oropharyngeal region. The lymphoid tissue of the tonsil is a secondary infiltration of that bed. Ectopic lymphoid tissue may be found in small nodules near the tonsillar fossa.

A 14-year-old girl presents with Horner syndrome, after surgi- cal removal of a mass growing in her posterior mediastinum. What nerve would be affected by the loss of sympathetic innervation to the head? (A) Lesser petrosal (B) Deep petrosal (C) Greater petrosal (D) Chorda tympani (E) Optic

The answer is B: Deep petrosal. The deep petrosal nerve is a sympathetic bundle that branches off the internal carotid plexus at the exterior of the base of the skull. It joins with the greater petrosal nerve to form the nerve of the pterygoid (vidian) canal. This nerve enters the pterygopalatine fossa where it joins the pterygopalatine ganglion. The postsynaptic sympathetic fibers, derived from the deep petrosal nerve, pass through the pterygopalatine ganglion without synapsing and supply the mucosal glands of the nasal cavity as well as the smooth muscle controlling vasomotion (vasoconstriction and vasodilation) of the arteries supplying the nasal cavity. Disruption of sympathetic fibers in the sympathetic chain causing Horner syndrome would greatly affect the deep petrosal nerve. Choice A (Lesser petrosal) is incorrect. The lesser petrosal nerve carries presynaptic parasympathetic nerve fibers derived from the glossopharyngeal nerve. It runs from the tympanic plexus in the middle ear to the otic ganglion in the infratemporal fossa, where it synapses. Then, the postsynaptic parasympathetic fibers follow the auriculotemporal nerve (a branch of the mandibular division of the trigeminal nerve) into the parotid gland, where they provide the secretomotor control for this salivary gland. Loss of sympathetic innervation to the head (Horner syndrome) would not affect this nerve. Choice C (Greater petrosal) is incorrect. The greater petrosal nerve branches off the facial nerve at the geniculum of the facial canal and carries presynaptic parasympathetic innervation. It coalesces with the deep petrosal nerve to form the nerve of the pterygoid (vidian) canal and synapses within the pterygopalatine ganglion. The postsynaptic parasympathetic fibers follow branches of the maxillary nerve to supply the emotional tearing to the lacrimal gland as well as other glands in the nasal cavity and palate. Loss of sympathetic innervation to the head (Horner syndrome) would not affect this nerve. Choice D (Chorda tympani) is incorrect. The chorda tympani nerve, a branch of the facial nerve, carries taste and presynaptic para- sympathetic fibers to the tongue and oral floor. It originates from the distal part of the facial nerve, runs across the middle ear cavity, enters the infratemporal fossa, and joins the lingual nerve (a branch of the mandibular division of the trigeminal nerve). Ultimately, the taste fibers distribute into the anterior two thirds of the tongue. The parasympathetic fibers synapse in the submandibular ganglion, with the postsynaptic fibers passing to the submandibular and sublingual salivary glands and to mucus glands in the oral floor. Loss of sympathetic innervation to the head (Horner syndrome) would not affect this nerve. Choice E (Optic) is incorrect. The optic nerve is one of the few cranial nerves (along with the olfactory, vestibulocochlear, and spinal accessory nerves), which do not trans- port autonomic fibers at some point in its distribution. This nerve (really the optic tract) extends from the retina traversing the optic canal to reach the diencephalon of the brain.

Identify the structure indicated with the letter "X" in this X-ray of the lateral neck. (A) Vertebral body (B) Pedicle (C) Intervertebral disc space (D) Superior articular process (E) Inferior articular process

The answer is B: Pedicle. The image is a lateral plane film (X-ray) of the cervical component of the vertebral column. This type of radiological imaging is used commonly for gen- eral evaluation of the neck, especially in severe neck injuries where fractures may be suspected. The pedicle is the con- stricted "foot" of the vertebral arch that links the neural arch with the body of a vertebra, and it is identified by the white leader line from the letter "B" in the labeled X-ray provided. In this view, the opposite pedicles are superimposed upon one another. Thus, these small structures produce a distinct radiodensity (radiopacity) in the film. Choice A (Vertebral body) is incorrect. The body is the large, block-like structure at the anterior aspect of the column, and it is identified by the white leader line from the letter "A" in the labeled X-ray provided. The margins of the cervical bodies typically present a smooth curvature on both the anterior and posterior aspects. Deviations of the normal curvature of the vertebral bodies may suggest a possible fracture and/or torn ligaments. Choice C (Intervertebral disc space) is incorrect. This radiolucent area, identified by the white leader line from the letter "C" in the labeled X-ray provided, is located between the vertebral bod- ies and is occupied by the intervertebral disc. The height of this space decreases with degeneration of the disc (from either pathology or aging), thus approximating the vertebral bodies. Choice D (Superior articular process) is incorrect. Because the articular processes in the cervical region are positioned at oblique inclinations, they appear as seemingly sharply pointed structures in the lateral view, which can be misin- terpreted as displaced bone fragments. The superior articular process, identified by the white leader line from the letter "D" in the labeled X-ray provided, is an upwardly projecting ele- ment, and it articulates with the inferior articular process of the more superior vertebra to create a zygapophysial (or facet) joint between the vertebral arches. Choice E (Inferior articu- lar process) is incorrect. The inferior articular process, identi- fied by the white leader line from the letter "E" in the labeled X-ray, can be seen projecting inferior in the lateral view. The inferior articular process articulates with the superior articular process of the more inferior vertebra to create a zygapophy- sial (or facet) joint between the vertebral arches. The articular processes are outgrowths of the vertebral arch at the junction of the pedicle and the lamina.

Startled by a loud noise while shaving his head with a straight razor, a young man accidentally cuts his scalp, severing branches of the supraorbital and superficial temporal vessels. The scalp wound appears modest but bleeds profusely. In what layer of the scalp do the severed vessels reside? (A) Skin (B) Dense connective tissue (C) Epicranial aponeurosis (D) Loose connective tissue (E) Pericranium

The answer is B: Dense connective tissue. The second layer of the scalp is composed of dense fibrous connective tissue, and this layer houses the main networks of nerves and vessels that enter the scalp from its periphery. The walls of the arteries within the scalp are tightly attached to the surrounding con- nective tissue bed. As a result, scalp lesions that penetrate the second layer tend to bleed profusely because the fibrous tissue holds apart the cut ends of the severed vessels. Conveniently, the scalp can be divided into five structural/functional layers, which create a mnemonic "SCALP" when moving from super- ficial to deep zones. Layer 1 (most superficial) is Skin. Layer 2 is the (dense) Connective tissue highlighted in this question. Layer 3 is Aponeurosis of the epicranius muscle (or the epicra- nial aponeurosis). Layer 4 is Loose (areolar) connective tissue that enables free movement of the first three layers of the scalp over the calvaria. Layer 5 is Pericranium (external periosteum), which is a dense layer of connective tissue fused to the outer surfaces of the neurocranium. The mnemonic "SCALP" can enable a student to recall the five layers of the scalp. Choice A (Skin) is incorrect. Skin is the first layer of the scalp, and it is the typically hair-bearing epidermis rich in sebaceous glands. Choice C (Epicranial aponeurosis) is incorrect. The epicra- nius (occipitofrontalis) muscle is formed by the frontalis and occipitalis muscles and the expansive aponeurosis that binds them together. This unit forms the third layer of the scalp. It is responsible for movement of the scalp by the actions of its muscular ends. Deep scalp wounds through the epicranius (especially the aponeurotic part in the frontal plane) tend to gape widely because the frontalis and occipitalis muscles pull in opposite directions and widen the lesion. Choice D (Loose connective tissue) is incorrect. Loose (areolar) connective tissue is the fourth layer of the scalp, and it forms the sub- aponeurotic space of the scalp, which becomes the plane of movement enabling the first three layers of the scalp to slide as a unit relative to the underlying pericranium. This loose con- nective tissue is also considered the danger layer of the scalp because infectious matter (blood, pus) can spread easily and widely through this fourth layer of the scalp. Also, infections can move from this layer through emissary (epiploic) veins into the intracranial dural venous sinuses. Choice E (Pericra- nium) is incorrect. The deepest (fifth) layer of the scalp is the pericranium (or external periosteum), which is a dense layer of connective tissue fused to the outer surfaces of the bones of the cranial vault.

A 59-year-old man went to his doctor unable to close his right eye. The physical examination also revealed asymmetry in his smile and an inability to wrinkle the right side of his forehead. What cranial nerve is affected in this patient? (A) Trigeminal nerve (B) Facial nerve (C) Glossopharyngeal nerve (D) Vagus nerve (E) Hypoglossal nerve

The answer is B: Facial nerve. The main trunk of the facial nerve (CN VII) exits the stylomastoid foramen and runs ante- riorly in close relationship to (or within) the parotid gland. This nerve branches into five terminal branches: Tempo- ral, Zygomatic, Buccal, Marginal mandibular, and Cervical. These branches innervate the muscles of facial expression derived from the mesoderm of the second pharyngeal arch and can be remembered by the mnemonics "Two Zebras Bit My Cheek" or "To Zanzibar By Motor Car". Damage to the main trunk of the facial nerve causes unilateral facial paraly- sis due to loss of innervation to these muscles. The patient would present ipsilaterally with an inability to close the eye (due to the loss of the sphincter of the eye, the orbicularis oculi muscle), inability to wrinkle the forehead (due to loss of the frontalis muscle), and asymmetry in the smile (due to the loss of many muscles acting on the labial commissure and upper lip). Unilateral facial paralysis is seen following damage to the facial nerve following a nerve lesion, brain tumor, stroke, or even Lyme disease. When no specific cause (idiopathic) can be found, it is called facial (Bell or CN VII) palsy. Choice A (Trigeminal nerve) is incorrect. The trigemi- nal nerve (CN V) is the main sensory nerve for the face and scalp, and it has three divisions, namely, the ophthalmic (CN V1), maxillary (CN V2), and mandibular (CN V3). CN V3 is the only division of the trigeminal nerve (CN V) that supplies motor innervation. It supplies the eight muscles derived from the mesoderm of the first pharyngeal arch, including the muscles of mastication. Losing innervation to the muscles supplied by CN V3 would not cause ipsilateral facial paralysis. Choice C (Glossopharyngeal nerve) is incorrect. The glossopharyngeal nerve (CN IX) supplies only one muscle, the stylopharyngeus, derived from the mesoderm of the third pharyngeal arch. Losing innervation to the stylopharyngeus, a muscle of the pharynx, would not cause ipsilateral facial paralysis. Choice D (Vagus nerve) is incorrect. The vagus nerve (CN X) supplies the muscles derived from the mesoderm of the fourth and sixth pharyngeal arches, including all the muscles of the larynx as well as most of the muscles of soft palate (exception = tensor veli palatini of CN V ) and pharynx (exception = stylopharyngeus of CN IX). Despite its extensive motor innervation, damage to the vagus nerve would not cause ipsilateral facial paralysis. However, it would result in dysphagia, or difficulty swallowing. Choice E (Hypoglossal nerve) is incorrect. The hypoglossal nerve (CN XII) innervates the intrinsic and (most of the) extrinsic muscles of tongue (exception = palatoglossus muscle of CN X). Losing innervation to the muscles of the tongue would not cause ipsilateral facial paralysis.

During a fight between two construction workers, one man strikes the other with a hammer at the pterion of the skull. Which of the following bones may be fractured? (A) Zygomatic process of the temporal bone (B) Greater wing of the sphenoid bone (C) Mastoid process of the temporal bone (D) Lateral pterygoid plate of the sphenoid bone (E) Coronoid process of the mandible

The answer is B: Greater wing of the sphenoid bone. The pterion is a significant craniometric landmark point in the temporal fossa on the lateral aspect of the skull. It is the roughly H-shaped junction of four bones: greater wing of the sphenoid, frontal, parietal, and squamous parts of the temporal. The pterion is clinically significant in that the bones here are relatively thin and susceptible to fracture from impact. Fur- thermore, the anterior branch of the middle meningeal artery typically lies tightly grooved against the interior of the skull at this point. The middle meningeal artery is the major vessel supplying the dura mater and the bones of the cranial vault. It can be readily ruptured in trauma to the pterion, and the middle meningeal artery is the primary vessel implicated in epidural hemorrhage. Choice A (Zygomatic process of the temporal bone) is incorrect. The zygomatic process of the temporal bone is an anterior projection articulating with the zygomatic bone. Together, these two components form the zygomatic arch. Choice C (Mastoid process of the temporal bone) is incorrect. The mastoid process of the temporal bone is a breast-like projection extending inferior from the tempo- ral bone behind the external acoustic meatus. It forms part of the posterior boundary of the infratemporal fossa. Addition- ally, it is the attachment site for certain neck muscles (e.g., sternocleidomastoid) and is largely hollowed by the mastoid air cells. Choice D (Lateral pterygoid plate of the sphenoid bone) is incorrect. The lateral pterygoid plate is a thin, wing- like, inferior extension of the sphenoid bone. It forms the medial (deep) boundary of the infratemporal fossa and pro- vides attachment for portions of the medial and lateral ptery- goid muscles. Choice E (Coronoid process of the mandible) is incorrect. The coronoid process of the mandible is a thin, blade-like, superior projection from the anterior aspect of the ramus of the mandible. It serves as the insertion area for the large temporalis muscle, a muscle of mastication.

A 47-year-old man has trouble with double vision (diplopia) after striking his head on the steering wheel in a car accident. During a subsequent eye examination, his ophthalmologist asks him to first look inward (toward his nose) and then upward (toward the ceiling). The integrity of which of the following extraocular muscles is being tested? (A) Superior oblique (B) Inferior oblique (C) Lateral rectus (D) Inferior rectus (E) Superior rectus

The answer is B: Inferior oblique. Paralysis of one or more of the extraocular muscles causes a lack of coordinated eye movements, often resulting in diplopia (double vision). Clini- cal evaluation of the muscles includes step-wise positioning of the eye in such a way as to test each individual muscle at its position of greatest mechanical efficiency (relative to the visual axis) to determine if that muscle is functioning prop- erly. Testing begins from the rest position, with the patient looking straight ahead. In this case, the first step (looking inward; adduction) tests the action of the medial rectus, the primary adductor of the eye. At the adducted position, the superior and inferior oblique muscles are aligned along their primary lines of action. The second step (looking upward; elevation) tests for function of the inferior oblique, which elevates the eye. Failure of either step indicates possible dam- age to the oculomotor nerve (CN III), which supplies both of these muscles. The inferior oblique and inferior rectus muscles can be impinged in head trauma cases when the floor of the orbit is fractured. These orbital (blowout) frac- tures lead to the downward herniation of orbital contents into the maxillary sinus, leading to the potential entrapment of these two extraocular muscles. The integrity of the infe- rior oblique muscle was tested in this question. Choice A (Superior oblique) is incorrect. Testing the superior oblique includes the same first step as for the inferior oblique, that is, adducting the eye to test the medial rectus and to position the oblique muscles along their primary lines of action. In this case, the second step is to look downward (depression) to test the superior oblique, which depresses the eye. Failure of this step indicates possible damage to the trochlear nerve (CN IV), which supplies only the superior oblique muscle. Choice C (Lateral rectus) is incorrect. The lateral rectus is the primary abductor of the eye from the rest position. Asking the patient to look outward (away from the nose) tests the ability of the lateral rectus to abduct at its position of greatest efficiency. Failure of this test indicates possible damage to the abducent nerve (CN VI), which supplies only the lateral rectus. Fur- thermore, at the adducted position, the superior and inferior rectus muscles are aligned along their primary lines of action and are set in best testing position. Choice D (Inferior rectus) is incorrect. At the abducted position, asking the patient to look downward tests the function of the inferior rectus, which depresses the eye. Failure to accomplish this task indicates possible damage to the oculomotor nerve, which supplies the inferior rectus muscle. Choice E (Superior rectus) is incor- rect. From the abducted position, asking the patient to look upward tests the superior rectus, which elevates the eye. Fail- ure to accomplish this task is another indicator of possible damage to the oculomotor nerve, which supplies the superior rectus muscle.

A 9-year-old girl with a history of strep throat has her palatine tonsils surgically removed. She returns to the hospital 3 days later with a high fever and chest pain. A physician orders a CT scan, which revealed spread of infection into the superior mediastinum. What is the most likely route for this infection to descend through the neck to reach the superior mediastinum? (A) Parapharyngeal space (B) Retropharyngeal space (C) Buccal space (D) Carotid sheath (E) Suprasternal space

The answer is B: Retropharyngeal space. The retropharyngeal space is the most frequent route for infection to spread through the neck into the superior mediastinum. This potential space exists between the prevertebral fascia and the anterior lamina of the prevertebral fascia, and it extends from the base of the skull to the superior mediastinum to the level of the third tho- racic vertebra. The retropharyngeal space allows movement of the esophagus, pharynx, larynx, and trachea relative to the vertebral column, but it is clinically important due to its poten- tial to provide a conduit for the spread of infection into the mediastinum. Choice A (Parapharyngeal space) is incorrect. The parapharyngeal space is located between the lateral wall of the upper pharynx, the medial pterygoid muscle, and the cervical vertebrae. An infection residing in this potential space is unable to reach the superior mediastinum, unless it com- municates directly with the retropharyngeal space. Choice C (Buccal space) is incorrect. The buccal space exists between the deep surface of the parotid gland and the mucosa of the cheek. An infection residing in this potential space is unable to reach the superior mediastinum. Choice D (Carotid sheath) is incorrect. The carotid sheath is a fascial investment that extends from the base of the skull to the root of the neck, and it does communicate with the mediastinum of the thorax. So, this fascial space does offer a potential pathway for the spread of infection into the mediastinum; however, based upon the source of the infection in the tonsillar fossa, the retropha- ryngeal space is the best answer to this question. Choice E (Suprasternal space) is incorrect. The suprasternal space is located above the manubrium of the sternum, and it is a nar- row interval located between a split in the investing layer of the deep cervical fascia. This space would allow a communica- tion between the anterior jugular veins to pass, but it would not allow an infection to spread through the neck and into the mediastinum.

A 45-year-old man goes to his physician complaining of having trouble with his speech (dysarthria) and involuntary contractions (fasciculations) within his tongue muscles. The doctor notes that his tongue deviates to the right when the patient attempts to protrude his tongue. What nerve is most likely damaged in this patient? (A) Left hypoglossal nerve (B) Right hypoglossal nerve (C) Left glossopharyngeal nerve (D) Left vagus nerve (E) Right vagus nerve

The answer is B: Right hypoglossal nerve. The hypoglossal nerve (CN XII) innervates all of the intrinsic muscles of the tongue and most of its extrinsic muscles with the lone exception being the palatoglossus muscle, innervated by the vagus nerve (CN X). Damage to the right CN XII produces the dysarthria and fasciculations within the tongue musculature, noted in the patient. The ipsilateral deviation of the tongue is due to the unopposed muscular contractions of the left (contralateral) genioglossus muscle. The mnemonic "The tongue licks the wound" will help you remember that the tongue deviates to the ipsilateral side in a lower motor lesion of CN XII. Choice A (Left hypoglossal nerve) is incorrect. The hypoglossal nerve (CN XII) innervates all of the intrinsic muscles of the tongue and most of its extrinsic muscles with the lone exception being the palatoglossus muscle, innervated by the vagus nerve (CN X). Therefore, the cutting of the left CN XII produces dysarthria and fasciculations, noted in this patient; however, the tongue would deviate to the left side during protrusion due to the unopposed muscular contractions of the right genioglossus muscle. Choice C (Left glossopharyngeal nerve) is incorrect. The glossopharyngeal nerve only innervates one muscle, the stylopharyngeus, which is a muscle of the pharynx. Without innervating a muscle of the tongue, the glossopharyngeal nerve is unable to affect the deviation of the tongue. Choice D (Left vagus nerve) is incorrect. Damage to the left vagus nerve causes asymmetry in soft palate elevation and contralateral deviation of the uvula to the right. These deficits result because pharyngeal branches of the vagus nerve innervate all of the musculature of the soft palate, except the tensor veli palatini (tensor of the soft palate), which is innervated by the mandibular division of the trigeminal nerve (CN V3). Within the tongue, the vagus nerve supplies only the palatoglossus muscle, which elevates the posterior aspect of the tongue during swallowing. Therefore, a lesion of the left vagus nerve would not affect tongue protrusion or result in ipsilateral tongue deviation. Choice E (Right vagus nerve) is incorrect. Damage to the right vagus nerve causes asymmetry in soft palate elevation and contralateral deviation of the uvula to the left. The pharyngeal branches of the vagus nerve innervate all of the musculature of the soft palate, except the tensor veli palatini (tensor of the soft palate), which is innervated by the mandibular division of the trigeminal nerve (CN V3). Within the tongue, the vagus nerve supplies only the palatoglossus muscle, which elevates the posterior aspect of the tongue during swallowing. Therefore, a lesion on the right vagus nerve would not affect tongue protrusion or result in ipsilateral tongue deviation.

You are shown a superior view of the head of a baby boy with scaphocephaly. Which of the following sutures closed prematurely in this infant? (A) Coronal (B) Sagittal (C) Lambdoid (D) Metopic (E) Squamous

The answer is B: Sagittal. Premature closure of the sagit- tal suture leads to scaphocephaly (G: boat-shaped head), or dolichocephaly, which presents with a disproportionately long and narrow skull. This marked increase in head length with narrowed width is particularly common in infants who are born prematurely. Surgical intervention is needed to treat sca- phocephaly to remove bone from the sagittal suture. Cranio- synostosis is the term that refers generally to the premature fusion of the cranial sutures. Choice A (Coronal) is incor- rect. Premature closure of the coronal suture leads to brachy- cephaly (G: short head), which involves a disproportionately wide skull with a short occipitofrontal diameter. Babies with brachycephaly have a characteristic square-shaped skull due to the short occipitofrontal diameter seen following premature closure of the coronal suture. Brachycephaly is more common in females, and surgical intervention can be implemented to remove bone from the coronal suture. If only one side of the coronal suture closes prematurely, the infant presents with an asymmetric cranium, a condition known as plagioceph- aly. In this baby boy, the long and narrow shape of the skull indicates premature closure of the sagittal suture. Choice C (Lambdoid) is incorrect. The lambdoid suture is a dense, fibrous connective tissue joint located on the back of the skull that connects the occipital bone with the posterior aspect of the parietal bone and petrous portion of the temporal bone. If only one side of the lambdoid suture closes prematurely, the infant presents with a twisted and asymmetric cranium, a condition known as plagiocephaly. In this baby boy, the long and narrow shape of the skull indicates premature closure of the sagittal suture. Choice D (Metopic) is incorrect. The metopic (or frontal) suture is a dense, fibrous connective tis- sue joint located between the two halves of the frontal bone. This suture usually begins fusion of the frontal bones at age 2 and disappears by age 6. If the metopic suture is not present at birth, trigonacephaly, a type of craniosynostosis, results in a keel-shaped deformity. In this baby boy, the long and narrow shape of the skull indicates premature closure of the sagit- tal suture. Choice E (Squamous) is incorrect. The squamous suture is a dense, fibrous connective tissue joint, which arches between the parietal bone and the squamous portion of the temporal bone, reaching anterior to the pterion. This suture is usually the last cranial suture to close around ages 35 to 39. In this baby boy, the long and narrow shape of the skull indicates premature closure of the sagittal suture.

A 24-year-old man came to his physician with a history of chronic maxillary sinusitis. A computed tomography (CT) scan reveals a soft-tissue mass in the superior aspect (or roof) of the right maxillary sinus. Functional endoscopic sinus surgery (FESS) was performed to biopsy the mass. Postoperatively, the patient experiences paresthesia and numbness of the skin of the right inferior eyelid and upper lip. Which nerve was most likely damaged during the surgery? (A) First division of trigeminal nerve (B) Second division of trigeminal nerve (C) Third division of trigeminal nerve (D) Zygomatic branch of facial nerve (E) Buccal branch of facial nerve

The answer is B: Second division of trigeminal nerve. The second (maxillary) division of the trigeminal nerve (CN V2) supplies the skin of the inferior eyelid and upper lip through the infraorbital nerve that courses through the superior aspect (roof) of the maxillary sinus, and due to its location, this nerve is most likely damaged during the biopsy. Damage to the infraorbital nerve causes paresthesia and numbness in the areas of cutaneous (sensory) distribution for this nerve. Choice A (First division of trigeminal nerve) is incorrect. The first (oph- thalmic) division of the trigeminal nerve (CN V1) supplies the skin of the upper eyelid, forehead, and scalp. CN V1 enters the middle cranial fossa via the superior orbital fissure, and its branches are distributed within the orbit, anterior cranial fossa, and nasal cavity. CN V1 does not supply sensory innervation to the maxillary sinus, so it would be spared during this FESS. However, the ophthalmic division of the trigeminal nerve does innervate three other paranasal sinuses (ethmoidal, frontal, and sphenoidal). Choice C (Third division of trigeminal nerve) is incorrect. The third (mandibular) division of the trigeminal nerve (CN V3) supplies the skin of the lower lip, chin, cheek, anterior auricle, and aspects of the lateral scalp. CN V3 leaves the middle cranial fossa via the foramen ovale, and it does NOT have a significant branch, which courses through the superior aspect (roof) of the maxillary sinus. Therefore, it could not be damaged in this patient. Choice D (Zygomatic branch of facial nerve) is incorrect. The zygomatic branch of the facial nerve (CN VII) is a terminal branch of the main trunk of CN VII. This nerve has only motor innervation, supplying the inferior part of the orbicularis oculi and other muscles of facial expression located below the orbit. Because it does not pass through the maxillary sinus or supply cutaneous innervation to any region, the zygomatic nerve of CN VII could not cause the numbness and paresthesia seen in this patient. Choice E (Buccal branch of facial nerve) is incorrect. The buccal branch of the facial nerve (CN VII) is a terminal branch of the main trunk of CN VII. This nerve has only motor innervation, sup- plying the risorius and muscles of the upper lip. Because it does not pass through the maxillary sinus or supply cutaneous innervation to any region, the buccal nerve of CN VII could not cause the numbness and paresthesia seen in this patient.

A 25-year-old woman notes the pictured asymmetry in her neck when she tenses the skin of her inferior face and neck. This asymmetry may be due to a limited mesodermal migration in which of the following embryonic structures? (A) First pharyngeal arch (B) Second pharyngeal arch (C) Third pharyngeal arch (D) Fourth pharyngeal arch (E) Fifth pharyngeal arch

The answer is B: Second pharyngeal arch. The asymmetry in the neck is due to an absence or underdevelopment of the platysma muscle on the patient's right side. The platysma is one of the facial muscles (or muscles of facial expression), even though it is located predominantly in the neck. The defining feature of the facial muscles is their origin or inser- tion into the subcutaneous tissue, which enables these mus- cles to convey facial expressions as well as alter the form of the facial orifices (orbits, nostrils, mouth, and external ears). All the facial muscles are derived from the mesoderm of the sec- ond pharyngeal (hyoid) arch and are innervated by the facial nerve (CN VII). Additional muscles derived from the second arch include the posterior belly of the digastric, stylohyoid, and stapedius. Choice A (First pharyngeal arch) is incorrect. The first pharyngeal (mandibular) arch folds on itself to form two parts: a dorsal maxillary process and a ventral mandibular process. This complex gives rise to the four muscles of mas- tication (temporalis, masseter, lateral pterygoid, and medial pterygoid) and four additional muscles: Mylohoid, Anterior belly of the Digastric, Tensor Veli Palatini, and Tensor Tym- pani (mnemonic = "MATT"). All are supplied by the mandibu- lar division of the trigeminal nerve (CN V3). Choice C (Third pharyngeal arch) is incorrect. The third pharyngeal arch gives rise to only a single skeletal muscle, the stylopharyngeus. This muscle is innervated by the glossopharyngeal nerve (CN IX). Choice D (Fourth pharyngeal arch) is incorrect. The fourth pharyngeal arch is the source of muscles of the soft palate (levator veli palatini, palatopharyngeus, muscle of uvula), pharynx (pharyngeal constrictors, salpingopharyngeus), tongue (palatoglossus), and larynx (cricothyroid). All are sup- plied by the superior laryngeal branch of the vagus nerve (CN X). Choice E (Fifth pharyngeal arch) is incorrect. The fifth pharyngeal arch is a rudimentary structure, along with the fifth pharyngeal pouch. These pharyngeal arch derivatives regress and disappear early in development. The absence of the fifth arch brings the sixth arch into such a close relationship with the fourth arch, and these pharyngeal arches are often consid- ered a combined 4-6 arch complex. The sixth arch gives rise to most of the intrinsic laryngeal muscles and the esophageal skeletal musculature. The link between the fourth and sixth arches is reinforced by the innervation of the sixth arch by the recurrent laryngeal branch of the vagus nerve. Thus, the vagus nerve supplies both the fourth and sixth arch musculature.

A bony overgrowth narrows the pterygomaxillary fissure, compressing the third part of the maxillary artery. As a result, blood flow will be reduced in which of the following arteries? (A) Superficial temporal (B) Sphenopalatine (C) Inferior alveolar (D) Middle meningeal (E) Ophthalmic

The answer is B: Sphenopalatine. The maxillary and superfi- cial temporal arteries are the terminal branches of the external carotid artery. The maxillary artery is the larger and deeper running of the two. It provides a wide distribution to the lat- eral and deep aspects of the head as it courses anterior across the infratemporal fossa and through the pterygomaxillary fissure into the pterygopalatine fossa. Thus, the pterygomax- illary fissure is the narrow opening from the deep, anterior part of the infratemporal fossa into the lateral aspect of the pterygopalatine fossa. The maxillary artery is divided into three parts by its relation to the lateral pterygoid muscle: first (retromandibular or prepterygoid) part, second (pterygoid) part, third (pterygopalatine; postpterygoid) part. The first part of the maxillary artery supplies the external ear, temporo- mandibular joint (TMJ), dura mater, calvaria, and lower jaw. The second part of this artery supplies muscles of mastica- tion and the cheek. The third part of the maxillary artery is the most difficult to envision because of its deep, distal loca- tion. It supplies the upper jaw, midface, palate, nasopharynx, and nasal passages. The sphenopalatine artery is one of the terminal branches off the third part of the maxillary artery (the other being the infraorbital artery). It arises within the pterygopalatine fossa; passes through the sphenopalatine fora- men; and supplies the nasal septum, lateral nasal wall, and adjacent paranasal sinuses. Choice A (Superficial temporal) is incorrect. The superficial temporal artery is the smaller, more superficially running of the terminal branches of the external carotid artery. It ascends anterior to the tragus of the auricle to supply the lateral scalp. Choice C (Inferior alveolar) is incor- rect. The inferior alveolar artery branches from the first part of the maxillary artery, in the infratemporal fossa. It travels with the inferior alveolar nerve to enter the mandible. It sup- plies the mandible, mandibular teeth and associated gingivae, and overlying skin. Choice D (Middle meningeal) is incorrect. This middle meningeal artery is a notable artery arising from the first part of the maxillary artery, just in front of the TMJ. It ascends through the foramen spinosum, enters the floor of the middle cranial fossa, and supplies the dura mater and calvaria. Choice E (Ophthalmic) is incorrect. The ophthalmic artery branches from the internal carotid artery within the cranium. It passes through the optic canal with the optic nerve supply the orbit and forehead.

A young child suffers a debilitating condition that includes progressive degeneration of the motor axons that innervate the masseter muscle. Which of the following muscles is most likely to exhibit the same fate? (A) Genioglossus (B) Tensor tympani (C) Orbicularis oris (D) Levator veli palatini (E) Stylopharyngeus

The answer is B: Tensor tympani. The masseter muscle is innervated by the masseteric branch of the mandibular division of the trigeminal nerve (CN V3). The mandibular divi- sion of the trigeminal supplies all the skeletal muscles derived from the first pharyngeal arch. Thus, axonal degeneration in the masseter may indicate more widespread neural prob- lems affecting any of the first arch muscles. These muscles, derived from the mesoderm of the first pharyngeal arch and innervated by the mandibular division of the trigeminal nerve (CN V3), include the four muscles of mastication (temporalis, masseter, lateral pterygoid, and medial pterygoid) and four additional muscles: Mylohoid, Anterior belly of the Digas- tric, Tensor Veli Palatini, and Tensor Tympani (mnemonic = "MATT"). The tensor tympani muscle attaches to the mal- leus, a middle ear ossicle, and acts to adjust the tension of the tympanic membrane (eardrum). The malleus is also derived from the first pharyngeal arch, along with the incus. Further, the tympanic membrane forms partly from the epithelium of the first pharyngeal pouch. The other muscle in the middle ear, the stapedius, is derived from the second pharyngeal arch, attaches onto the stapes (also formed from the second arch), and is supplied by the facial nerve. One way to remember the innervation relations of the tensor tympani and tensor veli palatini is to recall the 3T's: the Trigeminal supplies the Ten- sors, which have distinct Tendons. Choice A (Genioglossus) is incorrect. The genioglossus muscle is one of the extrinsic muscles of the tongue. All the extrinsic and intrinsic muscles of the tongue are supplied by the hypoglossal nerve (CN XII), with only one exception, the palatoglossus muscles, which are supplied by the vagus nerve (CN X). These muscles of the tongue are derived from occipital somites. Choice C (Orbicu- laris oris) is incorrect. The orbicularis oris is one of the facial muscles (or muscles of facial expression). All of these muscles are derived from the mesoderm of the second (hyoid) pha- ryngeal arch and are supplied by the facial nerve (CN VII). Choice D (Levator veli palatini) is incorrect. The levator veli palatini is related to both the soft palate and the pharyngotym- panic tube. It is derived from the fourth pharyngeal arch and is innervated by a pharyngeal branch of the vagus nerve (CN X). Choice E (Stylopharyngeus) is incorrect. The stylopha- ryngeus is a pharyngeal muscle. It is the sole skeletal muscle derived from the mesoderm of the third pharyngeal arch, and it is supplied by the glossopharyngeal nerve (CN IX).

In the provided X-ray, an opening in the skull is identified at the tip of the red arrow. If the nerve that traverses this cranial opening were damaged, what signs or symptoms would most likely be seen in the patient? (A) Unilateral muscles of facial expression paralysis (B) Unilateral muscles of mastication paralysis (C) Paresthesia of the upper lip, cheek, and lower eyelid (D) Decreased salivation of the submandibular gland (E) Decreased salivation of the parotid gland

The answer is B: Unilateral muscles of mastication paralysis. The foramen identified in the figure is the foramen ovale (oval foramen), which transmits the mandibular (third) divi- sion of the trigeminal nerve (CN V3). This nerve provides cutaneous (general) sensation to the skin of the lower lip, chin, cheek, anterior auricle, and lateral scalp. It also sup- plies motor innervation to the muscles derived from the mesoderm of the first pharyngeal arch, including the four muscles of mastication (temporalis, masseter, lateral ptery- goid, and medial pterygoid) and four additional muscles: Mylohoid, Anterior belly of the Digastric, Tensor Tympani, and Tensor Veli Palatini (mnemonic = "MATT"). Therefore, a lesion at the foramen ovale produces unilateral paralysis of the muscles of mastication. Choice A (Unilateral muscles of facial expression paralysis) is incorrect. The muscles of facial expression are supplied by the main trunk of the facial nerve, which leaves the cranium via the stylomastoid fora- men. This foramen is not identified in the X-ray, so unilateral paralysis of the muscles of facial expression is not likely. Choice C (Paresthesia of the upper lip, cheek, and lower eye- lid) is incorrect. The cutaneous (general) sensation to the skin of the upper lip, cheek, and lower eyelid is supplied by the maxillary (second) division of the trigeminal nerve (CN V2). CN V2 enters the infratemporal fossa via the foramen rotun- dum to reach the middle cranial fossa. The foramen rotundum was not identified in the X-ray, so paresthesia in these areas is unlikely. Choice D (Decreased salivation of the submandibu- lar gland) is incorrect. The chorda tympani nerve, a terminal branch of the facial nerve (CN VII), conveys taste sensation to the anterior two thirds of the tongue and carries presynap- tic (preganglionic) parasympathetic that will cause salivation of the submandibular (and sublingual) gland(s). The chorda tympani nerve exits the skull via the petrotympanic fissure, which is located in the infratemporal fossa. The petrotym- panic fissure was not identified in the X-ray, so loss of sali- vation from the submandibular gland is unlikely. Choice E (Decreased salivation of the parotid gland) is incorrect. The lesser petrosal nerve, a branch of the glossopharyngeal nerve (CN IX), carries presynaptic parasympathetic fibers that will cause salivation of the parotid gland. The lesser petrosal nerve may sometimes traverse the foramen ovale, which is identi- fied in the figure; however, it more often goes through other openings (petrosal foramen or sphenopetrosal fissure). There- fore, unilateral paralysis of the muscles of mastication is more likely.

A 60-year-old female yodeler with a 43-year history of smoking complains of pain during swallowing and hoarseness in her voice. A fiberoptic endoscopy reveals a laryngeal squamous cell carcinoma. What structure is most likely affected by this laryngeal cancer? (A) Infraglottic cavity (B) Vocal folds (C) Vestibularfolds (D) Laryngeal vestibule (E) Epiglottis

The answer is B: Vocal folds. The squamous cell carcinoma is identified on the mucosal surface at the inferiomedial border of the laryngeal ventricle, which would be the location of the vocal cords. The most common presenting symptom in laryngeal cancer, particularly in glottic tumors such as this one, is hoarse- ness of the voice while some people experience odynophagia (painful swallowing). Choice A (Infraglottic cavity) is incor- rect. The infraglottic cavity is the inferior part of the laryngeal cavity, so it lies inferior to the vocal folds and is continuous with the lumen of the trachea. Its inferior boundary would be the inferior border of the cricoid cartilage. Choice C (Vestibular folds) is incorrect. The laryngeal ventricle (laryngeal sinus) is a recess extending laterally within the middle portion of the laryngeal cavity. This recess lies between the superior vestibular folds and the inferior vocal folds. Choice D (Laryngeal vesti- bule) is incorrect. The laryngeal vestibule is located between the laryngeal inlet and the vestibular folds, and it represents the superior part of the laryngeal cavity. Choice E (Epiglottic cartilage) is incorrect. The epiglottic cartilage would form the anterior wall of the laryngeal vestibule, so it resides in the supe- rior part of the laryngeal cavity above the vestibular folds.

A 3-year-old girl presents with headache, vomiting, and papilledema. A sagittal MRI of the patient's head reveals a large craniopharyngioma, or a tumor arising from the remnants of Rathke pouch, as noted in the figure. This suprasellar tumor, located above the diaphragm sellae, is compressing the pituitary gland. Given the location and size of this tumor, what visual disturbances are likely to be seen in this patient? (A) Monocular blindness (B) Binasal hemianopsia (C) Bitemporal hemianopsia (D) Right homonymous hemianopsia (E) Left homonymous hemianopsia

The answer is C: Bitemporal hemianopsia. Bitemporal hemianopsia is commonly called "tunnel vision" due to the loss of vision in the temporal visual fields of both eyes. This visual impairment occurs when the medial retinal fibers are impinged while crossing within the optic chiasm. Tumors, such as pituitary adenomas, meningiomas, or craniopharyngiomas (as seen in this patient), are the most common cause of a lesion to the optic chiasm due to its midline location sit- ting superior to the sella turcica, which houses the pituitary gland. The craniopharyngioma seen on the MRI is compress- ing the pituitary gland, and it is most likely impinging the medial retinal fibers crossing within the optic chiasm, lead- ing to bitemporal hemianopsia. Choice A (Monocular blind- ness) is incorrect. Monocular blindness results from a lesion of the left or right optic nerve (CN II), which contains the retinal ganglion cell axons passing from the retina of the eye to the brain traversing the optic canal. The craniopharyngioma shown in this sagittal MRI is not extending anteriorly toward the termination of the optic nerves, so it is unlikely that this tumor would cause monocular blindness. Choice B (Binasal hemianopsia) is incorrect. Binasal hemianopsia, or loss of half the vision specifically in the nasal visual fields of both eyes, occurs when the uncrossed, lateral retinal fibers are impinged. Binasal hemianopsia is rarely seen, but can result due to con- genital hydrocephalus or calcification of both internal carotid arteries. The suprasellar craniopharyngioma in this patient would most likely impinge the medial retinal fibers, leading to bitemporal hemianopsia. Choice D (Right homonymous hemianopsia) is incorrect. Right homonymous hemianopsia results from a lesion of the left optic tract, left geniculocalcarine tract, or the left visual cortex. These structures are located too far posteriorly to be affected by the suprasellar craniopharyngioma seen in this patient. The word "homonymous" means same sided, so a right homonymous hemianopsia would refer to visual defects in both right halves of the visual field of each eye. Choice E (Left homonymous hemianopsia) is incorrect. Left homonymous hemianopsia, a visual deficit in the left halves of the visual fields in each eye, results from a lesion of the right optic tract, right geniculocalcarine tract, or the right visual cortex. These structures are located too far posteriorly to be affected by the suprasellar craniopharyngioma seen in this patient.

A 10-year-old boy underwent surgery, a bilateral palatine tonsillectomy. During a postoperative examination, the doctor noted the boy did not possess a gag reflex on the right side on the posterior tongue. He also complained of abnormal taste sensations in the back of his oral cavity. The soft pal- ate elevated symmetrically when the gag reflex was tested. No other signs or symptoms were noted. What cranial nerve was damaged during the tonsillectomy? (A) Trigeminal nerve (B) Facial nerve (C) Glossopharyngeal nerve (D) Vagus nerve (E) Hypoglossa lnerve

The answer is C: Glossopharyngeal nerve. The lingual and tonsillar branches of the glossopharyngeal nerve (CN IX) reside in the palatine tonsillar bed between the palatoglossal and palatopharyngeal folds. At this location, these branches of CN IX are susceptible to damage during a tonsillectomy, which would compromise taste and visceral sensation to the posterior one third of the tongue. Therefore, damage to CN IX at this location results in abnormal taste sensations and a loss of the afferent limb (CN IX) of the gag reflex on the ipsilateral side, respectively. These signs and symptoms were noted in this patient. Choice A (Trigeminal nerve) is incorrect. The trigeminal nerve (CN V) provides general sensation to the anterior two thirds of the tongue via the lingual nerve; however, in this patient, the right posterior part of the tongue is affected. Choice B (Facial nerve) is incorrect. Compromising the chorda tympani branch of the facial nerve leads to loss of taste sensation to the anterior two thirds of the tongue and decreased salivation due its parasympathetic innervation of the submandibular and sublingual salivary glands. The chorda tympani branch of the facial nerve performs these actions by joining the lingual nerve of the mandibular division of the trigeminal nerve (CN V3) to reach the oral cavity. However, the facial nerve is not involved in the gag reflex and does not supply taste to the posterior aspect of the tongue. Choice D (Vagus nerve) is incorrect. The vagus nerve does provide taste sensations in the area of the epiglottis and serves as the efferent limb of the gag reflex; however, CN X is not involved in this patient's signs and symptoms due to the symmetry in elevation of the soft palate noted during the postoperative examination. Choice E (Hypoglossal nerve) is incorrect. The hypoglossal nerve is responsible for innervation of all of the intrinsic and extrinsic muscles of the tongue, except the palatoglossus (innervated by the vagus nerve). Damage to CN XII would not affect the gag reflex or lead to abnormal taste sensations, but it would affect the musculature of the tongue.

A 57-year-old woman presents with right unilateral facial paralysis and dizziness. During an examination, the physician also notes a loss of hearing on the right side. An MRI of the patient's head reveals a brain tumor. Based upon the patient's presentation and MRI, where is the tumor located? (A) Foramen rotundum (B) Foramen ovale (C) Internal acoustic meatus (D) Facialcanal (E) Stylomastoid foramen

The answer is C: Internal acoustic meatus. This patient is displaying deficits associated with the facial nerve (CN VII), explaining the unilateral facial paralysis, and the vestibulocochlear nerve (CN VIII), evident from the dizziness and hear- ing loss on her right side. The internal acoustic meatus is the only location where both CN VII and CN VIII travel together. The MRI demonstrates a right-sided vestibular schwannoma (acoustic neuroma) located at the internal acoustic meatus, which confirms this diagnosis. This vestibular schwannoma resides at the cerebellopontine angle and affects the facial and vestibulocochlear nerves as they emerge from this location. This tumor would also increase intracranial pressure, poten- tially causing pontomedullary brain stem compression. Choice A (Foramen rotundum) is incorrect. The maxillary (second) division of the trigeminal nerve (CN V2) passes through the foramen rotundum carrying sensation from the cheek, lower eyelid, and upper lip. The facial and vestibulocochlear nerves, which show deficits in this patient, do not reside in this loca- tion. Choice B (Foramen ovale) is incorrect. The mandibular (third) division of the trigeminal nerve (CN V3) passes through the foramen ovale carrying cutaneous (sensory) information from the lower lip, chin, parotid region, anterior auricle, and lateral aspects of the scalp. CN V3 is the only division of the trigeminal nerve to carry motor (efferent) information, sup- plying the muscles of mastication derived from the mesoderm of first pharyngeal (branchial) arch. The facial and vestibu- locochlear nerves, deficient in this patient, do not reside in this location. Choice D (Facial canal) is incorrect. A tumor at this site would not impinge on the vestibulocochlear nerve, so hearing would not be impaired. A tumor in the facial canal would present with facial muscle paralysis and loss of the efferent limb of the corneal reflex, but would not explain the unilateral hearing impairment. Choice E (Stylomastoid foramen) is incorrect. A tumor located at this site would show facial muscle paralysis due to the involvement of the facial nerve; however, this lesion would not explain the involvement of the vestibulocochlear nerve (unilateral hearing loss).

A 27-year-old man comes to his family physician complaining of double vision. While sitting face-to-face, the doctor notes the patient exhibits strabismus, especially esotropia of the left eye, which gives the patient a "cross-eyed" appearance. When asked to follow the doctor's index finger with only his eyes, the patient is unable to look laterally, as illustrated in the figure. No other visual deficits are noted. What specific nerve is most likely damaged? (A) Left oculomotor nerve (B) Right oculomotor nerve (C) Left abducent nerve (D) Right abducent nerve (E) Left trochlear nerve

The answer is C: Left abducent nerve. The only extraocular muscle innervated by the left abducent nerve, CN VI, is the left lateral rectus muscle, which enables the left globe to move laterally following the doctor's index finger. When the left abducent nerve is damaged, the patient would display strabis- mus, causing diplopia, because the left pupil would rest in the adducted position, termed esotropia, due to the unopposed action of the medial rectus muscle. Because this patient dis- plays these symptoms, particularly the inability to abduct the left eye, the left abducent nerve is damaged. Choice A (Left oculomotor nerve) is incorrect. The left oculomotor nerve, CN III, innervates most of the extraocular eye muscles (with the exception of superior oblique and lateral rectus muscles) and the levator palpebrae superioris (which elevates the eyelid) in the ipsilateral (left) eye. Damage to the left CN III leads to diplopia (due to the left pupil resting in an abducted and lateral position) and ptosis (drooping of the eyelid) associ- ated with the left eye. Because of the esotropia evident in this patient without ptosis, this option can be eliminated. Choice B (Right oculomotor nerve) is incorrect. The right oculomotor nerve, CN III, innervates most of the extraocular eye muscles (with the exception of superior oblique and lateral rectus mus- cles) and the levator palpebrae superioris (which elevates the eyelid) in the ipsilateral (right) eye. Damage to the right CN III leads to diplopia (due to the right pupil resting in an abducted and lateral position) and ptosis (drooping of the eyelid) associ- ated with the right eye. Because of the esotropia evident in this patient without ptosis, this option can be eliminated. More- over, the left eye was affected in this patient. Choice D (Right abducent nerve) is incorrect. The only extraocular muscle innervated by the right abducent nerve, CN VI, is the right lateral rectus muscle, which directs the right globe laterally. If this nerve were damaged, the patient would display diplopia, and the right pupil would be resting in the adducted position due to the unopposed action of the other extraocular muscles. However, the left eye was affected in this patient. Choice E (Left trochlear nerve) is incorrect. The left trochlear nerve innervates only one muscle, the superior oblique muscle, in the orbit. This muscle pulls the eye inferolaterally, but it is hard to distinguish this movement from the combined move- ments of the inferior rectus muscle (innervated by the oculo- motor nerve) and the lateral rectus muscle (innervated by the abducent nerve), which move the globe inferior and lateral, respectively. Therefore, the left trochlear nerve is clinically tested by asking the patient to look inferiorly after the left eye is placed in an adducted position. Damage to this nerve also causes diplopia, or double vision, especially when the gaze is directed inferiorly and medially, but the esotropia is not evi- dent in the left eye of this patient.

A 75-year-old man tells his physician he has been having progressively more trouble opening his left eye because his upper eyelid tends to droop. Which of the following muscles is most likely weakened? (A) Orbicularis oculi (B) Frontalis (C) Levator palpebrae superioris (D) Superior rectus (E) Orbital muscle

The answer is C: Levator palpebrae superioris. The levator palpebrae superioris muscle attaches into the tarsal plate and skin of the upper eyelid and is the primary elevator of the eye- lid. Weakness in this muscle results in ptosis (drooping) of the upper eyelid and may reflect a problem with the oculomotor nerve (CN III). The oculomotor nerve provides the motor con- trol for the levator palpebrae superioris and most (4 of 6) of the extraocular muscles. The levator palpebrae superioris is assisted by the superior tarsal muscle, a thin smooth muscle sheet in the upper eyelid innervated by sympathetic fibers. Choice A (Orbicularis oculi) is incorrect. The orbicularis oculi is a muscle of facial expression that acts to close the eye, form- ing a sphincter-like arrangement around the orbit and extend- ing into the eyelids. Functional deficits of the orbicularis oculi muscle result in loss of the ability to blink, which endangers the health of the eye by hampering proper spread of tears across the eyeball. Choice B (Frontalis) is incorrect. The fron- talis muscle is the anterior component of the epicranius muscle in the scalp. It elevates the eyebrows and produces the hori- zontal wrinkles across the forehead, occurring when a person looks up (superior). Choice D (Superior rectus) is incorrect. The superior rectus muscle is one of the extraocular muscles. It attaches onto the sclera on the superior aspect of the eye and acts as the primary elevator of the eyeball. Choice E (Orbital muscle) is incorrect. The orbital muscle is a rudimentary smooth (nonstriated) muscle sling across the orbit that helps to support and position the eyeball within the orbit.

A 31-year-old woman with an ongoing, long-term history of alcoholism becomes pregnant. The embryo she is carrying suffers a neural crest insufficiency during the critical period of development. Which of the following structures is most likely to be malformed as a result of this condition? (A) Laryngeal cartilages (B) Parietal bones (C) Maxillary bones (D) Common carotid arteries (E) Thyroid gland

The answer is C: Maxillary bones. Neural crest cells arise in association with the neuroectoderm of the developing brain, and migrate into the pharyngeal arches, around the forebrain, and into the facial region. In these regions, neural crest cells account for numerous skeletal and other struc- tures, including cartilage, tendon, dentin, dermis, sensory ganglia, and the arachnoid and pia mater. Therefore, devel- opment and migration of sufficient numbers of neural crest cells are critical for formation of much of the head and neck, especially the craniofacial region. Because neural crest cells also contribute significantly to heart formation, many infants with craniofacial defects also exhibit cardiac malformations. Unfortunately, neural crest cells are highly sensitive to envi- ronmental teratogens such as alcohol and retinoic acid. Over- exposure to these substances (e.g., in fetal alcohol syndrome) may reduce production, kill, and/or limit the migration of the neural crest cells into their target regions. Neural crest cells form virtually all the facial skeleton, including the frontal, maxillary, zygomatic, squamous temporal, and mandibular bones, plus other smaller bones. Neural crest insufficiency is the major cause of palatofacial clefts that result from fail- ure of fusion of facial primordia. Choice A (Laryngeal carti- lages) is incorrect. Although neural crest cells form most of the skeletal structures (bone and cartilage) derived from the pharyngeal arches, the laryngeal cartilages are notable excep- tions, derived from the lateral plate mesoderm of the fourth and sixth arches. Choice B (Parietal bones) is incorrect. Most of the cranial vault (including the parietal, occipital, and petrous portion of the temporal bones) and the cranial floor are formed from paraxial mesoderm (somites and somi- tomeres). Choice D (Common carotid arteries) is incorrect. In general, the aortic arches that run within the pharyngeal arches are derived from the mesoderm of the pharyngeal arches, along with the muscles that originate there. Thus, the arteries derived from the aortic arches, including the com- mon carotid arteries, have mesodermal origins. Choice E (Thyroid gland) is incorrect. The thyroid gland originates as an epithelial (endodermal) growth in the floor of the embry- onic pharynx. It proliferates as a single, ventromedian diver- ticulum (thyroid diverticulum) off the pharynx, and descends into the neck.

A 35-year-old man complains to his physician that he feels congested, has trouble with nasal breathing, and is experienc- ing a yellowish nasal mucus discharge. He also mentions that his right side upper molar teeth ache terribly. A thorough phys- ical examination reveals maxillary sinusitis. The discharge from this sinus initially drains into the nasal cavity at which of the labeled points within this drawing of the lateral nasal wall? (A) Sphenoethmoidal recess (B) Superior nasal meatus (C) Middle nasal meatus (D) Inferior nasal meatus (E) Pharyngotympanic tube

The answer is C: Middle nasal meatus. The lateral nasal wall is formed largely by three nasal conchae (turbinate bones). The superior and middle conchae are parts of the ethmoid bone. The inferior concha is an independent bone. The conchae divide the nasal passage into four air channels: the sphenoeth- moidal recess (above the superior concha), the superior meatus (below the superior concha), the middle meatus (below the middle concha), and the inferior meatus (below the inferior concha). The paranasal sinuses open into the nasal passage on its lateral wall, in specific relations to the conchae and meati. The maxillary sinus is the largest paranasal sinus, occupying much of the maxillary bone. Infections there may affect the upper teeth, causing a toothache, because the sinus cavity is separated from the roots of the teeth by only a very thin layer of bone. The maxillary sinus opens into the most posterior part of the semilunar hiatus in the middle meatus. The frontal, anterior ethmoidal, and middle ethmoidal sinuses also open nearby into the middle meatus. Choice A (Sphenoethmoidal recess) is incorrect. The sphenoethmoidal recess receives the drainage of the sphenoidal sinus. Choice B (Superior nasal meatus) is incorrect. The superior nasal meatus contains the opening of the posterior ethmoidal sinuses (air cells). Choice D (Inferior nasal meatus) is incorrect. The inferior nasal meatus does not receive the opening of any paranasal sinus. Instead, it contains the opening of the nasolacrimal duct, thus receiving drainage of tears from the eye. Choice E (Pharyngotympanic tube) is incorrect. The pharyngotympanic (auditory; eusta- chian) tube opens into the nasopharynx, posterior to the nasal passage. It is not a component of any nasal meatus.

A 27-year-old woman with green eyes comes to her physician with noted asymmetry in her pupils. Her right pupil is abnormally dilated, and on examination, the right eye is slow to respond to light stimuli. Her visual acuity is not impaired, and no other signs and symptoms are noted. What structure is most likely affected in this patient? (A) Superior cervical ganglion (B) Optic nerve (C) Oculomotor nerve (D) Trochlear nerve (E) Abducent nerve

The answer is C: Oculomotor nerve. The slowness of the right pupil to respond to light stimuli is the first sign of compression of the oculomotor nerve (CN III). In this patient, it is the para- sympathetic component of the right oculomotor nerve that is affected due to the loss of the sphincter pupillae muscle, which results in dilation of her right pupil. When a patient has an ipsilateral dilation of the pupil and lethargy (the latter symptom was not noted in this patient), a physician must rule out a supratentorial (above the tentorium cerebelli) brain lesion caus- ing increased intracranial pressure (ICP). This type of lesion could result from a blockage of cerebrospinal fluid (CSF) flow, an intracranial bleed (hemorrhage), or a supratentorial tumor, and these lesions often affect the parasympathetic component of CN III by impinging this nerve component at the tentorial notch, often seen in an uncal herniation. Choice A (Superior cervical ganglion) is incorrect. Damage to the superior cervical ganglion results in loss of sympathetic innervation to the head. If this structure were damaged, a patient would exhibit miosis (constriction of the pupil), ptosis (drooping of the eyelid), and anhydrosis (inability to sweat) on the affected side, which is characterized as Horner syndrome, or loss of sympathetic innervation to the head. In this patient, the right pupil is dilated, so the sympathetic innervation is intact but unopposed by the loss of the parasympathetic component of the oculomotor nerve. Choice B (Optic nerve) is incorrect. If the right optic nerve were compromised, the patient would be blind in his right eye. The optic nerve is not affected because the visual acuity of the patient is not impaired. Choice D (Trochlear nerve) is incorrect. The right trochlear nerve pro- vides innervation to the superior oblique muscle in the orbit solely. This muscle pulls the eye inferolaterally, but it is tested clinically by putting the eye in an adducted position because the superior oblique muscle is the only extraocular muscle that can direct the gaze of the adducted eye inferiorly. Damage to the trochlear nerve leads to diplopia, or double vision. However, it would not account for the asymmetric pupils seen in this patient. Choice E (Abducent nerve) is incorrect. The lateral rectus muscle is the only extraocular muscle innervated by the abducent nerve. If this nerve were cut, the patient would display diplopia. However, involvement of the abdu- cent nerve would not account for the asymmetric pupils seen in this patient.

A 64-year-old male professional angler is diagnosed with a skin melanoma above his right eyebrow after years of excessive sun exposure. His dermatologist removes this cancerous lesion, but the doctor needs to rule out possible metastasis. What group of lymph nodes would his physician first check for possible spread of the cancer? (A) Deep cervical (B) Retroauricular (C) Parotid (D) Submental (E) Submandibular

The answer is C: Parotid. The parotid lymph nodes receive lymphatic drainage from the lateral part of the anterior scalp, anterior part of the auricle, and the lateral part of the face, including the upper and lower eyelids. Due to the site of the skin melanoma above the right eyebrow, the physician must first check the parotid lymph nodes for spread of cancer. This task can be accomplished by sentinel lymph node mapping, which is a technique for locating the lymph node that is most likely to receive primary drainage from the melanoma. The sentinel node, identified by this procedure, is the most likely lymph node to contain cancer and can be surgically removed if lymphogenous spread of cancer is suspected. Choice A (Deep cervical) is incorrect. The deep cervical lymph nodes run along the internal jugular vein to return lymph to the systemic venous return at the junction of the internal jugular and subclavian veins (or the venous angle). This set of lymph nodes receives lymphatic drainage from deep structures within the neck, including the larynx and pharynx. These lymph nodes would receive secondary drainage of the tumor site if the cancer is more advanced in staging; however, the deep cervical lymph nodes would not be the first nodes to be checked by the physician. Choice B (Retroauricular) is incor- rect. The retroauricular lymph nodes receive lymphatic drain- age from the posterior part of the auricle and lateral aspects of the middle scalp. However, these areas do not include the anterior scalp above the forehead where the melanoma was located in this patient. Choice D (Submental) is incorrect. The submental lymph nodes receive lymphatic drainage from the middle part of the lower lip, chin, frenulum and tip of the tongue, and the anterior floor of the mouth. These submental lymph nodes can be adversely affected by prolonged smoke- less tobacco (or cigar) use, in which the tobacco is placed posterior to the lower lip and anterior to the lower incisor teeth. Knowledge of the lymphatic drainage of the oral cav- ity is important clinically because oral and pharyngeal cancer is the sixth most common malignancy reported worldwide. Early detection is crucial because the 5-year survival rate for oral cancer is only 50%. However, this patient's melanoma, located above the right eyebrow, would not drain to the sub- mental lymph nodes. Choice E (Submandibular) is incorrect. The submandibular lymph nodes receive lymphatic drainage from the lateral parts of the anterior two thirds of the tongue, upper lips, lateral aspects of the lower lip, cheek, and nose. Concerning the oral cavity, the submandibular lymph nodes receive everything that is not directly drained by the submen- tal nodes, which also eventually drain into the submandibular nodes. Prolonged use of chewing tobacco can pathologically alter the labial mucosa in the lateral aspects of the mouth. However, this patient's melanoma, located above the right eye- brow, would not drain to the submandibular lymph nodes.

After asking a 47-year-old woman to open her mouth wide and say "Ah," the physician notes deviation of the uvula to the left side and asymmetry in the elevation of the soft palate, with the right side of the palate sagging. What specific nerve is most likely damaged? (A) Left glossopharyngeal nerve (B) Left vagus nerve (C) Right vagus nerve (D) Left hypoglossal nerve (E) Right hypoglossal nerve

The answer is C: Right vagus nerve. Damage to the right vagus nerve is causing asymmetry in soft palate elevation and contralateral deviation of the uvula of the soft palate to the left side. The pharyngeal branches of the vagus nerve innervate all the musculature of the soft palate, except the tensor veli palatini (tensor of the soft palate), which is innervated by the mandibular division of the trigeminal nerve (CN V3). On examination, the arch of the soft palate droops on the affected side (right side) and the uvula deviates to the unaffected side (left side) as a result of the unopposed action of the intact muscles acting on the soft palate. This patient may report dysphagia, difficulty in swallowing, and nasal regurgitation, due to reduced muscular tone within the soft palate. Choice A (Left glossopharyngeal nerve) is incorrect. The glossopharyngeal nerve only innervates one muscle, the stylopharyngeus, which is a pharyngeal muscle. Because it does not innervate any muscles of the soft palate, the glossopharyngeal nerve is unable to affect the elevation of the soft palate or deviation of the uvula. Choice B (Left vagus nerve) is incorrect. The left vagus nerve is not injured because the patient's uvula deviates to the left, a sign that the contralateral (right) vagus nerve is damaged. Choice D (Left hypoglossal nerve) is incorrect. The left hypoglossal nerve does not innervate any muscles of the soft palate, so it would not be involved in the deviation of the uvula. Damage to the left hypoglossal nerve causes the tongue to deviate ipsilaterally (to the left side) when the patient protrudes the tongue. Choice E (Right hypoglossal nerve) is incorrect. The right hypoglossal nerve does not innervate any muscles of the soft palate and would not cause the deviation of the uvula. A lesion of the right hypoglossal nerve causes the tongue to deviate ipsilaterally (to the right side) when the patient protrudes the tongue.

A 45-year-old man is in surgery. While seated at the head of the operating table, the anesthesiologist periodically checks the patient's pulse by palpating the artery located anterior to the tragus of the external ear. Which of the following arteries is being utilized to monitor the patient's pulse? (A) Maxillary (B) Posterior auricular (C) Superficial temporal (D) Facial (E) Internal carotid

The answer is C: Superficial temporal. The superficial tem- poral artery is the smaller terminal branch of the external carotid artery. It ascends through the parotid gland, anterior to the auricle, and crosses the zygomatic arch to reach the temporal fossa and scalp. Its pulse (the temporal pulse) can be palpated anterior to the tragus of the external ear where the artery lies against the underlying zygomatic arch. Choice A (Maxillary) is incorrect. The maxillary artery is the larger ter- minal branch of the external carotid artery. From its origin within the substance of the parotid gland, it passes anterior, deep to the ramus of the mandible, to enter the infratem- poral fossa. Its deep location makes this artery an unlikely candidate for taking a pulse. Choice B (Posterior auricular) is incorrect. The posterior auricular artery is a branch of the external carotid artery that runs posterior, near the styloid process to pass behind the ear. It supplies the auricle and the scalp, posterior to the auricle. It does not provide a pal- pable pulse location. Choice D (Facial) is incorrect. The facial artery is another branch of the external carotid artery. It ascends anterior, deep to the posterior belly of the digastric and stylohyoid muscles and the submandibular gland, and crosses the mandible to enter the face. Its pulse can be read- ily palpated at the point where it crosses the inferior mar- gin of the body of the mandible, at the anterior border of the masseter muscle. However, the pulse was taken anterior to the tragus of the auricle. Choice E (Internal carotid) is incorrect. This large vessel is one of the terminal branches of the common carotid artery. It arises deep in the upper neck and ascends into the base of the skull. Whereas the common carotid can be palpated lower in the neck, the internal carotid is too deep to detect a pulse.

A 47-year-old woman with a history of multiple sclerosis comes to her doctor complaining of sudden bursts (paroxysms) of pain in her mandible, especially in the lower lip, mandibular teeth and gingivae, and cheek on her right side. This debilitating pain is often triggered by eating, talking, or brushing her teeth and often gets worse as the day progresses. Which nerve is the source of her pain? (A) First division of trigeminal nerve (B) Second division of trigeminal nerve (C) Third division of trigeminal nerve (D) Buccal branch of facial nerve (E) Marginal mandibular branch of facial nerve

The answer is C: Third division of trigeminal nerve. The third (mandibular) division of the trigeminal nerve (CN V3) supplies general sensation to the skin of the lower lip, chin, cheek, and even the anterior auricle and the lateral scalp. This sensory innervation is supplied via three cutaneous nerves: mental, buccal, and auriculotemporal. The mandibular division of the trigeminal nerve also supplies innervation to the mandibular teeth and gingivae via the inferior alveolar nerve. This patient is suffering from trigeminal neuralgia (or tic douloureux), often seen in patients suffering from demyelinating diseases such as multiple sclerosis, affecting CN V3. Trigeminal neuralgia is characterized by episodes of pain that occur suddenly, and debilitating pain can often be triggered by stimuli within the distribution area of the nerve affected, which was seen following eating, talking, or brushing her teeth in this patient. Choice A (First division of the trigeminal nerve) is incorrect. The first (ophthalmic) division of the trigeminal nerve (CN V1) supplies sensory (cutaneous) innervation to the skin of the upper eyelid, anterior aspect of the nose, forehead, and ante- rior scalp. The sensory distribution of the ophthalmic division of the trigeminal nerve does not correlate to the areas of the face affected in this patient, so this option can be eliminated. Choice B (Second division of trigeminal nerve) is incorrect. The second (maxillary) division of the trigeminal nerve (CN V2) supplies sensory (cutaneous) innervation to the skin to the lower eyelid, cheek, and upper lip, upper dentition and gingivae, maxillary sinus, and lateral aspect of the nose. The sensory distribution of the maxillary division of the trigeminal nerve does not correlate to the areas of the face affected in this patient, so this option can be eliminated. Choice D (Buccal branch of facial nerve) is incorrect. The buccal branch of the facial nerve (CN VII) is one of five terminal branches of the main trunk of CN VII, which supplies the muscles of facial expression and other muscles derived from mesoderm in the embryonic second pharyngeal arch. This buccal branch is entirely efferent (motor) in its innervation supplying the buccinator muscle and muscles of the upper lip. It does not have a sensory component, so this nerve would not be the source of this patient's pain. Choice E (Marginal mandibular branch of the facial nerve) is incorrect. The marginal mandibular branch of the facial nerve (CN VII) is another one of five terminal branches of the main trunk of CN VII. This nerve only has an efferent (motor) component supplying the muscles of lower lip and chin. It does not have a sensory component, so this nerve would not be the source of this patient's pain.

A 50-year-old man presents with recurring dizziness, ataxia, vertigo, aphasia, and weakness in his right upper limb. A magnetic resonance angiogram (MRA) revealed a stenosis of the right subclavian artery (marked by the arrow) and poststenotic dilatation, which led to the diagnosis of subclavian steal syndrome. In this condition, blood is shunted from the left side arterial tree via collateral flow into the right side circulation. Through which of the following ipsilateral vessels is blood entering the right subclavian artery distal to the stenosis? (A) Internal thoracic artery (B) Common carotid artery (C) Vertebral artery (D) Superior thyroid artery (E) Suprascapular artery

The answer is C: Vertebral artery. The stenosis of the right subclavian artery depicted in the MRA causes reduced blood flow to the right upper limb, leading to weakness. This steno- sis causes the anatomical subclavian steal syndrome, in which the right vertebral artery, which is dilated in the provided MRA, delivers blood back into the occluded subclavian artery. With the stenosis of the proximal end of the right subclavian artery, the resulting arterial pressure differentials enable blood to enter the right vertebral artery after being "stolen" from the left (contralateral) internal carotid tract via the cerebral arterial circle (of Willis) and basilar circulation within the skull. In this patient, blood would travel retrograde within the right verte- bral artery to circumvent the stenosis in the proximal right subclavian artery and re-establish blood flow to the right limb. This collateral pathway enables continued use of the right limb; however, the blood being diverted away from the brain can cause brainstem and/or cerebral ischemia, and possibly lead to a stroke. The symptoms seen in this patient, including dizziness, ataxia, vertigo, and aphasia, are indicators of the vertebrobasilar insufficiency, which tells the physician that the collateral circulation is emanating from the cranial circulation. A mnemonic for the four branches of the subclavian artery is "VITamin C," which stands for the Vertebral artery, Internal thoracic artery, Thyrocervical trunk, and Costocervical trunk. Choice A (Internal thoracic artery) is incorrect. The internal thoracic (internal mammary) artery arises from the subcla- vian artery as its second branch, typically distal to the origin of the vertebral artery. It descends into the thorax along the edge of the sternum to supply the thoracic and abdominal walls, and it receives extensive and important collateral con- nections along its course. Theoretically, the internal thoracic artery could contribute to bypassing the stenosis of the subcla- vian artery; however, this artery is not dilated in the provided MRA and its involvement in this case of anatomical subcla- vian steal syndrome would not account for the cranial isch- emia and resulting symptoms. Choice B (Common carotid artery) is incorrect. The brachiocephalic trunk ends by divid- ing into its terminal branches, the right common carotid and right subclavian arteries. The stenosis depicted in the given MRA is distal to the origin of the common carotid artery, so it would not deter blood from entering into the common carotid artery and would not set up the arterial pressure differentials needed for reverse flow within it. The common carotid artery has no direct connections distal to the stenosis of the subcla- vian artery, so this artery is unable to bypass the occlusion. Choice D (Superior thyroid artery) is incorrect. The superior thyroid artery is the first branch of the external carotid artery, which supplies the thyroid gland and neighboring muscles. If blood traveled retrograde through the superior thyroid artery, it would travel back toward the common carotid artery, which is located proximal to the stenosis of the right subclavian artery. Choice E (Suprascapular artery) is incorrect. The thy- rocervical trunk arises as the third branch of the first part of the subclavian artery, proximal to the anterior scalene muscle. The thyrocervical trunk typically divides into four branches: suprascapular, inferior thyroid, ascending cervical, and trans- verse cervical arteries. The suprascapular artery runs across the root of the neck to the superior border of the scapula where it supplies blood to the dorsal aspect of the scapula. It does not have any relations to the branching pattern needed to compensate for the stenosis of the subclavian artery in this anatomical subclavian steal syndrome.

A 23-year-old man reports to physician due to shoulder weak- ness and instability. After removing his shirt, his left shoulder appears to reside lower than his right shoulder, asymmetry noted. During an examination, the patient is unable to abduct his left arm over his head and shows an inability to shrug (or elevate) his left shoulder against resistance. What nerve was most likely damaged in this patient? (A) Facial nerve (B) Glossopharyngeal nerve (C) Vagus nerve (D) Accessory nerve (E) Hypoglossal nerve

The answer is D: Accessory nerve. The accessory nerve (CN XI) provides motor innervation to the sternocleidomastoid muscle and the trapezius muscle. The asymmetry of the shoulders (shown in the figure) suggests paralysis of the left trapezius muscle. Actions of the trapezius include elevation of the scapula and lateral rotation of the scapula during abduction greater than 90 degrees. Both of these actions were affected in this patient. Damage to the accessory nerve also leads to "drooping" of the affected shoulder due to loss of innervation and subsequent loss of muscle tone (atrophy) of the trapezius muscle. Choice A (Facial nerve) is incorrect. Damage to the facial nerve (CN VII) could result in the loss of taste sensation to the anterior two thirds of the tongue, decreased salivation due its innervation of the submandibular and sublingual salivary glands, unilateral facial paralysis, hyperacusis due to loss of stapedius muscle function, and even dysphagia due to its innervation of the posterior belly of the digastric and stylohyoid muscles, which have attachments to the hyoid bone. These signs or symptoms were not noted in this patient. Choice B (Glossopharyngeal nerve) is incorrect. Damage to the glossopharyngeal nerve (CN IX) could result in decreased salivation due to its innervation of the parotid gland and loss of taste and visceral sensation to the posterior one third of the tongue. Therefore, a lesion compromising this nerve would result in a dry mouth, abnormal taste sensations, and dysphagia due to loss of a gag reflex and loss of innervation to the stylopharyngeus muscle. These signs or symptoms were not noted in this patient. However, glossopharyngeal nerve dysfunction may be encountered along with accessory nerve dysfunction because they exit the skull through the same open- ing, the jugular foramen. Choice C (Vagus nerve) is incorrect. Damage to the vagus nerve could result in problems with speech (dysphonia) and swallowing (deglutition) due to loss of the efferent limb of the gag reflex and loss of motor innervation to the majority of palatal, pharyngeal, and laryngeal musculature. These symptoms were not noted in this patient. However, vagus nerve dysfunction may be encountered along with accessory nerve dysfunction because they exit the skull through the same opening, the jugular foramen. Choice E (Hypoglossal nerve) is incorrect. The hypoglossal nerve is responsible for innervation of all of the intrinsic and extrinsic muscles of the tongue, except the palatoglossus (innervated by the vagus nerve). Damage to CN XII would not alter the actions of the left trapezius muscle, but it would affect the musculature of the tongue.

A 70-year-old woman goes to her physician complaining of headache, nausea, and vomiting. A head MRI reveals a tumor disrupting the visceral afferent fibers running into the solitary nucleus located within the medulla. What signs or symptoms would be manifested following the degeneration of these nerve fibers entering the solitary nucleus? (A) Blindness (B) Loss of sensation from the cornea (C) Loss of equilibrium (D) Altered taste (E) Deafness

The answer is D: Altered taste. The tumor impinges on the afferent fibers entering the solitary nucleus, which include visceral sensation and taste from the facial (CN VII), glossopharyngeal (CN IX), and vagus (CN X) nerves. Altered taste perceptions would most likely be present in this patient. These taste fibers are sometimes referred to as special visceral afferent (SVA) neurons, which relay the special senses of taste and olfaction. Damage to the taste information going to the solitary nucleus and carried by CN VII, CN IX, and CN X would result in altered taste. Moreover, the solitary nucleus receives visceral sensory inputs, which include chemoreceptors and baroreceptors located at the termination of the common carotid artery, and these nerve fibers would be involved with many reflexes within the head region, including the carotid sinus and aortic reflexes. Choice A (Blindness) is incorrect. Vision is a special sense that is relayed by the optic nerve (CN II). CN II input provides whole body (somatic) position and orientation in space (the external environment). This afferent information carried by CN II is referred to as special sensory (special somatic afferent or SSA) neurons, and it is not related to the solitary nucleus. Choice B (Loss of sensation from the cornea) is incorrect. Developmentally, the cornea is derived from surface ectoderm (i.e., the body soma). Sensory information from the cornea would be classified as general sensory (general somatic afferent or GSA) fibers, and this information is carried by the ophthalmic division of the trigeminal nerve (CN V1). This sensory information is not related to the solitary nucleus. Choice C (Loss of equilibrium) is incorrect. As with vision, equilibrium and balance are related to whole body orientation in space. Thus, these senses receive the special sensory (SSA) designation. This sensory information carried by the vestibular portion of the vestibulocochlear nerve (CN VIII) is not related to the solitary nucleus. Choice E (Deafness) is incorrect. As with vision, auditory sensations are related to whole body orientation in space. Thus, hearing receives the special sensory (SSA) designation. This sensory information carried by the cochlear portion of the vestibulocochlear nerve (CN VIII) is not related to the solitary nucleus.

A baby girl presents with a disproportionately wide skull with a short occipitofrontal diameter. What cranial suture most likely closed prematurely to result in this cranial deformity? (A) Sphenosquamous (B) Sphenoparietal (C) Lambdoid (D) Coronal (E) Sagittal

The answer is D: Coronal. Premature closure of the coronal suture leads to brachycephaly (G: short head), which leads to a disproportionately wide skull with a short occipitofron- tal diameter. The surface shaded CT reconstruction shows the complete closure of the coronal suture and depicts the characteristic square-shaped skull with a short occipitofron- tal diameter seen following premature closure of the coronal suture. Brachycephaly is more common in females, and sur- gical intervention can be implemented to remove bone from both coronal sutures. Interestingly, some infants wear molding caps to treat cranial deformities, if surgical intervention is not required. If only one side of the coronal suture closes prema- turely, the infant would present with an asymmetric cranium, a condition known as plagiocephaly. Craniosynostosis is the term that refers generally to the premature fusion of the cra- nial sutures. Choice A (Sphenosquamous) is incorrect. The sphenosquamous suture is a dense, fibrous connective tissue joint located on the side of the skull between the greater wing of the sphenoid bone and the squamous portion of the tem- poral bone. This suture later closes to help form the pterion, which is clinically relevant due to the fractures located at this structurally weak area on the side of the head, which may cause epidural hemorrhage via damage to the middle menin- geal artery. The premature closure of this suture, as well as the sagittal, parietotemporal, and sphenoparietal sutures, can be involved with scaphocephaly (G: boat-shaped skull). Choice B (Sphenoparietal) is incorrect. The sphenoparietal suture is a dense, fibrous connective tissue joint located on the side of the skull between the greater wing of the sphenoid bone and the parietal bone. This suture later closes to help form the pterion, which is clinically relevant due to the fractures located at this structurally weak area on the side of the head, which may cause epidural hemorrhage via damage to the middle menin- geal artery. The premature closure of this suture, as well as the sagittal, parietotemporal, and sphenosquamous sutures, can be involved with scaphocephaly (G: boat-shaped skull). Choice C (Lambdoid) is incorrect. The lambdoid suture is a dense, fibrous connective tissue joint located on the back of the skull that connects the occipital bone with the posterior aspect of the parietal bone and petrous portion of the temporal bone. If only one side of the lambdoid suture closes prematurely, the infant would present with a twisted and asymmetric cra- nium, a condition known as plagiocephaly. Choice E (Sagittal) is incorrect. The sagittal suture is a dense, fibrous connective tissue joint located between the two parietal bones in the mid- line of the skull. The premature closure of this suture, as well as the parietotemporal, sphenosquamous, and sphenoparietal sutures, can be involved with scaphocephaly (G: boat-shaped skull), which presents as a long and narrow cranium.

A physician noticed a keyhole appearance of the right pupil in a 21-year-old woman characteristic of a defect of the iris known as coloboma. When asked about her affected eye, the patient responds that she was born with the condition. What is the most likely cause of this coloboma of the iris? (A) Traumatic damage to the sphincter muscle of the pupil (B) Interruption of neural crest cell migration (C) Persistent pupillary membrane (D) Failure of fusion of the retinal fissure (E) Lack of fusion of inner and outer layers of the optic cup

The answer is D: Failure of fusion of retinal fissure. Coloboma of the iris results from failure of fusion of the retinal (or choroidal) fissure, a ventral groove formed by the invagination of the optic cup and its stalk by vascular mesenchyme, during the sixth week of development. It is characterized by a defect of the inferior portion of the iris in the pupillary margin, which gives the pupil a keyhole appearance. This congenital defect can be an autosomal dominant malformation, which may or may not affect vision. Choice A (Traumatic damage to the sphincter muscle of the pupil) is incorrect. The keyhole defect in the right iris appears as an irregular pupil that can be caused by damage to the iris itself, loss of innervation to the sphincter or dilator mus- cle of the pupil, or insults to the central nervous system. In this patient, the iris coloboma was congenital, so traumatic damage to the sphincter muscle of the pupil can be ruled out. Choice B (Interruption of neural crest cell migration) is incorrect. The iris and its dilator and sphincter muscles of the pupil are derived from neuroectoderm, so the interruption of neural crest cell migration would not cause coloboma of the iris. Choice C (Per- sistent pupillary membrane) is incorrect. Remnants of the pupil- lary membrane, which covers the anterior surface of the lens in the embryo, may persist in the pupils of newborns, especially premature infants. A persistent pupillary membrane appears as web-like strands of connective tissue over the pupil, but this tissue tends to atrophy over time. In this patient, a defect in the iris was seen in a 21-year-old woman, so a persistent pupillary membrane is unlikely. Choice E (Lack of fusion of inner and outer layers of the optic cup) is incorrect. Failure of fusion of the inner and outer layers of the optic cup may lead to a congenital detachment of the retina due to the persistent intraretinal space, which impairs vision and would be surgically repaired (if pos- sible) after birth. A detached retina would not lead to the defect of the iris seen in this 21-year-old patient.

In a CT scan of the head, an opening in the skull is identified as the jugular foramen. What cranial nerve exits the cranium via this opening? (A) Trochlear nerve (B) Trigeminal nerve (C) Facial nerve (D) Glossopharyngeal nerve (E) Hypoglossal nerve

The answer is D: Glossopharyngeal nerve. The jugular foramen is identified in this CT, and this cranial opening transmits the glossopharyngeal (CN IX), vagus (CN X), and accessory (CN XI) nerves. The glossopharyngeal nerve (CN IX) is the only nerve listed in this question that exits the posterior cranial fossa via the jugular foramen. Choice A (Trochlear nerve) is incorrect. The trochlear nerve (CN IV) is the only cranial nerve that emerges from the posterior surface of the midbrain, and it has the longest intracranial course of any cranial nerve. CN IV courses anteriorly to reach the orbit after traversing the superior orbital fissure. However, the jugular foramen was identified in the CT scan. Choice B (Trigeminal nerve) is incorrect. The trigeminal nerve (CN V) is the main sensory nerve for the face and scalp, and it has three divisions, namely, the ophthalmic (CN V1), maxillary (CN V2), and mandibular (CNV ). CN V travels from the orbit entering the middle cranial fossa via the superior orbital fissure. CN V2 travels through the foramen rotundum. Finally, CN V3 traverses the foramen ovale to reach the infratemporal region. However, the jugular fora- men was identified in the CT scan. Choice C (Facial nerve) is incorrect. The facial nerve (CN VII) exits the posterior cranial fossa through the internal acoustic meatus, along with the ves- tibulocochlear nerve (CN VIII). However, the jugular foramen was identified in the CT scan. Choice E (Hypoglossal nerve) is incorrect. The hypoglossal nerve (CN XII) exits the poste- rior cranial fossa through the hypoglossal canal. However, the jugular foramen was identified in the CT scan.

23-year-old man has an impacted left third mandibular molar (or wisdom) tooth extracted. Following the surgery, the patient reports numbness in the anterior aspect of his tongue. Which of the following nerves is damaged? (A) Chorda tympani nerve (B) Mylohyoid nerve (C) Inferior alveolar nerve (D) Lingual nerve (E) Glossopharyngeal nerve

The answer is D: Lingual nerve. The lingual nerve is a branch of the mandibular division of the trigeminal nerve (CN V3), which traverses the foramen ovale and resides in the infratemporal fossa. This nerve supplies general sensation to the anterior two thirds of the tongue, and this nerve is at risk during extraction of an impacted third mandibular molar tooth. Choice A (Chorda tympani nerve) is incorrect. The chorda tympani, a branch of the facial nerve (CN VII), joins the lingual branch of the mandibular nerve in the infratemporal fossa. The chorda tympani nerve conveys taste sensation to the anterior two thirds of the tongue and carries presynaptic parasympathetic fibers to the submandibular ganglion for innervation of the submandibular and sublingual salivary glands. While the chorda tympani nerve merges with the lingual nerve to reach its effector region, cutting the chorda tympani nerve would not result in numbness on the tip of the tongue. Choice B (Mylohyoid nerve) is incorrect. The mylohyoid nerve, a branch of the mandibular division of the trigeminal nerve (CN V3), supplies motor innervation to the anterior belly of the digastric muscle and the mylohyoid muscles. The mylohyoid nerve branches off the inferior alveolar nerve prior to the latter nerve entering the mandibular foramen to supply sensory innervation to the inferior dentition. Cutting the mylohyoid nerve would not result in numbness on the tip of the tongue, but it would result in paralysis of the mylohyoid and anterior belly of the digastric muscles. Choice C (Inferior alveolar nerve) is incorrect. The inferior alveolar nerve, a branch off the mandibular division of the trigeminal nerve (CN V3), enters the mandibular foramen to supply the lower teeth, periosteum, and gingivae of the mandible. The mental nerve, a terminal branch of the inferior alveolar nerve, supplies the skin and mucosa of the lower lip and chin. Though the inferior alveolar nerve may be dam- aged during this procedure, it is not responsible for giving general sensation to the anterior aspect of the tongue. Choice E (Glossopharyngeal nerve) is incorrect. The lingual branch of the glossopharyngeal nerve (CN IX) enters the tongue to pro- vide sensory and taste information to the posterior one third of the tongue. It also gives off a tonsillar branch, which gives sensation to the mucosa of the oropharynx, including the palatine tonsil. This nerve would not be responsible for the numbness on the tip of the tongue described in this patient.

A 23-year-old female professional student wakes up with a facial nerve (CN VII or Bell) palsy. What muscle will continue to function despite this affliction? (A) Zygomaticus major (B) Levator labii superioris (C) Buccinator (D) Masseter (E) Platysma

The answer is D: Masseter. Damage to the facial nerve would lead to loss of innervation to the muscles of facial expression, and the masseter muscle, a muscle of mastication, is the only listed muscle that will continue to function in a patient diagnosed with facial nerve (CN VII or Bell) palsy. The mandib- ular (third) division of the trigeminal nerve (CN V3) supplies the four muscles of mastication (masseter, temporalis, lateral pterygoid, and medial pterygoid) and four additional muscles: Mylohoid, Anterior belly of the Digastric, Tensor Tympani, and Tensor Veli Palatini (mnemonic = "MATT"). The masseter muscle primarily works to close the jaw, though its superficial fibers may play a limited role in protrusion of the mandible. It is the only muscle on this list of options that would con- tinue to function in facial nerve palsy. Choice A (Zygomaticus major) is incorrect. The zygomaticus major is a muscle of facial expression, so it would be paralyzed in facial nerve palsy. It functions as a dilator of the oral fissure by elevating the corners of mouth, as in smiling when the muscle contracts bilaterally or sneering to show disdain when the muscle contracts unilat- erally. It originates on the lateral aspect of the zygomatic bone, which is how it receives its name. Choice B (Levator labii superioris) is incorrect. The levator labii superioris is a muscle of facial expression, so it would be paralyzed in facial nerve palsy. It functions as a dilator of the oral fissure by retracting (elevating) the upper lip to show the upper teeth and deepens the nasolabial sulcus. It originates on the infraorbital margin of maxilla, above, and therefore covers, the infraorbital foramen. Choice C (Buccinator) is incorrect. The buccinator is a muscle of facial expression, so it would be paralyzed in facial nerve palsy. It originates on the alveolar ridges of maxillary and man- dibular molar teeth and contracts to give tension to the cheek to keep food between the occlusal surfaces of the teeth. The tone of the buccinator muscle provides resistance to keep teeth from tilting laterally and prevents patients from looking like a hamster, with food lodged in the oral vestibule, when they chew food. Choice E (Platysma) is incorrect. The platysma is a muscle of facial expression, so it would be paralyzed in facial nerve palsy. It resides in the neck and lower face to depress the mandible and wrinkle the skin of neck, as seen when a person is placed in a stressful situation. The platysma originates in the subcutaneous tissue near the clavicle and inserts into the modiolus, lateral to the labial commissures

During a mixed martial arts (MMA) fight, one fighter punched his opponent in the anterior neck, resulting in a fracture of the hyoid bone. Which of the following muscles would be most directly affected by this injury? (A) Palatopharyngeus (B) Stylopharyngeus (C) Superior pharyngeal constrictor (D) Middle pharyngeal constrictor (E) Inferior pharyngeal constrictor

The answer is D: Middle pharyngeal constrictor. Fracture of the hyoid bone can be accomplished by blunt trauma to the anterior neck region. Because it is the attachment site for 10 muscles in the neck, pharynx, and oral floor, loss of move- ment of the hyoid may impair swallowing and normal separa- tion of the GI and respiratory passages. The three pharyngeal constrictor muscles form a circular series that collectively attach in an overlapping fashion along the posteromedian raphe of the pharynx. However, each has a distinctly separate origin. The middle constrictor arises from the greater and lesser horns of the hyoid bone. Thus, fracture of the hyoid disrupts the com- pressive action of the middle pharyngeal constrictor muscle in swallowing (or deglutition). Choice A (Palatopharyngeus) is incorrect. The palatopharyngeus muscle descends from the palate to attach into the pharyngeal wall and thyroid cartilage. It constricts the nasopharynx and elevates the pharynx and larynx during swallowing. It does not have a direct connection to the hyoid bone and would be only peripherally affected in this patient. Choice B (Stylopharyngeus) is incorrect. The sty- lopharyngeus muscle is the only skeletal muscle derived from the mesoderm of the third pharyngeal arch and innervated by the glossopharyngeal nerve (CN IX). It arises from the styloid process, descends through the pharyngeal wall between the superior and middle pharyngeal constrictors, and inserts into the thyroid cartilage to assist in elevating the pharynx and lar- ynx during swallowing. It does not have a direct connection to the hyoid bone and would be only peripherally affected in this patient. Choice C (Superior pharyngeal constrictor) is incorrect. The superior pharyngeal constrictor muscle arises from the pterygoid region on the base of the skull, the ptery- gomandibular raphe, and the mandible. It acts to compress the upper pharynx during swallowing. However, this muscle does not have a direct connection to the hyoid bone and would be only peripherally affected in this patient. Choice E (Inferior pharyngeal constrictor) is incorrect. The inferior pharyngeal constrictor muscle originates from the thyroid and cricoid car- tilages of the larynx to compress the lower pharynx during swallowing. This muscle does not have a direct connection to the hyoid bone and would be only peripherally affected in this patient.

As a result of facial injuries suffered in an automobile accident, a 17-year-old girl is unable to close her lips together tightly. Which of the following muscles is paralyzed? (A) Zygomaticus major (B) Buccinator (C) Levator labii superioris (D) Orbicularis oris (E) Mentalis

The answer is D: Orbicularis oris. The orbicularis oris is a muscle of facial expression (mimetic muscle), and these mus- cles insert or originate in the subcutaneous tissue of the skin, which enables them to convey mood via facial expressions as well as alter the form of the facial orifices. All the muscles of facial expression are derived from the mesoderm of the second pharyngeal (hyoid) arch and innervated by the facial nerve (CN VII). Most of these muscles are located within the face; however, a few extend into the scalp (e.g., epicranius) or the neck (platysma). The most important, and therefore most clin- ically relevant, role of the facial muscles is to control and oper- ate the facial orifices, that is, the eyes, nose, mouth, and ears. Facial expression is a secondary byproduct of the ability to finely control the facial orifices. The mouth is controlled by an extensive, interweaving array of facial muscles that influence feeding, respiration, and articulate speech. The orbicularis oris is a broad, very complex muscle that encircles the mouth in a sphincter-like fashion. It interacts with the other orofacial muscles to modify the form and tension of the lips and their margins. By itself, it acts to close the mouth by bringing the lips together tightly. Choice A (Zygomaticus major) is incor- rect. The zygomaticus major is an elongated muscle of facial expression that originates on the lateral aspect of the zygomatic bone and attaches into the angle of the mouth. It contracts to dilate oral fissure, but it also elevates the labial commissures bilaterally to smile (show happiness) or unilaterally to sneer (show disdain). Choice B (Buccinator) is incorrect. The buc- cinator (L: trumpeter) muscle is a wide, thin muscle of facial expression that lies relatively deep in the cheek, coursing from the pterygopalatine raphe and the alveolar ridges of maxil- lary and mandibular molar teeth to insert into the orbicularis oris at the angle of the mouth. The buccinator compresses the cheeks and lips against the teeth and gums, provides resis- tance to keep the teeth from tilting laterally, and prevents patients from looking like a hamster when they chew food. This muscle is important in all phases of feeding. During mas- tication, it assists in positioning food between the occlusal surfaces of the teeth. In suckling, it creates pressure within the oral cavity. This pressure also serves in blowing air, as when playing a wind instrument. Choice C (Levator labii superio- ris) is incorrect. The levator labii superioris is a relatively large muscle of facial expression that runs from the infraorbital margin of the maxilla into the upper lip. Acting in concert with its neighbors, it dilates the oral fissure by elevating and everting the upper lip to show the upper (maxillary) teeth. It also deepens nasolabial sulcus to show sadness. Please note that this muscle covers the infraorbital foramen, lying over the emerging infraorbital nerve and vessels. Choice E (Mentalis) is incorrect. The mentalis is a small, conical muscle of facial expression that originates on the incisive fossa of the mandible and inserts into the skin of the chin and the base of the lower lip. When pouting, this muscle raises, protrudes, and everts the lower lip. This muscle also elevates the skin of the chin to show doubt.

In the process of removing cervical lymph nodes during a radical neck dissection, a surgeon mistakenly lesions the ansa cervicalis. Which of the following deficits may occur? (A) Decreased blood flow to the larynx (B) Lymphedema in the carotid triangle of the neck (C) Reduced sensation in the skin over the posterior triangle of the neck (D) Paralysis of several infrahyoid (strap) muscles (E) Paralysis of the intrinsic laryngeal muscles

The answer is D: Paralysis of several infrahyoid (strap) muscles. The ansa cervicalis is a motor nerve loop derived from the cervical plexus. It is formed by the union of its two parts: the superior root (descendens hypoglossi, from C1) and the inferior root (descendens cervicalis, from C2 and C3). This delicate structure normally lies on the superficial surface of the carotid sheath, with its inferior margin at about the level of the cricoid cartilage. The ansa cervicalis innervates most of the infrahyoid (strap) muscles (omohyoid, sternohyoid, sternothyroid). The exception is the thyrohyoid muscle being supplied by C1 via the hypoglossal nerve. Choice A (Decreased blood flow to the larynx) is incorrect. The ansa cervicalis is a motor nerve route to skeletal muscles in the neck. It does not carry autonomic fibers that influence vaso- motor control. These fibers are derived from the cervical sympathetic chain. Choice B (Lymphedema in the carotid tri- angle of the neck) is incorrect. The ansa cervicalis is located mainly in the carotid triangle. However, it does not regulate lymphatic flow. Removal of lymph nodes in this region may produce some degree of lymphedema due to removal of the lymphatic channels, but that result is not related to lesion of the ansa cervicalis. Choice C (Reduced sensation in the skin over the posterior triangle of the neck) is incorrect. The ansa cervicalis does not convey cutaneous sensation. The cutane- ous branches of the cervical plexus emerge together from under the posterior border of the sternocleidomastoid muscle, at a point termed the "punctum nervosum" (nerve point of the neck). From this location, the cutaneous nerves of the cervical plexus distribute across the skin of the anterior and posterior triangles of the neck, posterior scalp, lower face, and ante- rior shoulder. Choice E (Paralysis of the intrinsic laryngeal muscles) is incorrect. The intrinsic muscles controlling the larynx are all supplied by branches of the vagus nerve (CN X). Most are innervated by the recurrent laryngeal nerve of CN X. The cricothyroid muscle is supplied by the external laryngeal nerve off the superior laryngeal nerve of CN X.

Holoprosencephaly is a complex of developmental abnormalities characterized by the loss of midline structures to greater or lesser degrees. An infant suffers multiple aspects of this disorder, including the midline cleft indicated. This specific developmental malformation is termed "premaxillary agenesis," developmental failure of formation of the intermaxillary segment of the face. Which of the following structures is most likely to be affected in this condition? (A) Nasal bones (B) Soft palate (C) Inferior nasal conchae (D) Philtrum (E) Mandibular incisors

The answer is D: Philtrum. The intermaxillary segment of the face is derived from the fused medial nasal processes of the facial primordia. It is the midline segment of the upper jaw and is composed of three parts: (1) the philtrum of the upper lip, (2) the premaxilla, and (3) the primary palate. The philtrum is the midline section of the upper lip that normally appears as a small fossa directly under the nasal aperture. The premaxilla is the bony segment that carries the four upper incisors. The primary palate is the small triangular shelf of bone directly posterior to the upper incisors. Its apex is the incisive foramen. Anterior palatal clefts typically lie along one or both lateral edges of the intermaxillary segment, where the medial nasal process would have fused with the maxillary process. Developmental failure of the medial nasal processes to merge causes a midline defect that may include absence of the entire intermaxillary segment (premaxillary agenesis). Choice A (Nasal bones) is incorrect. The nasal bones are derived from the frontonasal process primordial element, per- haps including the medial nasal processes. However, the nasal bones are not components of the intermaxillary segment of the face forming parts of the upper jaw. Choice B (Soft palate) is incorrect. The soft palate is formed from the posterior por- tion of the secondary palate. The secondary palate is derived from the palatine shelves of the maxillary processes. Posterior palatal clefts are located posterior to the intermaxillary seg- ment and may include defects in the soft palate. Choice C (Inferior nasal conchae) is incorrect. These delicate bones are located on the inferolateral sides of the nasal cavities. Each is derived from the maxillary process, which also gives rise to the maxillary and zygomatic bones in the face and the lat- eral part of the upper lip, lateral to the philtrum. Choice E (Mandibular incisors) is incorrect. These teeth are located in the lower jaw. However, the upper incisors are held in the premaxillary part of the upper jaw and would be affected by premaxillary agenesis.

A 68-year-old woman with thyroid cancer undergoes a total thyroidectomy. Postoperatively, the surgeon notes hoarseness and dysphonia, or altered voice production, while conversing with the patient. What nerve was damaged during the thyroidectomy? (A) Lingual branch of glossopharyngeal nerve (B) Accessory nerve (C) Superior laryngeal nerve (D) Recurrent laryngeal nerve (E) Hypoglossal nerve

The answer is D: Recurrent laryngeal nerve. The recurrent laryngeal nerve, a branch of the vagus nerve (CN X), passes posterior to the thyroid gland as it ascends in the neck to innervate most of the muscles of the larynx. Iatrogenic damage to the recurrent laryngeal nerve occurs in approximately 1% to 2% of thyroid surgeries and leads to the patient's hoarseness and dysphonia due to ipsilateral paralysis of all the musclesof the larynx, except the cricothyroid muscle. Choice A (Lingual branch of glossopharyngeal nerve) is incorrect. The lingual branch of the glossopharyngeal nerve (CN IX) provides sensation and taste to the posterior one third of the tongue. Damage to the lingual branch of the glossopharyngeal nerve (CN IX) would not cause the speech difficulties displayed in this patient. Choice B (Accessory nerve) is incorrect. The accessory nerve (CN XI) innervates the sternocleidomastoid and trapezius muscles, so damage to CN XI would cause an inability to shrug the shoulders and weakness turning the head to the contralateral side. These signs and symptoms were not seen in this patient. Choice C (Superior laryngeal nerve) is incorrect. Cutting the superior laryngeal nerve of the vagus nerve (CN X) produces numbness of the superior part of the larynx above the vestibular (false) vocal folds. It also inner- vates the cricothyroid muscle, a muscle of the larynx, involved in increasing the length of the true vocal folds. Though this muscle is important in increasing the pitch of the voice, it would not produce the hoarseness and dysphonia seen in this patient. Choice E (Hypoglossal nerve) is incorrect. The hypoglossal nerve (CN XII) is located in the anterior cervical region, and it innervates all of the intrinsic and most of the extrinsic muscles of the tongue, with the lone exception being the palatoglossus muscle, innervated by the vagus nerve (CN X). Cutting CN XII produces a speech impediment and devia- tion of the tongue to the ipsilateral side during protrusion due to atrophy of the tongue musculature. However, these deficits were not seen in this patient.

A paralyzed right true vocal fold is most likely associated with which of the following situations? (A) Repair of a patent ductus arteriosus (B) Repair of an aortic aneurysm (C) A gunshot wound below the second rib (D) Surgical removal of the thyroid gland (E) Obstruction of the thoracic duct

The answer is D: Surgical removal of the thyroid gland. The true vocal folds are controlled by the inferior laryngeal nerves, the terminal branches of the recurrent laryngeal nerves off the vagus nerve (CN X). The right recurrent laryngeal nerve branches from the right vagus nerve high in the superior mediastinum, loops under the right subclavian artery, and ascends through the neck along the tracheoesophageal groove. In its course, it runs on the deep aspect of the thyroid gland, where it is vulnerable to injury during thyroidectomy. Lesion of the recurrent laryngeal nerve paralyzes the ipsilateral intrin- sic laryngeal muscles (except the cricothyroid) and eliminates sensation below the true vocal fold. Choice A (Repair of a patent ductus arteriosus) is incorrect. The ductus arteriosus is a fetal shunt connecting the root of the left pulmonary artery with the inferior side of the arch of the aorta. It nor- mally closes soon after birth, becoming the ligamentum arte- riosum. However, it may remain patent after birth, especially in premature infants, and may require surgical repair. The left recurrent laryngeal nerve branches from the left vagus nerve at the inferior edge of the arch of the aorta, loops under the arch immediately lateral to the ductus arteriosus, and ascends through the neck on a pathway mirroring the right recurrent laryngeal nerve. Because of its close relation to the ductus arte- riosus, the left recurrent laryngeal nerve is vulnerable to injury during surgery related to the ductus arteriosus. Choice B (Repair of an aortic aneurysm) is incorrect. The aorta ascends from the heart, curves to the left to form the arch of the aorta, and descends into the thorax offset on the left side of the vertebral column. The left recurrent laryngeal nerve is closely related to the arch of the aorta from its origin to its initial ascent into the neck. It is vulnerable in an aneurysm of the aortic arch, where it may be stretched, and in surgi- cal repair of the aneurysm. Remember that the right recurrent laryngeal nerve is not closely related to the aorta. Choice C (A gunshot wound below the second rib) is incorrect. A pen- etrating wound directly into the right side of the chest is nor- mally too low to affect the right recurrent laryngeal nerve due to the nerve's high point of origin. However, a similar wound on the left side might affect the left recurrent laryngeal nerve. Choice E (Obstruction of the thoracic duct) is incorrect. The thoracic duct ascends through the thorax, diverges to the left side in the superior mediastinum, and joins the left venous angle (i.e., the junction of the left internal jugular and subcla- vian veins) in the root of the neck. It is not closely related to the right recurrent laryngeal nerve.

An 8-year-old boy comes to his physician with a painless and smooth mass located in the midline of his neck at the level of the hyoid bone. This palpable, midline neck mass was asymptomatic, but due to recent expansion, it has caused difficulty and pain when swallowing. When he swallows or protrudes his tongue, the mass moves superiorly. What is the most likely diagnosis? (A) Enlarged deep cervical lymph node (B) Thyroid nodule (C) Benign parathyroid adenoma (D) Thyroglossal duct cyst (E) Branchial cyst

The answer is D: Thyroglossal duct cyst. A thyroglossal duct cyst is a fibrous cyst that forms within the thyroglossal duct, so it is located in, or close to, the midline of the neck. Most commonly (∼50%), it is found near the body of the hyoid bone; however, the thyroglossal cyst in this patient is located inferior to this point. The thyroglossal cyst is usually painless and smooth, and this mass moves upward during swallowing or protrusion of the tongue, as reported in this patient. The thyroglossal duct is a remnant of the descent of the thyroid gland, which first appears as a single, ventromedian diver- ticulum (thyroid diverticulum) off the floor of the embryonic pharynx between the tuberculum impar and copula of the incipient tongue. As the thyroid gland descends along the midline, anterior to the gut tube, it remains connected to the tongue by a narrow canal called the thyroglossal duct. The thyroglossal duct usually solidifies and is obliterated after the final descent of the thyroid gland; however, if it persists, a thyroglossal cyst may develop, usually in individuals under the age of 20. Choice A (Enlarged deep cervical lymph node) is incorrect. The deep cervical lymph nodes lie within or in close proximity to the carotid sheath correlating in location with the internal jugular vein. While a few of the deep cervi- cal lymph nodes could reside near the location of the mass, they do not tend to be found superficially on the midline of the neck and an enlarged deep cervical lymph node would not lead to the dysphagia (or difficulty swallowing) seen in this patient. Choice B (Thyroid nodule) is incorrect. A thyroid nodule refers to any abnormal growth that forms within the thyroid gland. Adult women (4% to 8%) are particu- larly prone to thyroid nodules, but fortunately, only 10% of thyroid nodules are reported to be cancerous. The majority of thyroid nodules are asymptomatic; however, if the cells of the thyroid nodule are producing thyroid hormones, either thyroxine (T4) or triiodothyronine (T3), a thyroid nodule can lead to hyperthyroidism. Patients with hyperthyroidism may present with heart palpitations, weight loss, anxiety, insom- nia, fatigue, heat intolerance, excessive sweating, exophthal- mos (protruding eyes), and even amenorrhea (an absence of menstrual flow). Due to the midline location of this mass, lack of hyperthyroidism symptoms, and age of this patient, a thy- roglossal cyst is the most likely diagnosis. Choice C (Benign parathyroid adenoma) is incorrect. A parathyroid adenoma is a benign tumor of the parathyroid glands, which usually increases the circulation of parathyroid hormone (PTH). This condition of hyperparathyroidism leads to an increase in blood calcium levels due to elevated resorption of bone and may be asymptomatic in many patients. Symptomatic patients would present with lethargy, muscle pain, nausea, constipa- tion, confusion, kidney stones, and even an increased risk of bone fractures due to the increased bone resorption. A phy- sician can perform blood tests to test for calcium, chloride, potassium, and bicarbonate levels, and women over the age of 60 have the highest risk for developing hyperparathyroidism. The presence of a parathyroid adenoma has been reported in 80% to 85% of patients who present with hyperparathyroid- ism; however, the symptoms and age of this patient do not cor- relate with the presence of a parathyroid adenoma. Choice E (Branchial cyst) is incorrect. Branchial cysts are located along the anterior border of the sternocleidomastoid muscle. Most often, these cysts are the remnants of the second pharyngeal cleft, located just below the angle of the mandible. Second branchial cleft cysts represent approximately 67% to 93% of all pharyngeal apparatus anomalies. However, branchial cysts may be found anywhere along the anterior margin of the ster- nocleidomastoid muscle but not on the anterior midline of the neck. Very frequently, branchial cysts are inconspicuous at birth, becoming evident as they enlarge throughout child- hood. Due to the midline location of this mass, a branchial cyst can be ruled out in this patient.

A tumor growing at the base of the skull impinges upon the foramen spinosum, severely compressing its contents. Which of the following conditions is the most likely result? (A) Venous drainage from the base of the brain is obstructed (B) Mucus secretion in the oral floor is reduced (C) Sensation from the mandibular teeth is lost (D) Motor control of the upper pharynx is lost (E) Arterial supply to the dura mater is reduced

The answer is E: Arterial supply to the dura mater is reduced. The indicated opening is the foramen spinosum. This small foramen conveys the middle meningeal artery (a branch of the maxillary artery) and the spinous nerve (a branch of the mandibular division of the trigeminal nerve, or CN V3) from the infratemporal fossa into the cranium, in the floor of the middle cranial fossa. The middle meningeal artery provides the major blood supply to the dura mater and the cranial bones. It does not supply the brain. The middle meningeal artery is often involved with epidural hematomas. Choice A (Venous drainage from the base of the brain is obstructed) is incorrect. The main venous drainage of the brain is through the dural venous sinuses into the internal jugular veins, which exit the cranial cavity through the jugular foramina. This pri- mary drainage route is supplemented by several emissary veins that pass through various other openings in the skull, includ- ing the foramen ovale and carotid canal in the floor of the middle cranial fossa. However, the foramen spinosum does not normally carry emissary veins. Choice B (Mucus secretion in the oral floor is reduced) is incorrect. Mucus secretion in the oral floor is controlled by parasympathetic neurons within the chorda tympani nerve. The chorda tympani branches from the facial nerve in the facial canal, passes through the tympanic (middle ear) cavity, and leaves that space through a small opening (petrotympanic fissure) to enter the infratem- poral fossa. Choice C (Sensation from the mandibular teeth is lost) is incorrect. Afferent neurons from the mandibular teeth are carried in the inferior alveolar nerve, a branch of the mandibular division of the trigeminal nerve (CN V3). CN V3 passes through the foramen ovale in the base of the skull. The inferior alveolar nerve passes through the mandibular fora- men as it leaves the mandible conveying sensation from the mandibular teeth. Choice D (Motor control of the pharyngeal constrictor muscles is lost) is incorrect. Motor control of the pharyngeal constrictor muscles is from branches of the vagus nerve (CN X). The vagus exits the cranial cavity through the jugular foramen, in company with the glossopharyngeal (CN IX) and accessory (CN XI) nerves.

A 68-year-old man arrived at the ER with sudden onset of the worst headache of his life, lethargy, and nuchal rigidity. He quickly loses consciousness and dies. Autopsy reveals no traumatic injury; however, the man's subarachnoid space is filled with blood. Damage to what blood vessel most likely led to the death of this patient? (A) Common carotid artery (B) Middle meningeal artery (C) Facial artery (D) Superior cerebral veins (E) Cerebral arterial circle

The answer is E: Cerebral arterial circle. The cerebral arterial circle (of Willis) is an anastomoses of arteries located on the inferior surface of the brain in the area of the interpeduncu- lar fossa, optic chiasm, and hypothalamus. These vessels are prone to saccular (berry) aneurysm, particularly where the arteries join together. Upon rupturing, blood spills into the subarachnoid space, causing the subarachnoid hemorrhage seen in autopsy. Remember that all major vessels of the brain travel within the subarachnoid space, and subarachnoid hem- orrhages are one of the few cerebral hemorrhages that can occur in the absence of trauma, often seen following rupture of a saccular aneurysm within the cerebral arterial circle. The three cardinal signs and symptoms of a subarachnoid hemor- rhage are (1) loss of consciousness (lethargy), (2) nuchal rigid- ity (stiff neck), and (3) a sudden onset of the "worst headache of your life," which were all present in this patient. Choice A (Common carotid artery) is incorrect. The common carotid artery, which gives rise to the internal and external carotid arteries at the level of the fourth cervical vertebra, is a major artery of the neck that lies outside of the skull. If this artery were lacerated, it would not cause the sudden onset of the severe headache seen in this patient due to its extracranial location. Choice B (Middle meningeal artery) is incorrect. This artery, which supplies blood to the dura mater, is often torn following impact to the side of the head, fracturing the skull in the area of the pterion. The anterior division of the middle meningeal artery is at risk if the pterion, an osteologi- cal feature on the side of the head that marks the junction of the parietal, frontal, squamous temporal, and sphenoid bones, is fractured because this artery runs in close proximity to the pterion. A traumatic blow to the side of the head, and subse- quent fracture of the pterion, leads to an epidural hemorrhage as the blood pools between the endosteal layer of the dura mater and the calvaria. Because no trauma was reported and blood was not found in the epidural space, this option can be eliminated. Choice C (Facial artery) is incorrect. The facial artery lies outside of the skull. If this artery were lacerated, it would not cause the signs and symptoms, sudden onset of the worst headache of his life, lethargy, and nuchal rigidity, seen in this patient. Choice D (Superior cerebral veins) is incorrect. Traumatic impact to the front of the head can tear the supe- rior cerebral veins. The impact shears the superior cerebral veins as they empty into the superior sagittal sinus, leading to a subdural hematoma. Because no trauma was reported in this patient, this option can be eliminated.

A tumor is discovered embedded in the posterior wall of the tympanic cavity in a 45-year-old man. If the tumor erodes through this wall, which of the following structures will it first encounter? (A) Internal jugular vein (B) Tympanic membrane (C) Internal carotid artery (D) Brain (E) Facial nerve

The answer is E: Facial nerve. The tympanic (middle ear) cavity is a small air-filled space within the petrous part of the temporal bone. Its position, shape, and relations make for challenging spatial concepts in anatomy because it is buried so deeply within the skull. It has six walls: the roof (tegmental wall), floor (jugular wall), anterior (carotid) wall, posterior (mastoid) wall, medial (labyrinthine) wall, and lateral (membranous) wall. Each wall has a close relationship to one or more significant neighboring structures. The upper part of the posterior wall contains an opening (aditus) that leads to the mastoid antrum and air cells. The distal limb of the facial canal (containing the main branch of the facial nerve) descends below the aditus, behind the posterior wall, on its way to its termination at the stylomastoid foramen. Thus, a tumor pierc- ing the posterior wall may invade the facial canal and/or the mastoid air sinuses. Choice A (Internal jugular vein) is incor- rect. The superior bulb of the internal jugular vein lies beneath the floor (jugular wall) of the tympanic cavity. Because this tumor is embedded in the posterior wall of the middle ear cav- ity, the internal jugular vein would not be affected. Choice B (Tympanic membrane) is incorrect. The tympanic membrane (eardrum) forms most of the lateral (membranous) wall. The epitympanic recess, situated above the tympanic membrane, completes the lateral wall. The handle of the malleus and the crossing chorda tympani nerve lie against the inside of the tympanic membrane. Because this tumor is embedded in the posterior wall of the tympanic cavity, the tympanic membrane would not be affected. Choice C (Internal carotid artery) is incorrect. The carotid canal (containing the internal carotid artery) lies outside the anterior (carotid) wall. Also, the phar- yngotympanic (auditory) tube and the canal for the tensor tympani muscle open into the upper part of the anterior wall. Because this tumor is embedded in the posterior wall of the tympanic cavity, the internal carotid artery would not be affected. Choice D (Brain) is incorrect. The roof (tegmental wall) of the tympanic cavity is the tegmen tympani (roof of tympanum). This thin bony layer forms part of the floor of the middle cranial fossa, separating the dura mater and temporal lobe of the brain from the middle ear. Thus, fractures here may result in leakage of cerebrospinal fluid into the tympanic cav- ity and subsequently into the nasopharynx (via the pharyngo- tympanic tube) or external acoustic meatus (if the tympanic membrane is ruptured). The sixth wall (medial; labyrinthine) separates the middle ear from the internal ear. Its main fea- tures are the promontory, oval window, and round window.

A 78-year-old man presents with signs of reduced blood flow into the right side of his face. His physician wishes to take a pulse of the facial artery on both sides to help evaluate the situation. The pulse of the facial artery can be readily palpated at which of the following locations? (A) Lateral side of the body of the hyoid bone (B) Inferior edge of the zygomatic arch (C) Apex of the zygomatic bone (D) Lateral surface of the nasal bone (E) Inferior margin of the body of the mandible

The answer is E: Inferior margin of the body of the mandible. After originating from external carotid artery, the facial artery ascends in the upper neck deep to the posterior belly of the digastric and stylohyoid muscles and the submandibular gland. The facial artery then crosses the body of the man- dible anterior to the masseter muscle to enter the face. Its pulse can be readily palpated at the point where it crosses the inferior margin of the body of the mandible, at the ante- rior border of the masseter muscle. The artery continues ascending across the face, taking a tortuous (or winding) path relatively close to the angle of the mouth. The folding of the vessel allows it to accommodate being stretched dur- ing wide opening of the mouth. It gives branches to the lips and side of the nose, finally terminating at the medial can- thus (angle) of the eye. Choice A (Lateral side of the body of the hyoid bone) is incorrect. The facial artery arises from the external carotid artery in the upper neck. It courses superior to the hyoid bone. Choice B (Inferior edge of the zygomatic arch) is incorrect. The transverse facial artery arises within the parotid gland as a small branch of the superficial temporal artery. As it runs anterior, the transverse facial artery crosses the masseter muscle and is located just above the parotid duct and below the zygomatic arch. Choice C (Apex of the zygo- matic bone) is incorrect. This bony feature is the point of the "cheek bone." No significant vessels reside at this location, so taking a pulse at the apex of the zygomatic bone is not pos- sible. Choice D (Lateral surface of the nasal bone) is incorrect. As it ascends across the face, that facial artery gives branches to the ala and side of the nose (lateral nasal artery) and its ter- minal branch to the medial angle (canthus) of the eye (angular artery). However, these vessels are small and do not normally give pulse points.

A physician directs a small light into only the left eye of a patient to test pupillary constriction. The left pupil does not respond to the light; however, the right pupil constricts. What nerve is most likely damaged in this patient? (A) Right optic nerve (B) Left optic nerve (C) Right ophthalmic nerve (D) Right oculomotor nerve (E) Left oculomotor nerve

The answer is E: Left oculomotor nerve. The left oculomotor nerve (CN III) is damaged in this patient. The physician is performing the pupillary light reflex, which tests the integrity of the sensory and motor functions of the eye. The afferent limb of the reflex is the optic nerve, and the efferent limb is the oculomotor nerve. In this patient, the constriction of the right pupil (a consensual response to the light) implies the affer- ent limb (left optic nerve) of the light reflex is intact because one of the pupils responded to the light. However, the effer- ent limb of the left eye is likely damaged due to the lack of a direct response (pupillary constriction) to the light. Under normal circumstances, both pupils constrict in response to increased light intensity due to a bilateral projection from the pretectal nucleus within the upper medulla to the Edinger- Westphal nucleus, which then projects its parasympathetic fibers along the oculomotor nerve causing pupillary constric- tion. Choice A (Right optic nerve) is incorrect. Because the physician did not direct the light into the right eye, the integ- rity of the right optic nerve was not tested. To test this afferent limb of the pupillary light reflex, the physician must shine the light directly into the right eye. Because there was no direct response (pupillary constriction) of the left eye, the physician did confirm damage to the left oculomotor nerve. Choice B (Left optic nerve) is incorrect. The left optic nerve is intact in this patient because the right pupil constricted (a consensual response), which implies the patient's left optic nerve recog- nized the increase in light intensity. Choice C (Right ophthal- mic nerve) is incorrect. The ophthalmic nerve (or first division of the trigeminal nerve) supplies sensory (cutaneous) innerva- tion to the skin of the upper eyelid, cornea, anterior aspect of the nose, forehead, and anterior scalp. This nerve did not participate in the pupillary light reflex, so this option can be eliminated. Choice D (Right oculomotor nerve) is incorrect. The right oculomotor nerve is intact because the right pupil constricted in response to the increased luminescence. The bilateral projections within the pupillary light response path- way enable this consensual response.

A 38-year-old woman comes to her family physician complaining of repeatedly tripping and double vision when descending stairs. While testing the eye movements in a cranial nerve examination, she was unable to move her left eye inferiorly when she followed the physician's finger to her right side. What specific nerve is most likely damaged? (A) Left oculomotor nerve (B) Right oculomotor nerve (C) Left abducent nerve (D) Right abducent nerve (E) Left trochlear nerve

The answer is E: Left trochlear nerve. The left trochlear nerve innervates only one muscle, the superior oblique, located in the orbit. Acting individually, this muscle pulls the eye inferolaterally, but it would be hard to distinguish this movement from the combined movements of the inferior rectus muscle (innervated by the oculomotor nerve) and the lateral rectus muscle (innervated by the abducent nerve), which also move the globe inferior and lateral, respectively. Therefore, the left trochlear nerve is clinically tested by asking the patient to look inferiorly after the left eye is placed in an adducted position. Due to its course and involvement with the trochlea of the superior oblique muscle, this extraocular muscle is the only muscle that can strongly direct the gaze of the adducted eye inferiorly. Damage to this nerve also explains her diplopia, or double vision, and clumsiness when descending stairs. Choice A (Left oculomotor nerve) is incorrect. The left oculomotor nerve, CN III, innervates most of the extraocular eye muscles (with the exception of superior oblique and lateral rectus muscles) and the levator palpebrae superioris (which elevates the eyelid) in the ipsilateral (left) eye. Damage to the left CN III leads to diplopia (due to the left pupil resting in an abducted and lateral position) and ptosis (drooping of the eyelid) associated with the left eye. Choice B (Right oculomotor nerve) is incorrect. The right oculomotor nerve, CN III, innervates most of the extraocular eye muscles (with the exception of superior oblique and lateral rectus muscles) and the levator palpebrae superioris (which elevates the eyelid) in the ipsilateral (right) eye. Damage to the right CN III leads to diplopia (due to the right pupil resting in an abducted and lateral position) and ptosis (drooping of the eyelid) associated with the right eye. However, abnormal positioning of the resting pupil and ptosis were not seen in this patient. Choice C (Left abducent nerve) is incorrect. The only extraocular muscle innervated by the left abducent nerve, CN VI, is the left lateral rectus muscle, which directs the left globe laterally. If the left abducent nerve were damaged, the patient would display diplopia, and the left pupil would be resting in the adducted position due to the unopposed action of the other extraocular muscles. Choice D (Right abducent nerve) is incorrect. The only extraocular muscle innervated by the right abducent nerve, CN VI, is the right lateral rectus muscle, which directs the right globe laterally. If this nerve were damaged, the patient would display diplopia, and the right pupil would be resting in the adducted position due to the unopposed action of the other extraocular muscles. Because extraocular eye movements were affected in the left eye of this patient, this option can be easily eliminated.

A 46-year-old woman was cut on the right side of her face by a window that shattered. Her laceration was located at the anterior border of the inferior part of the masseter muscle. When she returns to her doctor to have the stitches removed, her physician notes asymmetry in her lower lip when she grimaces. Damage to what nerve would cause the facial asymmetry seen in this patient? (A) Zygomatic branch of facial nerve (B) Temporal branch of facial nerve (C) Cervical branch of facial nerve (D) Buccal branch of facial nerve (E) Marginal mandibular branch of facial nerve

The answer is E: Marginal mandibular branch of facial nerve. The marginal mandibular branch of the facial nerve (CN VII) is one of five terminal branches of the main trunk of CN VII that supply the muscles of facial expression. This nerve sup- plies muscles of the lower lip and chin, including the depres- sor anguli oris, depressor labii inferioris, and mentalis muscles. The loss of muscular tone in the lower lip of this patient is causing asymmetry in her facial expression. Therefore, the marginal mandibular branch of the facial nerve was damaged by the broken glass. Remember, the main trunk of the facial nerve (CN VII) branches into five terminal branches, named in order: (1) Temporal, (2) Zygomatic, Buccal, Marginal mandib- ular, and Cervical; they can be remembered by the mnemonics "Two Zebras Bit My Cheek" or "To Zanzibar By Motor Car". Choice A (Zygomatic branch of facial nerve) is incorrect. The zygomatic branch of the facial nerve (CN VII) innervates pri- marily the inferior part of the orbicularis oculi muscle to close the lower eyelid ipsilaterally. In this patient, asymmetry in the lower lip was noted, so the zygomatic branch of the facial nerve was not injured. Choice B (Temporal branch of facial nerve) is incorrect. The temporal branch of the facial nerve (CN VII) innervates muscles that move the auricle as well as the frontalis muscle and the superior part of the orbicularis oculi. The fron- talis muscle raises the eyebrows and wrinkles the skin of the forehead. The orbicularis oculi acts as the sphincter of the orbit by closing the eyelids to cover the eye. In this patient, asym- metry in the lower lip was noted, so the temporal branch of the facial nerve was not injured. Choice C (Cervical branch of the facial nerve) is incorrect. The cervical branch of the facial nerve (CN VII) innervates the platysma muscle, which wrinkles the skin of the inferior face and neck, which is often seen when a person is under stress. In this patient, asymmetry in the lower lip was noted, so the cervical branch of the facial nerve was not injured. Choice D (Buccal branch of facial nerve) is incorrect. The buccal branch of facial nerve (CN VII) supplies the buc- cinator, orbicularis oris, and muscles of upper lip. Damage to this nerve results in the loss of muscular tone within the cheek and an inability to elevate the labial commissure, purse the upper lip, or show the upper teeth ipsilaterally. In this patient, asymmetry in the lower lip was noted, so the buccal branch of the facial nerve was not injured.

A 32-year-old man presents with unilateral paralysis of the muscles of mastication on the right side. This condition has resulted in facial asymmetry. Though he is uncomfortable with his appearance and has difficulty when chewing his food, his chief complaint is his difficulty swallowing (dysphagia). What muscle is most likely involved in his dysphagia? (A) Sternohyoid (B) Stylohyoid (C) Cricothyroid (D) Stylopharyngeus (E) Mylohyoid

The answer is E: Mylohyoid. The mylohyoid muscle is inner- vated by the mandibular division of the trigeminal nerve (CN V3), and this nerve is damaged in this patient due to the right-sided paralysis of the muscles of mastication, facial asymmetry (noted in the figure), and dysphagia. The man- dibular division of the trigeminal nerve (CN V3) is the only division of the trigeminal nerve (CN V) that supplies motor innervation. It supplies the muscles derived from the meso- derm of the first pharyngeal arch, including the four muscles of mastication (temporalis, masseter, lateral pterygoid, and medial pterygoid) and four additional muscles: Mylohoid, Anterior belly of the Digastric, Tensor Tympani, and Tensor Veli Palatini (mnemonic = "MATT"). The mylohyoid muscle is a suprahyoid muscle that depresses the mandible against resistance when the infrahyoid muscles fix or depress the hyoid bone. Movement of the hyoid bone is crucial during swallowing (or deglutition), so paralysis of the mylohyoid and anterior belly of the digastric muscles causes dysphagia in a patient with damage to CN V3. Choice A (Sternohyoid) is incorrect. The sternohyoid is innervated by the ansa cervicalis from the cervical plexus (C1-3). This muscle would not be affected by paralysis of the muscles of mastication, which is caused by damage to the mandibular division of the trigemi- nal nerve (CN V3). The sternohyoid muscle is an infrahyoid (strap) muscle that anchors and depresses the hyoid bone, so paralysis of this muscle may cause dysphagia. However, this muscle is not the best selection, as it is not innervated by the mandibular division of the trigeminal nerve (CN V3). Choice B (Stylohyoid) is incorrect. The stylohyoid is a suprahyoid mus- cle that fixes and depresses the hyoid bone during swallow- ing, so losing this muscle may cause dysphagia noted in this patient. However, the patient presents with unilateral paralysis of the muscles of mastication on the right side, which implies damage to the mandibular division of the trigeminal nerve (CN V3). Damage to CN V3 would not involve the stylohyoid muscle because this muscle is derived from the mesoderm of the second pharyngeal (hyoid) arch and is therefore inner- vated by the facial nerve (CN VII). Choice C (Cricothyroid) is incorrect. The cricothyroid muscle is innervated by the external branch of the superior laryngeal nerve (from CN X), and it acts in lengthening and tensing the vocal ligament by pulling the thyroid cartilage anteroinferiorly. Paralysis of this muscle would change the pitch of the patient's voice but would not cause the dysphagia seen in this patient. Choice D (Stylopharyngeus) is incorrect. The stylopharyngeus muscle is innervated by the glossopharyngeal nerve (CN IX). Loss of this muscle's function would affect deglutition (swallowing) as it elevates the pharynx, but it is not the best choice due to its innervation by the glossopharyngeal nerve (CN IX). More- over, it would not be affected by damage to the mandibular division of the trigeminal nerve (CN V3).

While planning a delicious dinner for his former anatomy professors, a doctor finds himself salivating at the thought of the feast. What description accurately describes the secretomotor pathway for innervation of the submandibular gland? (A) Parasympathetic fibers via the inferior alveolar nerve (B) Sympathetic fibers via the lingual nerve (C) Parasympathetic fibers via the mandibular branch of the facial nerve (D) Sympathetic fibers via the hypoglossal nerve (E) Parasympathetic fibers via the chorda tympani nerve

The answer is E: Parasympathetic fibers via the chorda tym- pani nerve. All the salivary glands receive their secretomotor innervation from the parasympathetic nervous system. Both the submandibular and sublingual salivary glands are sup- plied by the facial nerve by way of its chorda tympani branch. The chorda tympani nerve originates from the distal part of the main trunk of the facial nerve and runs across the middle ear cavity. It earns its name at this location by its thread-like form lying across the deep surface of the tympanic mem- brane and the handle of the malleus. The nerve then enters the infratemporal fossa and merges with the lingual nerve (a branch of the mandibular division of the trigeminal nerve, or CN V3). The parasympathetic fibers synapse in the sub- mandibular ganglion, with the postsynaptic fibers passing to the submandibular and sublingual salivary glands and to mucus glands in the oral floor. Remember, the chorda tympani nerve also carries taste fibers from the anterior two thirds of the tongue. The remaining salivary gland, the parotid, is sup- plied by the glossopharyngeal nerve, by way of the following pathway: tympanic nerve/lesser petrosal nerve/otic ganglion/ auriculotemporal nerve. Choice A (Parasympathetic fibers via the inferior alveolar nerve) is incorrect. The inferior alveolar nerve is a branch of the mandibular division of the trigeminal nerve (CN V3) within the infratemporal fossa. It gives off the mylohyoid nerve, which innervates the mylohyoid and ante- rior digastric muscles, and then enters the mandible traversing the mandibular foramen. Most of its distribution is concerned with general sensation in the lower jaw (including the teeth and gums) and the skin overlying the mandible, including the lower lip. It has no direct role in secretion of any salivary gland. Choice B (Sympathetic fibers via the lingual nerve) is incorrect. Sympathetic neurons do not supply the salivary glands. The lingual nerve does carry autonomic fibers to the submandibular and sublingual salivary glands. However, this autonomic innervation is derived from parasympathetic fibers of the facial nerve/chorda tympani nerve pathway. Choice C (Parasympathetic fibers via the mandibular branch of the facial nerve) is incorrect. The (marginal) mandibular branch of the facial nerve is one of the five terminal branches of the main trunk of the facial nerve innervating the muscles of facial expression. The (marginal) mandibular branch of the facial nerve does not carry autonomic fibers to any salivary glands despite the fact that this nerve transverses the parotid gland. Choice D (Sympathetic fibers via the hypoglossal nerve) is incorrect. Again, the sympathetic division does not supply the salivary glands. The hypoglossal nerve may carry a small hitchhiking sympathetic bundle for part of its course. How- ever, its dominant content is motor fibers to the intrinsic and most of the extrinsic skeletal muscles of the tongue.

A 17-year-old woman presents with an anterior dislocation of her temporomandibular joint (TMJ). With her mandible stuck in the protruded (protracted) position, her dentist pulls the mandible inferiorly to enable the tone of a muscle to retrude (retract) the mandible to its normal position. Which muscle returns the mandibular condyle back into its normal position after it clears the articular eminence? (A) Lateral pterygoid (B) Masseter (C) Medial pterygoid (D) Stylohyoid (E) Temporalis

The answer is E: Temporalis. The posterior fibers of the tem- poralis muscle run in a horizontal plane, and during contrac- tion, enable retrusion (or retraction) of the mandible allow- ing it to return to its normal position in the TMJ. By pulling the mandible down, the dentist frees the mandibular condyle from its stuck position on the articular eminence of the tempo- ral bone. The temporalis muscle is also involved with closing of the mouth (elevation of the mandible); however, its poste- rior, horizontally oriented fibers are the primary retractors of the mandible. Choice A (Lateral pterygoid) is incorrect. The lateral pterygoid muscle acts on the TMJ to allow protrusion (or protraction) of the mandible. During contraction of the lat- eral pterygoid muscle, the mandibular condyle slides anterior (translation) to be located inferior to the articular eminence of the temporal bone, which enables the mouth to open due to the effects of gravity. It has no role in the retraction of the mandible. In this patient, the tone of the temporalis muscle returned the mandibular condyle to the TMJ. Choice B (Masseter) is incor- rect. The masseter muscle primarily works to close the jaw, but its superficial fibers also play a limited role in protrusion of the mandible. The vertical (deep) fibers of this muscle may also play a limited role in the retrusion of the mandible; however, the temporalis muscle is the prime mover in retrusion at the TMJ. Choice C (Medial pterygoid) is incorrect. The medial pterygoid muscle elevates the mandible and contributes to protrusion. Therefore, it does not function in the retraction of the mandible, which is the action that returned the mandibular condyle to the TMJ in this patient. Choice D (Stylohyoid) is incorrect. The stylohyoid is a suprahyoid muscle that depresses the mandible against resistance when the infrahyoid muscles have the hyoid bone fixed in position. It would not be involved in correcting the anterior dislocation of the TMJ seen in this patient.


Kaugnay na mga set ng pag-aaral

The Reign of Louis XIV chapter 21 section 2

View Set

Chapter 2: Ultrasound Transducers

View Set

Jawetz ch. 33 & 44: HIV, Herpesviruses

View Set

EAQ ch 66- Care of Patients with Urinary Problems

View Set

Chapter 6 What is a Business Plan

View Set

Medical Terminology Midterm Exam (Q&A)

View Set

Organizational Behavior Final Exam

View Set